Psych Exam 1 Review

Réussis tes devoirs et examens dès maintenant avec Quizwiz!

Axis I (DSM IV-TR)

*Major Psychiatric Diagnosis* - Bipolar Disorder - Schizophrenia - Generalized Anxiety Disorder - Alcohol Dependence - Major Depression - Post-Traumatic Stress Disorder

A patient with acute depression states, "God is punishing me for my past sins"/ What is the nurses most therapeutic response a) you sound very upset about this b) God always forgives us for our sins c) why do you think you are being punished d) if you feel this way, you should talk to your minister

A

APRN-PMH (advanced practice registered nurse-psychiatric mental health)

A nurse generalist who has obtained additional training to provide care as a clinical nurse specialist with advanced nursing expertise or as a nurse practitioner who -diagnoses - prescribes - treats psychiatric disorders

cultural competence

An understanding of how a patient's cultural background shapes his beliefs, values, and expectations for therapy.

Which principle should guide the nurse in determining the extent of silence to use during patient interview sessions a) a nurse is responsible for breaking silences b) patients withdraw is silences are prolonged c) silence can provide meaningful moments for reflection d) silence helps patients know that what they said was understood

C

A nursing student new to psychiatric mental health nursing asks a peer what resources he can use to figure out which symptoms are present in a specific psychiatric disorder. The best answer would be: a. Nursing Interventions Classification (NIC) b. Nursing Outcomes Classification (NOC) c. NANDA d. DSM-5

D

A patient would benefit from therapy in which peers as well as staff have a voice in determining patients' privileges and psychoeducational topics. Which approach would be best? a. Milieu therapy b. Cognitive therapy c. Short-term dynamic therapy d. Systematic desensitization

ANS: A Milieu therapy is based on the idea that all members of the environment contribute to the planning and functioning of the setting. The distracters are individual therapies that do not fit the description.

10 guiding principles of recovery

- *Self-directed* - *Individual- and person-centered* - *Empowering* - *Holistic* (whole life body/mind/spirit/community) - *Nonlinear* (continual growth, occ setbacks, learning from experience) - *Strengths-based* - *Peer-supported* - *Respect* - *Responsibility* - *Hope*

DSM-V & DSM IV-TR

- A WAY OF ORGANIZING INFORMATION: ---> to determine medical diagnosis ---> as multi axial system for diagnosis ---> contains axes I through V (1 - 5) ---> tool for data collection related to psych diagnosis CLASSIFY DISORDERS NOT PEOPLE: We do not think in terms of "the bipolar in 257-A", but instead "the patient with bipolar disorder in 257-A".

Psychiatric Nurse's Roles

- Counselor - Patient Advocate - Crisis Manager - Promoter of Self Care - Health Promoter & Maintainer - Health teacher-to patient and family - Assertiveness Trainer - Milieu Manager - Case Manager: --- Planner and evaluator of care --- Discharge planning

5 signs of mental health

- Happiness - Control over behavior - Accurate Appraisal of reality - Effectiveness in work - Healthy self-concept

DSM-V

- Now collapses Axis 1 - 3 into one axis or diagnostic statement. - The Environmental stressors are now addressed using specific codes. - One instrument for measuring those is the WHO Disability Assessment Schedule (WHODAS 2.0) (in the DSM folder-Canvas)

influences that impact individuals mental health

- available support system - spirituality - family influences - developmental events - personality traits and states - demographic/geographic location - negative influences -psychosocial stressors -poverty -impaired/inadequate parenting - cultural beliefs & values - health practice/belief - hormonal influences - biological influences - inherited factors - environmental experiences

A nurse uses Maslow's hierarchy of needs to plan care for a patient with mental illness. Which problem will receive priority? The patient: a. refuses to eat or bathe. b. reports feelings of alienation from family. c. is reluctant to participate in unit social activities. d. is unaware of medication action and side effects.

ANS: A The need for food and hygiene are physiological and therefore take priority over psychological or meta-needs in care planning.

A student nurse exhibits the following behaviors or actions while interactions with her patient. Which os these are appropriate as part of a therapeutic relationship? a) sitting attentively in selves with a withdrawn patient until the patient chooses to speak b) offering the patient advice on how he could cope more effectively with stress c) controlling the pace of the relationship by selecting topics for each interaction d) limiting the discussion of termination issues so as not to madden the patient unduly

A

Holistic

Recovery encompasses an individual's whole life, including mind, body, spirit, and community.

phenomena of concern

The central interests of a particular discipline. In nursing they are commonly considered to be person, health, environment, and nursing.

incidence

The number of new cases of a disease within a given period of time

epidemiology

is quantitative study of the distribution of mental disorders in human population.

DSM-5

manual to diagnose and classify mental disorders

Axis IV (DSM IV-TR)

*environmental stressors* - Illness in the family - financial problems - Living alone - Unemployment - Homelessness - Lack of transportation - Recent arrest

Axis III (DSM IV-TR)

*general medical conditions* - Arthritis - Head trauma, remote - Colitis - Hepatitis - Diabetes - Hypertension

DSM IV-TR

*include 5 axis* Axis I = psychiatric diagnosis (major clinical disorders) Axis II = personality disorders Axis III = general medical conditions Axis IV = environmental stressors Axis V = Scale 0-100: Global Assessment of Function

Axis II (DSM IV-TR)

*personality disorders* OR *mental retardation* - Borderline Personality Disorder - Antisocial Personality Disorder - Schizotypal Personality Disorder - Narcissistic Personality Disorder - Obsessive-Compulsive Personality Disorder - Must state "_______ Personality Disorder" *OR* - Mental Retardation

DSM-5 Organized Structor (young to old disorders)

1. Neurodevelopmental Disorders 2. Schizophrenia Spectrum Disorders 3. Bipolar and Related Disorders 4. Depressive Disorders 5. Anxiety Disorders 6. Obsessive-Compulsive Disorders 7. Trauma and Stressor-Related Disorders 8. Dissociative Disorders 9. Somatic Symptom Disorders 10. Feeding and Eating Disorders 11. Elimination Disorders 12. Sleep-Wake Disorders 13. Sexual Dysfunctions 14. Gender Dysphoria 15. Disruptive, Impulse Control, and Conduct Disorders 16. Substance Related and Addictive Disorders 17. Neurocognitive Disorders 18. Personality Disorders 19. Paraphiliac Disorders 20. Other Disorders

Clinical descriptions of mental and behavior disorders are divided into 11 disease classifications

1. Organic—including symptomatic—mental disorders 2 .Mental and behavioral disorders due to psychoactive substance use 3. Schizophrenia, schizotypal, and delusional disorders 4. Mood (affective) disorders 5. Neurotic, stress-related, and somatoform disorders 6. Behavioral syndromes associated with physiological disturbances and physical factors 7. Disorders of adult personality and behavior 8. Mental retardation 9. Disorders of psychological development 10. Behavioral and emotional disorders with onset usually occurring in childhood and adolescence 11. Unspecified mental disorder

Aa a nurse discharges a patient, the patient gives the nurse a card of appreciation made in an arts and crafts groups. What is the nurses best action a) recognize the effectiveness of the relationship and patients thoughtfulness. Accept the card b) inform the patient that accepting gifts violated policies of the facility. Decline the card c) acknowledge the patents transition thought ht extermination phase but decline the card d) accept the card and invite the patient to return to participate in other arts and craft groups

A

1. Which characteristic in an adolescent female is sometimes associated with the prodromal phase of schizophrenia? a. Always afraid another student will steal her belongings. b. An unusual interest in numbers and specific topics. c. Demonstrates no interest in athletics or organized sports. d. Appears more comfortable among males.

A

2. Which nursing intervention is particularly well chosen for addressing a population at high risk for developing schizophrenia? a. Screening a group of males between the ages of 15 and 25 for early symptoms. b. Forming a support group for females aged 25 to 35 who are diagnosed with substance use issues. c. Providing a group for patients between the ages of 45 and 55 with information on coping skills that have proven to be effective. d. Educating the parents of a group of developmentally delayed 5- to 6-year-olds on the importance of early intervention.

A

Which organ secrets hormones that are a normal component of the boys general response to stress ? select all that apply a) thyroid gland b) hypothalamus c) pituitary gland d) adrenal gland e) parathyroid glands

B, C, D

In which situations would a nurse have the duty to intervene and report ? select all that apply a) a peer has difficulty writing measurable outcomes b) a health care provider gives a telephone order for medication c) a peer tries to provide patient care in an alcohol impaired state d) a team member violates relationship boundaries with a patient e) a patient refuses medication prescribed by a licensed health care provider

C, D

cultural bound syndrome

Disorder only common to a society

milieu

Environment or surroundings

key areas of care promoted by QSEN

Patient-centered care Teamwork and collaboration Evidence-based practice Quality improvement Safety Informatics

Nonlinear

Recovery is based on continual growth, occasional setbacks, and learning from experience

electronic health care

The provision of health care through methods which are not face-to-face but rather through an electronic medium.

Culture

There is no standard measure for mental health, in part because it is culturally defined and is based on interpretations of effective functioning according to societal norms ex: Japanese may consider suicide to be an act of honor

clinical epidemiology

a broad field that examines health and illness at the population level. ex) study of diagnostic screening test observation/experimental studies of interventions used to treat pell with illness

dysthymia

a condition characterized by mild, but chronic, depressive symptoms that last for at least 2 years

comorbid condition

a condition that occurs along with another disorder Two disorders or conditions that occur or are diagnosed together. They co-occur.

mental illness

refers to all mental disorder with definable diagnoses.

mental health

state of well-being in which individuals can: - able to realize his or her own potential - cope w/ normal stresses of life - work productively - make contribution to community

psychiatric mental health nursing

this specialty area in nursing and core mental health profession - promotes mental health through the nursing process in the treatment of mental health problems and psychiatric disorders

Prevalence

total number of cases (new & existing)

A nurse administering a benzodiazepine should understand that the therapeutic effect of benzodiazepine results from potentiating the neurotransmitters a) GABA b) dopamine c) serotonin d) ACh e) a and c

A

Which theorist is associated with behavioral therapy a) Freud b) Skinner c) Sullivan d) Peplau

B

A patient says "I've done a lot of cheating and manipulation in my relationships". select a nonjudgemental response by the nurse a) how do you feel about that b) im glad that you realize this c) thats not a good way to behave d0 have your outgrown that type of behavior

A

A nurse introduces the matter of a contract during the first session with a new patient because contracts a) specify what the nurse will do for the patient b) sell out the participation and responsibilities of each party c) indicate the feeling tone established between the participants d) are binding and prevent either part from prematurely ending the relationship

B

Which imagine techniques can provide information about brain function a) computed tomography (CT) scan b) positron emission tomography (PET) scan c) Magnetic resonance imagine (MRI) scan d) skull radiograph

B

Which of the following activities would be considered nursing care and appropriate to be performed by a basic level nurse for a patient suffering from mental illness? a. teaching major depression b. teaching coping skills for a specific family dynamic c. conducting pyschotherapy d. prescribing antidepressant medication

B

Which student behavior is consistent with therapeutic communication a) offering your opinion when asked in tower to convey support b) summarizing the essence of the patients comments in your own words c) interrupting periods of silence before they become awkward for the patients d) telling the patient he did well when you approve his statements or actions

B

Which statement about mental illness is true? a. mental illness is a matter of individual nonconformity with sociotal terms b. mental illness is present when irrational and illogical behavior occurs c. mental illness changes with culture, time in history , political systems and the groups defining it d. mental illness is evaluated solely by considering individual control over behavior and appraisal of reality

C

A nurse finds a psychiatric advance directive in the medical record of a patient experiencing psychosis. the directive was executed during a period when the patient was stable and competent. the nurse should a) review the directive with the patient to ensure it is current b) ensure that the directive is respected in treatment planning c) consider the directive only if there is a cardiac or respiratory arrest d) encourage the patient to revise the directive in light of the current health problem

B

A nurse prepares to administer a scheduled injection of haloperidol decanoate (haldol depot) to an outpatient with schizophrenia. As the nurse swabs the six, the patient shouts "stop i don't want the medicine anymore. i hate the side effects" select the nursing best action a) assemble other staff for a show of force and proceed with the injection, using restraint if necessary b) stop the medication administration procedure and say to the patient "tell me more about the side effects you've been having" c) proceed with the injection but explain to the patient that there are medications that will help reduce the unpleasant side effects d) say to the patients "since I've already drawn the medication in the syringe I'm requiring to give it, but lets talk to the doctor about delaying next months dose"

B

A nurse wants to enhance growth of a patient by showing positive regard. The nurses action most likely to achieve this goal is a) making rounds daily b) staying with a tearful patient c) administering medication as prescribed d) examining personal feelings about a patients

B

A patient diagnosed with schizophrenia believes a local minister stirred evil spirit. the patient threatens to bomb a local church. the psychiatrist notifies the minister. select the answer with the correction rational. the psychiatrist a) released information without proper authorization b) demonstrated the duty to warn and protect c) violated the patients confidentiality d) avoided charges of malpractice

B

A patient diagnosed with schizophrenia tells the nurse"the CIA is monitoring us through the fluorescent lights in this room, Be careful w hat you say". Which response by the nurse if the most therapeutic a) lets talk about something other than the CIA b) it sounds like you're concerned about your privacy c) the CIA is prohibited form operating in health care facilities d) you have lost touch with reality, which is a symptom of your illness

B

An adolescent hospitalized ager a violent physical outburst tells the nurse "I'm going to kill my father, but you can't tell anyone". select the nurses best response a) you are right. federal law requires me to keep clinical information private b) i am obligated to share that information with the treatment team c) those kinds of thoughts will make your hospitalization longer d) to should share this thought with your psychiatrist

B

An advanced practice nurse observed a novice nurse expressing irritability regarding a patient with a long term history of alcoholism and suspects the new nurse is experiencing countertransference. Which comment by the new nurse confirms this suspicion a) the patient continues to deny problems resulting from drinking b) my parents were alcoholism and often neglected our family c) the patient cannot identify any goals for improvement d) the patient said i have many traits like her mother

B

An aide in a psychiatric hospital says to the nurse "we don't have time every day to help each patient complete a menu selection. lets tell dietary to prepare popular choices and send them to our unit" select the nurses best response a) thanks for the suggestions but that idea may not work because so many patients take MAOI (monoamine oxidase inhibitor) antidepressants b) thanks for the idea but its important to treat patients as individuals. giving choices is one was we can respect passions individuality c) thank you for the suggestion but the patients bill of rights requires us to allow patients to select their own diet d) thank you that is a veery good idea it will make meal preparation easier for the dietary department

B

Which function is classified as a circadian rhythm a) sex drive b) sleep cycle c) skeletal muscle contraction d) maintenance of a focused stream of consciousness

B

You have been working closely with a patient for the past month. Today he tells you he is looking forward to meeting with his new psychiatrist but frowns and avoids eye contact while reporting this to you. Which of the following repossess would most likely be therapeutic a) a new psychiatrist is s chance to start fresh. Im sure it will do well for you b) you say you look forward to the meeting, but you appear anxious or unhappy c) i notice that you frowned and avoided eye contact just no, don't you feel well d) i ge the impression you don't really want to see your psychiatrist. Can you tell me why

B

a patient experiencing psychosis asks a psychiatric technician. "whats the matter with me." the technician replies "nothing is wrong with you. you just need to use some self control." the nurse who overheard the exchange should take action based on a) the technicians unauthorized disclosure of confidential clinical information b) violation of the patients rights to be treated with dignity and respect c) the nurses obligation to report caregiver negligence d) the patients right to social interaction

B

select an example of a tort a) the plan of care for a patient is not completed within 24 hours of the patients admission b) a nurse fives a PRN dose of an antipsychotic drug to an agitated patient because the unit is short staffed c) an advanced practice nurse recommends hospitalization for a patient who is dangerous to self and others d) a patients admission status changed from involuntary to voluntary after the patents hallucinations subside

B

which action by a nurse constitutes a breach of a patients right to privacy a) documenting the patients daily behavior during hospitalization b) releasing information to the patients employer without consent c) discussing the patients history with other staff during care planning d) asking family to share information about a patients pre-hospitalization behavior

B

which using intervention demonstrated false imprisonment a) a confused and combative patient says "I'm getting out of here, and no one can stop me" the nurse restrains this patient without a health care providers order an then promptly obtains an order b) a patient has been irritating and attention seeking much of the day. a nurse escorts the patient down the hall saying "stay in your room, or you'll be put in seclusion" c) an involuntarily hospitalized patient with suicidal ideation runs out of the psychiatric unit. the nurse rushes after the patient and convinces the patient to return to the unit d) an involuntarily hospitalized patient with homicidal ideation attempts to leave the facility. a nurse call the security team and uses established protocols to prevent the patient from leaving

B

Select the example of a tort. A.The plan of care for a patient is not completed within 24 hours of the patient's admission. B.A nurse gives a PRN dose of an antipsychotic drug to an agitated patient because the unit is short-staffed. C.An advanced practice nurse recommends hospitalization for a patient who is dangerous to self and others. D.A patient's admission status changed from involuntary to voluntary after the patient's hallucinations subside.

B A tort is a civil wrong against a person that violates his or her rights. Giving unnecessary medication for the convenience of staff controls behavior in a manner similar to secluding a patient; thus, false imprisonment is a possible charge. The other options do not exemplify torts.

The WHO describes health as a " state of complete physical, mental, and social wellbeing and not merely the absence of disease or infirmity." Which statement is true in regards to overall health? (SATA) a. there is no relationship between physical and mental health b. poor physical health can lead to mental distress and disorders c. poor mental health does not lead to physical illness d. there is a strong relationship between physical health and mental health e. mental health needs to take precedence over physical health needs

B, D

consider bot Sullivan's term security operations and Freud's term defense mechanisms. Which statement suggest that they client's specialized treatment foal has been successfully met a) i really think i can succeed in school now b) I'm experiencing much less anxiety about school now c) going back to school is hard and ill need support d) i know that I'm not the only person who has a difficult time in school

B

Which descriptors exemplify consistency regarding nurse patient relationships? Select all that apply a) encouraging a patient to share initial impressions of staff b) having the same nurse care for a patient on a daily basis c) providing a schedule of daily activities to a patient d) setting a time for regular sessions with a patient e) offering solutions to a patients problems

B, C, D

Respect

Community, systems, and societal acceptance and appreciation of consumers—including protecting their rights and eliminating discrimination and stigma—are crucial in achieving recovery

A patient says "please pmt share this information about me with the other people". How should the nurse respond a) i will not share information with your family or friends without permission, but i share information about you with other staff b) a therapeutic relationship is just between the nurses and the patient. it is up to you to tell theories what you want them to know c) it depends on what you choose to tell me. i will be glad to disclose at the end of h each sessions what i will report to others d) i cannot tell anyone about you. it will be as though i am talking about my own problems, and we can help each other but keeping it between us

A

A psychiatric nurse best applies the ethical principle of autonomy by: A.exploring alternative solutions with a patient, who then makes a choice. B.suggesting that two patients who were fighting be restricted to the unit. C.intervening when a self-mutilating patient attempts to harm self. D.staying with a patient demonstrating a high level of anxiety.

A Autonomy is the right to self-determination, that is, to make one's own decisions. By exploring alternatives with the patient, the patient is better equipped to make an informed, autonomous decision. The distracters demonstrate beneficence, fidelity, and justice.

A Puerto Rican American patient uses dramatic body language when describing emotional discomfort. Which analysis most likely explains the patients behavior? The patient a) has a histrionic personality disorder b) believes dramatic body language is sexually appealing c) wishes to impress stagg wit the degree of emotional pain d) belongs to a culture in which dramatic body language is the norm

D

After leaving work, a nurse realizes documentation of administration of a PRN medication was omitted. This off duty nurse phones the nurse of duty and says "please document administration of the medication for me. my password is alpha1". the nurse receiving the call should a) fulfill the request promptly b) document the callers password c) refer the mater to the charge nurse to resolve d) report the request to the patents health care provider

C

After several therapeutic encounters with a patient who recently attempted suicide, which occurrence should cause the nurse to consider the possibility of countertransference a) the patients reactions toward the nurse seem realistic and appropriate b) the patients states "talking to you feels like talking to my parents" c) the nurse feels unusually happy when the patients mood begins to list d) the nurse develops a trusting relationship with the patient

C

13. A nurse consistently strives to demonstrate caring behaviors during interactions with patients. Which reaction by a patient indicates this nurse is effective? A patient reports feeling: a. distrustful of others. b. connected with others. c. uneasy about the future. d. discouraged with efforts to improve.

ANS: B A patient is likely to respond to caring with a sense of connectedness with others. The absence of caring can make patients feel distrustful, disconnected, uneasy, and discouraged.

A nurse is concerned that the agents policies are inadequate. Which understanding about the relationship between substandard institutional policies and individual nursing practice should guide nursing practice a) agency policies do not exempt an individual nurse of responsibility to practice according to professional standards of nursing care b) agency policies are the legal standard by which a professional nurse must act and therefore override other standards of care c) faced with substandard policies, a nurse has a responsibility to inform the supervisor and discontinue patient care immediately d) interpretation of policies by the judicial system is rendering on an individual basis and therefore cannot be predicted

A

cyclothymia

Milder form of bipolar disorder - mania bi-polar & - depression for 2 + yrs

A nurse saus "i am the only one who truly understands this patent. Other staff members are too critical". The nurses statement indicates a) boundary blurring b) sexual harassment c) positive regard d) advocacy

A

Which statement demonstrates the nurses understanding of the effect of environment factors on a patients mental health? a. " I'll need to assess how the patients family views mental health illness." b. " there is a history of depression in the patients extended family" c. " i'm not familiar with the patients Japanese cultural view on suicide" d. " the patients ability to pay for mental health services needs to be assessed."

C

Why is it important for a nurse to be aware of the multiple factors that can influence an individuals mental health a) rate of illness differ amount various groups b) the DSM cannot be used without information on multiple factors c) the nurse diagnoses and treats human responses which are influenced by many factors d) the nurse must contribute these data for epidemiological research

C

a new antidepressant is prescribed for an elderly patient with major depression, but the dose is more than usual geriatric dose. the nurse should a) consult a reliable drug reference b) teach the patients about possible side effects and adverse effects c) withhold the medication and confer wit the health care provider d) encourage the patient to increase oral fluids to reduce drug concentration

C

basic level registered nurse

Any nurse with basic training (diploma, associate degree, baccalaureate degree) in nursing.

diathesis-stress model

Asserts that most psychiatric disorders result from a combination of genetic vulnerability and negative environmental stressors

A nurse wants to demonstrate genuineness with a patient diagnosed with schizophrenia. The nurse should a) restate what the patient says b) use congruent communication strategies c) use self-revelation in patient interactions d) consistently interpret the patents behaviors

B

Freud believed that individuals cope with anxiety by implementing which mechanism a) the superego b) defense mechanism c) security operations d)cognitive distortions

B

The nurse caring for a client prescribed an antidepressant medication that produced anticholinergic side effects should assess for which possible side effects ? (select all that apply) a) memory dysfunction b) ejaculatory dysfunction c) blurred vision d) dry mouth e) constipation

C, D, E

Peer-supported

Consumers encourage and engage each other in recovery and provide a sense of belonging, supportive relationships, valued roles, and community.

In order to release information to another health care facility or third party regarding a patient diagnosed with a mental illness, the nurse must obtain a) signed consent by the patent for release of information stating specific information to be released b) verbal consent for information release from the patent and the patens guardian or next of kin c) permission from members of the health care team who participate in treatment planning d) approval from the attending psychiatrist to authorize the release of information

A

a cognitive therapist would help a client restructure the thought "i am stupid" to which statement a) what is did was stupid b) i am not as smart as others c) thing usually go wrong for me d) things like this should not happen to anymore

A

a patient in alcohol rehabilitation reveals to the nurse "i feel terrible guilt for sexually abusing my 2 year old before i was admitted". select the nurses most important action a) anonymously report the abuse by phone to the local child protection agency b) reply "I'm glad you feel comfortable talking to me about it" c) file a written report with the agency ethic committee d) respect nurse patient relationship confidentially

A

Individual- and person-centered

Recovery is based on unique strengths and resiliencies, as well as needs, preferences, experiences (including past trauma), and cultural backgrounds

A 38 year old patient diagnosed with major depression states, "my provider said something about the medicine I've been prescribed will affect my neurotransmitters. What exactly are neurotransmitters?" What is the nurses best response to the patients questions a) Neurotransmitters are chemical messengers in the brain that help regulate specific functions such as depression b) neurotransmitters are too complicated to explain easily. just know that the medication will help your mood and make you less depressed c) neurotransmitters are chemicals in that brain that are the reason you are depressed d) i will ask your provider to give you a more in depth explanation about why this medication will help your depression

A

A community mental health nurse has worked with a patient for 3 years but is moving out of the city and terminated the relationship. When a novice nurse beings work with this patient, what is the starting point for the relationship a) begin at the orientation phase b) resume the working relationship c) initially establish a social relationship d) return to the emotional catharsis phase

A

A nurse explanis to the family of a mentally ill patient how a nurse patient relationship differs from social relationships. Which is the best explanation a) the focus is on the patient. problems are discussed by th nurse and patient, but solutions are implemented by the patient b) the focus shifts form nurse tot patient as the relationship develops. advice is given by both and solutions are implemented c) the focus of the relationship is socialization. Mutual needs are met and feelings are shared openly d) the focus is creation of a partnership in which each member is concurred with growth an satisfaction of the other

A

A patient is having difficulty making a decision. The nurse has mixed feelings about whether to provide advice. Which principle usually applies? Giving advice: a) is rarely helpful b) fosters independence c) life the burden of personal decision making d) helps the patient develop feelings of personal adequacy

A

Amanda was raised by a rejecting and abusive father and had a difficult childhood. As an inpatient, she frequently comments on how hard her nurse, Jane, works and on how other staff members of not seem to care as much about their patients as Jane does. Jane finds herself agreeing with Amanda. Jane appreciated her insightfulness, and realized that the other staff members do not appreciate how hard she works and takes her for granted. Have enjoys the time she spends with Amanda and seeks out opportunities to interact with her. Which phenomenon is occurring here, and which response by Jane would most benefit her an the patient a) Amanda is experiencing transference. Have should help Amanda to understand that she's is empathizing in Jane those qualities that were missing in her father b) jane is idealizing Amanda, seeing her strengths and abilities that Amanda does not possess. Jane should temporarily distance herself somewhat from Amanda c) Amanda is over identifying with Jane, seeing similarities that do not in reality exist. Jane should label and explore this phenomenon in her interactions with Amanda d) jane is experiencing countertransference in response to Amanda's meeting Jane'es needs for greater appreciation. Jane should seek clinical supervision to explore these dynamic

A

At what point in the nurse patient relationship should a nurse place to first address termination a) during the orientation phase b) at the end of the working phase c) near the beginning of the termination phase d) when the patent initially brings up the topic

A

During a one-on-one interaction with the nurse, a patient frequently looks nervously at the door. Select the best comment by the nurse regarding the nonverbal communication a) i notice you keep looking toward the door b) this is our time together. No one is going to interrupt us c) it looks as if you are eager to end our discussion for today d) if you are uncomfortable in this room, we can move someplace else

A

Insurance will not pay for continued private hospitalization of a mentally ill patient. the family considers transferring the patient to a public hospital bu expresses concern that the patient will not get any treatment if transferred. select the nurses most helpful reply a) by law, treatment must be provided. hospitalization without treatment violates patents rights b) all patients in public hospitals have the right to choose both a primary therapist and a primary nurse c) you have a justifiable concern because the right to treatment extends only to provision of food, shelter and safety d) much will depend on other patents because the right to treatment for a psychotic patient takes precedence over the right to treatment of a patient who is stable

A

The incoherent thought and speech patterns of the client diagnosed with schizophrenia are related to the brains inability to perform which function a) regulate conscious mental activity b) retain and recall past experience c) regular social behavior d) maintain homeostasis

A

The medication prescribed for a client acts by blocking repute of both serotonin and norepinephrine. The nurse evaluated the treatment as successful when observing which client behavior a) laughing at a joke b) apologizing for being sarcastic c) writing down his telephone number d) doing to his room to 'calm down'

A

The nurse administered each of the following drugs to various patients. The patient who should be most carefully assessed for fluid and electrolyte imbalance is the one receiving a) lithium b) clozapine c) diazepam d) amiptriptyline

A

The nurse if working with a client experiencing depression stemming from low self esteem. The client is distrustful of unit staff and "just wants to go home". Initially what is the nurses priority a) making the client feel physically and emotionally safe b) teaching the client effecting coping skills c) identifying the clients positive traits d) focusing on preparing the client for speedy discharge

A

The nurse is responsible for the care of a client prescribed clonazepam should evaluate treatment as being successful when the client demonstrated which behavior a) less anxiety b) normal appetite c) improved physical balance d) reduced auditory hallucinations

A

What is the function of a neuron a) conduction of electrical impulses b) diffuses an impulse across a space c) provide a place at an axon terminal d) provides an attachment point of the cells surface

A

What is the premise underlying behavioral therapy a) behavior is learned and can be modified b) behavior is a product of unconscious drives c) motives must change before behaviors change d) behavior is determine by conditions, change in cognitions produces new behaviors

A

What ter is used to identify the structures that respond to stimuli, conduct electrical impulses an release neurotransmitters a) neurons b) synapses c) dendrites d) receptors

A

When a female Mexican American patient and a female nurse sit together, the patient often holds the nurses hand. The patient also links arms with the nurse when they walk. The nurse is uncomfortable with this behavior. Which analysis is most accurate a) the patient is accustomed to touch during conversation, as are members of many Hispanic subcultures b) the patient understands that touch makes the nurse uncomfortable and controls the relationship based on this factor c) the patient is afraid of being alone. When touching the nurse, the patient is reassured and confronted d) the patient is trying to manipulate the nurse using nonverbal techniques

A

When a tumor of the cerebellum is present, the nurse should expect that the client would initially demonstrate which sign/symptom a) impaired balance b) abnormal eye movement c) impaired social judgement d) blood pressure irregularities

A

When treating mental illnesses with psychotropic drugs, what is the focus of the treatment a) altering brain neurochemistry b) correcting brain anatomical defects c) regulating social behaviors d) activating the boys normal response to stress

A

Which nursing intervention demonstrates the theory behind operant conditioning a) rewarding the client with a token for avoiding an argument with another client b) showing the client how to be assertive without being aggressive c) demonstrating deep breathing techniques to a group of clients d) explaining to the client the consequences of not following unit rules

A

Which of the following actions best represents the basis or foundation of all other psychiatric nursing care a) the nurse assesses the patient at regular intervals b) the nurse administered psychotropic medications c) the nurse spends time sitting with a withdrawn patient d) the nurse participates in team meetings with other professions

A

Which statement shows nurse has empathy for a patient who made a suicide attempt a) you must have been very upset when you tried to hurt yourself b) it makes me sad to see you going through such a difficult experience c) if you tell me what is troubling you, i can help you solve your problems d) suicide is a drastic solution to a problem that may not be such a serious matter

A

Which technique will best communicate to a patient that the nurse is interested in listening a) restating a feeling or through the patient has expressed b) asking a direct question such as "did you feel angry" c) making a judgment about the patients problems d) saying "u understand what you're saying"

A

While talking wit ha patient diagnosed with major depression, a nurse noticed the patient is unable to maintain eye contact. The patients chin lowers to the chest, while the patient looks at the floor. Which aspect of communication has the nurse assessed? a) nonverbal communication b) a message filter c) a cultural barrier d) social skills

A

a person in the community asks "why aren't people with mental illness kept in state institutions anymore?" select the nurses best response a) less restrictive settings are available not to care for individuals with mental illness b) there are fewer persons with mental illness, so less hospital beds are needed c) most people with mental illness are still in psychiatric institutions d) psychiatric institutions violated patients rights

A

a psychiatric nurse best applies the ethical principle of autonomy by a) exploring alternative solutions with a patient, who then makes a choice b) suggesting that 2 patients who were fighting be restricted to the unit c) intervening when a self mutilating patient attempts to harm self d) staying with a patient demonstrating a high level of anxiety

A

A patient in alcohol rehabilitation reveals to the nurse, "I feel terrible guilt for sexually abusing my 6-year-old before I was admitted." Select the nurse's most important action. A. Anonymously report the abuse by phone to the local child protection agency B. Reply, "I'm glad you feel comfortable talking to me about it." C. File a written report with the agency's ethics committee. D. Respect nurse-patient relationship confidentiality.

A Laws regarding child abuse reporting discovered by a professional during the suspected abuser's alcohol or drug treatment differ by state. Federal law supersedes state law and prohibits disclosure without a court order except in instances in which the report can be made anonymously or without identifying the abuser as a patient in an alcohol or drug treatment facility.

A nurse is concerned that an agency's policies are inadequate. Which understanding about the relationship between substandard institutional policies and individual nursing practice should guide nursing practice? A. Agency policies do not exempt an individual nurse of responsibility to practice according to professional standards of nursing care. B. Agency policies are the legal standard by which a professional nurse must act and therefore override other standards of care. C. Faced with substandard policies, a nurse has a responsibility to inform the supervisor and discontinue patient care immediately. D. Interpretation of policies by the judicial system is rendered on an individual basis and therefore cannot be predicted.

A Nurses are professionally bound to uphold standards of practice regardless of lesser standards established by a health care agency or a state. Conversely, if the agency standards are higher than standards of practice, the agency standards must be upheld. The courts may seek to establish the standard of care through the use of expert witnesses when the issue is clouded.

In order to release information to another health care facility or third party regarding a patient diagnosed with a mental illness, the nurse must obtain: A. a signed consent by the patient for release of information stating specific information to be released. B. a verbal consent for information release from the patient and the patient's guardian or next of kin. C. permission from members of the health care team who participate in treatment planning. D. approval from the attending psychiatrist to authorize the release of information.

A Nurses have an obligation to protect patients' privacy and confidentiality. Clinical information should not be released without the patient's signed consent for the release.

A person in the community asks, "Why aren't people with mental illness kept in state institutions anymore?" Select the nurse's best response. A. "Less restrictive settings are available now to care for individuals with mental illness." B. "There are fewer persons with mental illness, so less hospital beds are needed." C. "Most people with mental illness are still in psychiatric institutions." D. "Psychiatric institutions violated patients' rights."

A The community is a less restrictive alternative than hospitals for treatment of persons with mental illness. The distracters are incorrect and part of the stigma of mental illness.

Insurance will not pay for continued private hospitalization of a mentally ill patient. The family considers transferring the patient to a public hospital but expresses concern that the patient will not get any treatment if transferred. Select the nurse's most helpful reply. A. "By law, treatment must be provided. Hospitalization without treatment violates patients' rights." B. "All patients in public hospitals have the right to choose both a primary therapist and a primary nurse." C. "You have a justifiable concern because the right to treatment extends only to provision of food, shelter, and safety." D. "Much will depend on other patients, because the right to treatment for a psychotic patient takes precedence over the right to treatment of a patient who is stable."

A The right to medical and psychiatric treatment was conferred on all patients hospitalized in public mental hospitals with the enactment of the federal Hospitalization of Mentally Ill Act in 1964.

mental health continuum

A conceptual line used to represent levels of mental health and mental illness that vary from person to person and vary for a particular person over time.

A nurse ends a relationship with a patient. Which actions by the nurse should be included in the termination phase? select all that apply a) focus dialogues wit the patent on problems that may occur in the future b) help the patient express feelings about the relationship wit the nurse c) help the patient prioritize and modify socially unacceptable behaviors d) reinforce expectations regarding the parameters of the relationship e) help the patient to identify strength,s limitations and problems

A, B

Which benefits are most associated with use of telehealth technologies? Select all that apply a) cost savings for patients b) maximize care management c) access to services for patients in rural areas d) prompt reimbursement by third party payers e) rapid development of trusting relationships with patients

A, B, C

Gine is admitted for treatment of depression with suicidal ideation triggered by marital discord. Her spouse visits one night and informs Gina that he has decided to file for divorce. The staff were aware of the visit and the husban'ds intentions regarding divorce but takes not further action, feeling that the queue 15 minutes suicide checks Gina is already on are sufficient. 30 minutes after the visit ends, staff make rounds and discover Gina has hanged herself in her bathroom, using hospital pajamas she tied together into a rope, which of the following statements best describes this situation? select all that apply a) the nurses have created liability for themselves and their employer by failing in their duty to protect Gina b) the nurse have breaches their duty to reassess Gina for increased suicide risk after her husbands visit c) given Gina's history, the nurses should have expected an increased risk of suicide ager her husbands announcement d) the nurses correctly reasoned that suicides cannot always be prevented and did their best to keep Gina safe through checks every 15 minutes e) the nurses are subject to a tart of professional negligence for failing to prevent the suicide by increasing the suicide precautions in response to Gina's increased risk f) had the nurses restricted Gina's movement or increased their checks on her, they would have been liable for false imprisonment and invasion of privacy, respectively

A, B, C, E

Factors that affect a persons mental health are (select all that apply) a) support systems b) development events c) socioeconomic status d) cultural beliefs

A, B, D

A nursing is interacting with patients in a psychiatric unit. Which statements reflect use of therapeutic communications? Select all that apply a) tell me more about the situation b) lets talk about something else c) i notice you are pacing a lot d) ill stay with you a while e) why did you do that

A, C, D

Based on the current understanding of brain physiology, which neurotransmitter would be the expected target of medication prescribed to manage depression (select all that apply) a) dopamine b) y-aminobutyric acid (GABA) c) serotonin d) norepinephrine

A, C, D

The nurse if caring for a client prescribed risperidone observes the client carefully for which possible side effects? select all that apply a) daytime sleepiness b) reports sof heartburn c) a rapid heartbeat d) sexual dysfunction e) weight gain

A, D, E

Which statement made by a patient demonstrates a healthy degree of resilience?(SATA) a. " i try to remember not to take other peoples bad moods personally." b. " i know that is I get really mad I'll end up being depressed." c. " i really feel that sometimes bad things are meant to happen" d. " I've learned to calm down before trying to defend my opinions" e. " i know that discussing issues with my boss would help me get my point across."

A, D, E

A novice psychiatric nurse has a parent with bipolar disorder. This nurse angrily recalls feelings of embarrassment about the patents behavior in the community .Select the best ways for the nurse to cope with these feelings. Select all that apply a) seek ways to use the understanding gained from childhood to help patient cope with their own illnesses b) recognize that these feelings are unhealthy. the nurse should try to suppress them when working with patents c) recognize the psychiatric nursing is more an appropriate career choice. explore other nursing specialities d) the nurse should begin new patient relationships by saying "my own patent had mental illness so i accept it without stigma" e) recognize that the feeling mat add sensitivity to the nurses practice, but supervision is important

A, E

9. Kyle, a patient with schizophrenia, began to take the first-generation antipsychotic haloperidol (Haldol) last week. One day you find him sitting very stiffly and not moving. He is diaphoretic, and when you ask if he is okay he seems unable to respond verbally. His vital signs are: BP 170/100, P 110, T 104.2°F. What is the priority nursing intervention? Select all that apply. a. Hold his medication and contact his prescriber. b. Wipe him with a washcloth wet with cold water or alcohol. c. Administer a medication such as benztropine IM to correct this dystonic reaction. d. Reassure him that although there is no treatment for his tardive dyskinesia, it will pass. e. Hold his medication for now and consult his prescriber when he comes to the unit later today.

A,B

3. To provide effective care for the patient diagnosed with schizophrenia, the nurse should frequently assess for which associated condition? Select all that apply. a. Alcohol use disorder b. Major depressive disorder c. Stomach cancer d. Polydipsia e. Metabolic syndrome

A,B,D,E

GENERALIZED ANXIETY DISORDER

A. Excessive anxiety and worry (apprehensive expectation), occurring more days than not for at least 6 months, about a number of events or activities (such as work or school performance). B. The individual finds it difficult to control the worry. C. The anxiety and worry are associated with three (or more) of the following six symptoms (with at least some symptoms having been present for more days than not for the past 6 months): (Note: Only one item is required in children.) 1.Restlessness or feeling keyed up or on edge 2.Being easily fatigued. 3.Difficulty concentrating or mind going blank. 4.Irritability. 5.Muscle tension 6.Sleep disturbance (difficulty falling or staying asleep, or restless, unsatisfying sleep). D. The anxiety, worry, or physical symptoms cause clinically significant distress or impairment in social, occupational, or other important areas of functioning.

11. A client is diagnosed with bipolar disorder: manic episode. Which nursing intervention would be implemented to achieve the outcome of "Client will gain 2 lb by the end of the week?" 1. Provide client with high-calorie finger foods throughout the day. 2. Accompany client to cafeteria to encourage adequate dietary consumption. 3. Initiate total parenteral nutrition to meet dietary needs. 4. Teach the importance of a varied diet to meet nutritional needs.

ANS: 1 Rationale: The nurse should provide the client with high-calorie finger foods throughout the day to help the client achieve the outcome of gaining 2 lb by the end of the week. Because of the hyperactive state, the client will have difficulty sitting still to consume large meals. Cognitive Level: Application Integrated Process: Implementation

4. A client diagnosed with bipolar disorder: depressive episode intentionally overdoses on sertraline (Zoloft). Family members report that the client has experienced anorexia, insomnia, and recent job loss. Which nursing diagnosis should a nurse prioritize? 1. Risk for suicide R/T hopelessness 2. Anxiety: severe R/T hyperactivity 3. Imbalanced nutrition: less than body requirements R/T refusal to eat 4. Dysfunctional grieving R/T loss of employment

ANS: 1 Rationale: The priority nursing diagnosis for this client should be risk for suicide R/T hopelessness. The nurse should always prioritize client safety. This client is at risk for suicide because of his or her recent suicide attempt. Cognitive Level: Analysis Integrated Process: Diagnosis

17. Which of the following rationales by a nurse explain to parents why is it difficult to diagnose a child or adolescent exhibiting symptoms of bipolar disorder? (Select all that apply.) 1. Bipolar symptoms mimic attention deficit-hyperactivity disorder symptoms. 2. Children are naturally active, energetic, and spontaneous. 3. Neurotransmitter levels vary considerably in accordance with age. 4. The diagnosis of bipolar disorder cannot be assigned prior to the age of 18. 5. Genetic predisposition is not a reliable diagnostic determinant.

ANS: 1, 2 Rationale: It is difficult to diagnose a child or adolescent with bipolar disorder, because bipolar symptoms mimic attention deficit-hyperactivity disorder symptoms and because children are naturally active, energetic, and spontaneous. Symptoms may also be comorbid with other childhood disorders, such as conduct disorder.

15. Which of the following instructions regarding lithium therapy should be included in a nurse's discharge teaching? (Select all that apply.) 1. Avoid excessive use of beverages containing caffeine. 2. Maintain a consistent sodium intake. 3. Consume at least 2,500 to 3,000 mL of fluid per day. 4. Restrict sodium content. 5. Restrict fluids to 1,500 mL per day.

ANS: 1, 2, 3 Rationale: The nurse should instruct the client taking lithium to avoid excessive use of caffeine, maintain a consistent sodium intake, and consume at least 2,500 to 3,000 mL of fluid per day. The risk of developing lithium toxicity is high because of the narrow margin between therapeutic doses and toxic levels. Fluid or sodium restriction can impact lithium levels. Cognitive Level: Application Integrated Process: Implementation

13. A client has been diagnosed with major depressive episode. After treatment with fluoxetine (Prozac), the client exhibits pressured speech and flight of ideas. Based on this symptom change, which physician action would the nurse anticipate? 1. Increase the dosage of fluoxetine. 2. Discontinue the fluoxetine and rethink the client's diagnosis. 3. Order benztropine (Cogentin) to address extrapyramidal symptoms. 4. Order olanzapine (Zyprexa) to address altered thoughts.

ANS: 2 Rationale: A full manic episode emerging during antidepressant treatment (medication, electroconvulsive therapy, etc.), but persisting beyond the physiological effect of that treatment is sufficient evidence for a manic episode and, therefore, a Bipolar I diagnosis.

14. Which is the basic premise of a recovery model used to treat clients diagnosed with bipolar disorder? 1. Medication adherence 2. Empowerment of the consumer 3. Total absence of symptoms 4. Improved psychosocial relationships

ANS: 2 Rationale: The basic premise of a recovery model is empowerment of the consumer. The recovery model is designed to allow consumers primary control over decisions about their own care and to enable a person with a mental health problem to live a meaningful life in a community of his or her choice while striving to achieve his or her full potential.

5. A client diagnosed with bipolar I disorder: manic episode refuses to take lithium carbonate (Lithobid) because of excessive weight gain. In order to increase adherence, which medication should a nurse anticipate that a physician may prescribe? 1. Sertraline (Zoloft) 2. Valproic acid (Depakote) 3. Trazodone (Desyrel) 4. Paroxetine (Paxil)

ANS: 2 Rationale: The nurse should anticipate that the physician may prescribe valproic acid in order to increase this client's medication adherence. Valproic acid is an anticonvulsant medication that can be used to treat bipolar disorder. One of the side effects of this medication is weight loss. Cognitive Level: Application Integrated Process: Planning

2. A client diagnosed with bipolar disorder is distraught over insomnia experienced over the last 3 nights and a 12-pound weight loss over the past 2 weeks. Which should be this client's priority nursing diagnosis? 1. Knowledge deficit R/T bipolar disorder AEB concern about symptoms 2. Altered nutrition: less than body requirements R/T hyperactivity AEB weight loss 3. Risk for suicide R/T powerlessness AEB insomnia and anorexia 4. Altered sleep patterns R/T mania AEB insomnia for the past 3 nights

ANS: 2 Rationale: The nurse should identify that the priority nursing diagnosis for this client is altered nutrition: less than body requirements R/T hyperactivity AEB weight loss. Because of the client's rapid weight loss, the nurse should prioritize interventions to ensure proper nutrition and physical health. Cognitive Level: Analysis Integrated Process: Diagnosis

9. What tool should a nurse use to differentiate occasional spontaneous behaviors of children from behaviors associated with bipolar disorder? 1. "Risky Activity" tool 2. "FIND" tool 3. "Consensus Committee" tool 4. "Monotherapy" tool

ANS: 2 Rationale: The nurse should use the "FIND" tool to differentiate occasional spontaneous behaviors of children from behaviors associated with bipolar disorder. FIND is an acronym that stands for frequency, intensity, number, and duration and is used to assess behaviors in children. Cognitive Level: Application Integrated Process: Assessment

7. A client began taking lithium carbonate (Lithobid) for the treatment of bipolar disorder approximately 1 month ago. The client asks if it is normal to have gained 12 pounds in this time frame. Which is the appropriate nursing response? 1. "That's strange. Weight loss is the typical pattern." 2. "What have you been eating? Weight gain is not usually associated with lithium." 3. "Weight gain is a common, but troubling, side effect." 4. "Weight gain only occurs during the first month of treatment with this drug."

ANS: 3 Rationale: The nurse should explain to the client that weight gain is a common side effect of lithium carbonate. The nurse should educate the client on the importance of medication adherence and discuss concerns with the prescribing physician if the client does not wish to continue taking the medication. Cognitive Level: Application Integrated Process: Implementation

3. A nurse is planning care for a client diagnosed with bipolar disorder: manic episode. In which order should the nurse prioritize the client outcomes in the exhibit? Client Outcomes: 1. Maintains nutritional status 2. Interacts appropriately with peers 3. Remains free from injury 4. Sleeps 6 to 8 hours a night 1. 2, 1, 3, 4 2. 4, 1, 2, 3 3. 3, 1, 4, 2 4. 1, 4, 2, 3

ANS: 3 Rationale: The nurse should order client outcomes based on priority in the following order: Remains free of injury, maintains nutritional status, sleeps 6 to 8 hours a night, and interacts appropriately with peers. The nurse should prioritize the client's safety and physical health as most important. Cognitive Level: Analysis Integrated Process: Planning

12. A nursing instructor is teaching about bipolar disorders. Which statement differentiates the DSM-5 diagnostic criteria of a manic episode from a hypomanic episode? 1. During a manic episode, clients may experience an inflated self-esteem or grandiosity, and these symptoms are absent in hypomania. 2. During a manic episode, clients may experience a decreased need for sleep, and this symptom is absent in hypomania. 3. During a manic episode, clients may experience psychosis, and this symptom is absent in hypomania. 4. During a manic episode, clients may experience flight of ideas and racing thoughts, and these symptoms are absent in hypomania.

ANS: 3 Rationale: Three or more of the following symptoms may be experienced in both hypomanic and manic episodes: Inflated self-esteem or grandiosity, decreased need for sleep (e.g., feels rested after only 3 hours of sleep), more talkative than usual or pressure to keep talking, flight of ideas and racing thoughts, distractibility, increase in goal-directed activity (either socially, at work or school, or sexually) or psychomotor agitation, excessive involvement in pleasurable activities that have a high potential for painful consequences (e.g., the person engages in unrestrained buying sprees, sexual indiscretions, or foolish business investments). If there are psychotic features, the episode is, by definition, manic. Cognitive Level: Analysis Integrated Process: Assessment

1. A highly agitated client paces the unit and states, "I could buy and sell this place." The client's mood fluctuates from fits of laughter to outbursts of anger. Which is the most accurate documentation of this client's behavior? 1. "Rates mood 8/10. Exhibiting looseness of association. Euphoric." 2. "Mood euthymic. Exhibiting magical thinking. Restless." 3. "Mood labile. Exhibiting delusions of reference. Hyperactive." 4. "Agitated and pacing. Exhibiting grandiosity. Mood labile."

ANS: 4 Rationale: The nurse should document that this client's behavior is "Agitated and pacing. Exhibiting grandiosity. Mood labile." The client is exhibiting mood swings from euphoria to irritability. Grandiosity refers to the attitude that one's abilities are better than everyone else's. Cognitive Level: Application Integrated Process: Evaluation

6. A client diagnosed with bipolar I disorder is exhibiting severe manic behaviors. A physician prescribes lithium carbonate (Eskalith) and olanzapine (Zyprexa). The client's spouse questions the Zyprexa order. Which is the appropriate nursing response? 1. "Zyprexa in combination with Eskalith cures manic symptoms." 2. "Zyprexa prevents extrapyramidal side effects." 3. "Zyprexa increases the effectiveness of the immune system." 4. "Zyprexa calms hyperactivity until the Eskalith takes effect."

ANS: 4 Rationale: The nurse should explain to the client's spouse that olanzapine can calm hyperactivity until the lithium carbonate takes effect. Lithium carbonate may take 1 to 3 weeks to begin to decrease hyperactivity. Monotherapy with the traditional mood stabilizers like lithium carbonate, or atypical antipsychotics like olanzapine, has been determined to be the first-line treatment for bipolar I disorder. Cognitive Level: Application Integrated Process: Implementation

8. A client diagnosed with bipolar disorder has been taking lithium carbonate (Lithobid) for one year. The client presents in an emergency department with a temperature of 101F (38C), severe diarrhea, blurred vision, and tinnitus. How should the nurse interpret these symptoms? 1. Symptoms indicate consumption of foods high in tyramine. 2. Symptoms indicate lithium carbonate discontinuation syndrome. 3. Symptoms indicate the development of lithium carbonate tolerance. 4. Symptoms indicate lithium carbonate toxicity.

ANS: 4 Rationale: The nurse should interpret that the client's symptoms indicate lithium carbonate toxicity. The initial signs of toxicity include ataxia, blurred vision, severe diarrhea, nausea and vomiting, and tinnitus. Lithium levels should be monitored monthly with maintenance therapy to ensure proper dosage. Cognitive Level: Application Integrated Process: Evaluation

10. A nursing instructor is discussing various challenges in the treatment of clients diagnosed with bipolar disorder. Which student statement demonstrates an understanding of the most critical challenge in the care of these clients? 1. "Treatment is compromised when clients can't sleep." 2. "Treatment is compromised when irritability interferes with social interactions." 3. "Treatment is compromised when clients have no insight into their problems." 4. "Treatment is compromised when clients choose not to take their medications."

ANS: 4 Rationale: The nursing student is accurate when stating that the most critical challenge in the care of clients diagnosed with bipolar disorder is that treatment is often compromised when clients choose not to take their medications. Clients diagnosed with bipolar disorder feel most productive and creative during manic episodes. This may lead to purposeful medication nonadherence. Symptoms of bipolar disorder will reemerge if medication is stopped. Cognitive Level: Application Integrated Process: Evaluation

16. A nurse is assessing an adolescent client diagnosed with cyclothymic disorder. Which of the following DSM-5 diagnostic criteria would the nurse expect this client to meet? (Select all that apply.) 1. Symptoms lasting for a minimum of two years 2. Numerous periods with manic symptoms 3. Possible comorbid diagnosis of a delusional disorder 4. Symptoms cause clinically significant impairment in important areas of functioning 5. Depressive symptoms that do not meet the criteria for major depressive episode

ANS: 4, 5 Rationale: The following are selected criteria for the diagnosis of cyclothymic disorder. For at least one year in children and adolescents there have been numerous periods with hypomanic, not manic symptoms that do not meet criteria for hypomanic episode and numerous periods with depressive symptoms that do not meet the criteria for a major depressive episode. The symptoms are not better accounted for by schizoaffective disorder and are not superimposed on schizophrenia, schizophreniform disorder, delusional disorder, or psychotic disorder not elsewhere classified. The symptoms cause clinically significant distress or impairment in social, occupational, or other important areas of functioning. Cognitive Level: Application Integrated Process: Implementation

A patient repeatedly stated, "I'm stupid." Which statement by that patient would show progress resulting from cognitive behavioral therapy? a. "Sometimes I do stupid things." b. "Things always go wrong for me." c. "I always fail when I try new things." d. "I'm disappointed in my lack of ability."

ANS: A "I'm stupid" is a cognitive distortion. A more rational thought is "Sometimes I do stupid things." The latter thinking promotes emotional self-control. The distracters reflect irrational or distorted thinking. This item relates to an audience response question.

Which technique is most applicable to aversion therapy? a. Punishment b. Desensitization c. Role modeling d. Positive reinforcement

ANS: A Aversion therapy is akin to punishment. Aversive techniques include pairing of a maladaptive behavior with a noxious stimulus, punishment, and avoidance training.

A nurse and patient discuss a problem the patient has kept secret for many years. Afterward the patient says, "I feel so relieved that I finally told somebody." Which term best describes the patient's feeling? a. Catharsis b. Superego c. Cognitive distortion d. Counter-transference

ANS: A Freud initially used talk therapy, known as the cathartic method. Today we refer to catharsis as "getting things off our chests." The superego represents the moral component of personality.

A patient is suspicious and frequently manipulates others. To which psychosexual stage do these traits relate? a. Oral b. Anal c. Phallic d. Genital

ANS: A The behaviors in the stem develop as the result of attitudes formed during the oral stage, when an infant first learns to relate to the environment. Anal-stage traits include stinginess, stubbornness, orderliness, or their opposites. Phallic-stage traits include flirtatiousness, pride, vanity, difficulty with authority figures, and difficulties with sexual identity. Genital-stage traits include the ability to form satisfying sexual and emotional relationships with members of the opposite sex, emancipation from parents, a strong sense of personal identity, or the opposites of these traits.

Which patient is the best candidate for brief psychodynamic therapy? a. An accountant with a loving family and successful career who was involved in a short extramarital affair b. An adult with a long history of major depression who was charged with driving under the influence (DUI) c. A woman with a history of borderline personality disorder who recently cut both wrists d. An adult male recently diagnosed with anorexia nervosa

ANS: A The best candidates for psychodynamic therapy are relatively healthy and well-functioning individuals, sometimes referred to as the "worried well," who have a clearly circumscribed area of difficulty and are intelligent, psychologically minded, and well-motivated for change. Patients with psychosis, severe depression, borderline personality disorders, and severe character disorders are not appropriate candidates for this type of treatment.

A 4-year-old grabs toys from siblings and says, "I want that now!" The siblings cry, and the child's parent becomes upset with the behavior. According to Freudian theory, this behavior is a product of impulses originating in which system of the personality? a. Id b. Ego c. Superego d. Preconscious

ANS: A The id operates on the pleasure principle, seeking immediate gratification of impulses. The ego acts as a mediator of behavior and weighs the consequences of the action, perhaps determining that taking the toy is not worth the mother's wrath. The superego would oppose the impulsive behavior as "not nice." The preconscious is a level of awareness. This item relates to an audience response question.

3. A patient has dementia. The health care provider wants to make a differential diagnosis between Alzheimer disease and multiple infarctions. Which diagnostic procedure should a nurse expect to prepare the patient for first? a. Computed tomography (CT) scan b. Positron emission tomography (PET) scan c. Functional magnetic resonance imaging (fMRI) d. Single-photon emission computed tomography (SPECT) scan

ANS: A A CT scan shows the presence or absence of structural changes, including cortical atrophy, ventricular enlargement, and areas of infarction—information that will be helpful to the health care provider. The other tests focus on brain activity and are more expensive; they may be ordered later.

Consider this comment from a therapist: "The patient is homosexual but has kept this preference secret. Severe anxiety and depression occur when the patient anticipates family reactions to this sexual orientation." Which perspective is evident in the speaker? a. Theory of interpersonal relationships b. Classical conditioning theory c. Psychosexual theory d. Behaviorism theory

ANS: A The theory of interpersonal relationships recognizes the anxiety and depression as resulting from unmet interpersonal security needs. Behaviorism and classical conditioning theories do not apply. A psychosexual formulation would focus on uncovering unconscious material that relates to the patient problem.

11. A patient who immigrated to the United States from Honduras was diagnosed with schizophrenia. The patient took an antipsychotic medication for 3 weeks but showed no improvement. Which resource should the treatment team consult for information on more effective medications for this patient? a. Clinical algorithm b. Clinical pathway c. Clinical practice guideline d. International Statistical Classification of Diseases and Related Health Problems (ICD)

ANS: A A clinical algorithm is a guideline that describes diagnostic and/or treatment approaches drawn from large databases of information. These guidelines help the treatment team make decisions cognizant of an individual patient's needs, such as ethnic origin, age, or gender. A clinical pathway is a map of interventions and treatments related to a specific disorder. Clinical practice guidelines summarize best practices about specific health problems. The ICD classifies diseases.

A 26-month-old displays negative behavior, refuses toilet training, and often says, "No!" Which stage of psychosexual development is evident? a. Oral b. Anal c. Phallic d. Genital

ANS: B The anal stage occurs from age 1 to 3 years and has as its focus toilet training and learning to delay immediate gratification. The oral stage occurs between birth and 1 year. The phallic stage occurs between 3 and 5 years, and the genital stage occurs between age 13 and 20 years.

29. A patient diagnosed with schizophrenia begins to talks about "macnabs" hiding in the warehouse at work. The term "macnabs" should be documented as: a. a neologism. c. thought insertion. b. concrete thinking. d. an idea of reference.

ANS: A A neologism is a newly coined word having special meaning to the patient. "Macnabs" is not a known common word. Concrete thinking refers to the inability to think abstractly. Thought insertion refers to thoughts of others are implanted in one's mind. Ideas of reference are a type of delusion in which trivial events are given personal significance. REF: Page 205-206 TOP: Nursing Process: Assessment

27. The family of a patient diagnosed with schizophrenia is unfamiliar with the illness and family's role in recovery. Which type of therapy should the nurse recommend? a. Psychoeducational c. Transactional b. Psychoanalytic d. Family

ANS: A A psychoeducational group explores the causes of schizophrenia, the role of medication, the importance of medication compliance, support for the ill member, and hints for living with a person with schizophrenia. Such a group can be of immeasurable practical assistance to the family. The other types of therapy do not focus on psychoeducation. REF: Page 214 (Box 12-5) | Page 221 TOP: Nursing Process: Implementation

A patient diagnosed with delirium is experiencing perceptual alterations. Which environmental adjustment should the nurse make for this patient? a. Provide a well-lit room without glare or shadows. Limit noise and stimulation. b. Maintain soft lighting day and night. Keep a radio on low volume continuously. c. Light the room brightly day and night. Awaken the patient hourly to assess mental status. d. Keep the patient by the nurse's desk while awake. Provide rest periods in a room with a television on.

ANS: A A quiet, shadow-free room offers an environment that produces the fewest sensory perceptual distortions for a patient with cognitive impairment associated with delirium. The other options have the potential to produce increased perceptual alterations.

2. Which action by a psychiatric nurse best supports a patient's right to be treated with dignity and respect? a. Consistently addressing a patient by title and surname. b. Strongly encouraging a patient to participate in the unit milieu. c. Discussing a patient's condition with another health care provider in the elevator. d. Informing a treatment team that a patient is too drowsy to participate in care planning.

ANS: A A simple way of showing respect is to address the patient by title and surname rather than assuming that the patient would wish to be called by his or her first name. Discussing a patient's condition with a health care provider in the elevator violates confidentiality. Informing a treatment team that the patient is too drowsy to participate in care planning violates patient autonomy. Encouraging a patient to participate in the unit milieu exemplifies beneficence and fidelity.

9. A nurse assesses that a patient is suspicious and frequently manipulates others. Using the Freudian theory, these traits are related to which psychosexual stage? a. Oral b. Anal c. Phallic d. Genital

ANS: A According to Freud, each of the behaviors mentioned develops as the result of attitudes formed during the oral stage, when an infant first learns to relate to the environment. Anal stage traits include stinginess, stubbornness, orderliness, or their opposites. Phallic stage traits include flirtatiousness, pride, vanity, difficulty with authority figures, and difficulties with sexual identity. Genital stage traits include the ability to form satisfying sexual and emotional relationships with members of the opposite sex, emancipation from parents, and a strong sense of personal identity.

9. A nurse should assess a patient taking a medication with anticholinergic properties for inhibited function of the: a. parasympathetic nervous system. b. sympathetic nervous system. c. reticular activating system. d. medulla oblongata.

ANS: A Acetylcholine is the neurotransmitter found in high concentration in the parasympathetic nervous system. When acetylcholine action is inhibited by anticholinergic drugs, parasympathetic symptoms such as blurred vision, dry mouth, constipation, and urinary retention appear. The functions of the sympathetic nervous system, the reticular activating system, and the medulla oblongata are not affected by anticholinergic medications.

Goals of care for an older adult patient diagnosed with delirium caused by fever and dehydration will focus on: a. returning to premorbid levels of function. b. identifying stressors negatively affecting self. c. demonstrating motor responses to noxious stimuli. d. exerting control over responses to perceptual distortions.

ANS: A The desired overall goal is that the delirious patient will return to the level of functioning held before the development of delirium. Demonstrating motor response to noxious stimuli is an indicator appropriate for a patient whose arousal is compromised. Identifying stressors that negatively affect the self is too nonspecific to be useful for a patient with delirium. Exerting control over responses to perceptual distortions is an unrealistic indicator for a patient with sensorium problems related to delirium.

17. A patient diagnosed with schizophrenia is very disturbed and violent. After several doses of haloperidol (Haldol), the patient is calm. Two hours later the nurse sees the patient's head rotated to one side in a stiff position, the lower jaw thrust forward, and drooling. Which problem is most likely? a. An acute dystonic reaction c. Waxy flexibility b. Tardive dyskinesia d. Akathisia

ANS: A Acute dystonic reactions involve painful contractions of the tongue, face, neck, and back. Opisthotonos and oculogyric crisis may be observed. Dystonic reactions are considered emergencies requiring immediate intervention. Tardive dyskinesia involves involuntary spasmodic muscular contractions that involve the tongue, fingers, toes, neck, trunk, or pelvis. It appears after prolonged treatment. Waxy flexibility is a symptom seen in catatonic schizophrenia. Internal and external restlessness, pacing, and fidgeting are characteristics of akathisia. REF: Page 215-216 (Table 12-4) TOP: Nursing Process: Assessment

9. A bill introduced in Congress would reduce funding for the care of people diagnosed with mental illnesses. A group of nurses write letters to their elected representatives in opposition to the legislation. Which role have the nurses fulfilled? a. Advocacy b. Attending c. Recovery d. Evidence-based practice

ANS: A An advocate defends or asserts another's cause, particularly when the other person lacks the ability to do that for himself or herself. Examples of individual advocacy include helping patients understand their rights or make decisions. On a community scale, advocacy includes political activity, public speaking, and publication in the interest of improving the individuals with mental illness; the letter-writing campaign advocates for that cause on behalf of patients who are unable to articulate their own needs.

19. Priority teaching for a patient taking clozapine (Clozaril) should include which instruction? a. Report sore throat and fever immediately. b. Avoid foods high in polyunsaturated fat. c. Use water-based lotions for rashes. d. Avoid unprotected sex.

ANS: A Clozapine therapy may produce agranulocytosis; therefore signs of infection should be immediately reported to the health care provider. In addition, the patient should have white blood cell levels measured weekly. The other options are not relevant to clozapine administration.

33. A nurse asks a patient diagnosed with schizophrenia, "What is meant by the old saying 'You can't judge a book by looking at the cover.'?" Which response by the patient indicates concrete thinking? a. "The table of contents tells what a book is about." b. "You can't judge a book by looking at the cover." c. "Things are not always as they first appear." d. "Why are you asking me about books?"

ANS: A Concrete thinking refers to an impaired ability to think abstractly. Concreteness is often assessed through the patient's interpretation of proverbs. Concreteness reduces one's ability to understand and address abstract concepts such as love or the passage of time. The incorrect options illustrate echolalia, an unrelated question, and abstract thinking. REF: Page 205-206 TOP: Nursing Process: Assessment

An older adult patient takes multiple medications daily. Over 2 days, the patient developed confusion, slurred speech, an unsteady gait, and fluctuating levels of orientation. These findings are most characteristic of: a. delirium. b. dementia. c. amnestic syndrome. d. Alzheimer's disease.

ANS: A Delirium is characterized by an abrupt onset of fluctuating levels of awareness, clouded consciousness, perceptual disturbances, and disturbed memory and orientation. The onset of dementia or Alzheimer's disease, a type of dementia, is more insidious. Amnestic syndrome involves memory impairment without other cognitive problems.

An elderly person presents with symptoms of delirium. The family reports, "Everything was fine until yesterday." What is the most important assessment information for the nurse to gather? a. A list of all medications the person currently takes b. Whether the person has experienced any recent losses c. Whether the person has ingested aged or fermented foods d. The person's recent personality characteristics and changes

ANS: A Delirium is often the result of medication interactions or toxicity. The distracters relate to MAOI therapy and depression.

6. A nurse says, "When I was in school, I learned to call upset patients by name to get their attention; however, I read a descriptive research study that says that this approach does not work. I plan to stop calling patients by name." Which statement is the best appraisal of this nurse's comment? a. One descriptive research study rarely provides enough evidence to change practice. b. Staff nurses apply new research findings only with the help from clinical nurse specialists. c. New research findings should be incorporated into clinical algorithms before using them in practice. d. The nurse misinterpreted the results of the study. Classic tenets of practice do not change.

ANS: A Descriptive research findings provide evidence for practice but must be viewed in relation to other studies before practice changes. One study is not enough. Descriptive studies are low on the hierarchy of evidence. Clinical algorithms use flow charts to manage problems and do not specify one response to a clinical problem. Classic tenets of practice should change as research findings provide evidence for change.

18. An acutely violent patient diagnosed with schizophrenia receives several doses of haloperidol (Haldol). Two hours later the nurse notices the patient's head rotated to one side in a stiffly fixed position, the lower jaw thrust forward, and drooling. Which intervention by the nurse is indicated? a. Administer diphenhydramine (Benadryl) 50 mg IM from the PRN medication administration record. b. Reassure the patient that the symptoms will subside. Practice relaxation exercises with the patient. c. Give trihexyphenidyl (Artane) 5 mg orally at the next regularly scheduled medication administration time. d. Administer atropine sulfate 2 mg subcut from the PRN medication administration record.

ANS: A Diphenhydramine, trihexyphenidyl, benztropine, and other anticholinergic medications may be used to treat dystonias. Swallowing will be difficult or impossible therefore, oral medication is not an option. Medication should be administered immediately, so the intramuscular route is best. In this case, the best option given is diphenhydramine. REF: Page 215-216 (Table 12-4) TOP: Nursing Process: Implementatio

An older adult is prescribed digoxin (Lanoxin) and hydrochlorothiazide daily as well as lorazepam (Ativan) as needed for anxiety. Over 2 days, the patient developed confusion, slurred speech, an unsteady gait, and fluctuating levels of orientation. What is the most likely reason for the patient's change in mental status? a. Drug actions and interactions b. Benzodiazepine withdrawal c. Hypotensive episodes d. Renal failure

ANS: A Drug actions and interactions are common among elderly persons and predispose this population to delirium. Delirium is characterized by an abrupt onset of fluctuating levels of awareness, clouded consciousness, perceptual disturbances, and disturbed memory and orientation. The patient takes lorazepam on a PRN basis, so withdrawal is unlikely. Hypotensive episodes or problems with renal function may occur associated with the patient's drug regime, but interactions are more likely the problem

1. Planning for patients with mental illness is facilitated by understanding that inpatient hospitalization is generally reserved for patients who: a. present a clear danger to self or others. b. are noncompliant with medications at home. c. have no support systems in the community. d. develop new symptoms during the course of an illness.

ANS: A Hospitalization is justified when the patient is a danger to self or others, has dangerously decompensated, or needs intensive medical treatment. The incorrect options do not necessarily describe patients who require inpatient treatment.

10. The therapeutic action of monoamine oxidase inhibitors (MAOIs) blocks neurotransmitter reuptake, causing: a. increased concentration of neurotransmitters in the synaptic gap. b. decreased concentration of neurotransmitters in serum. c. destruction of receptor sites. d. limbic system stimulation.

ANS: A If the reuptake of a substance is inhibited, then it accumulates in the synaptic gap and its concentration increases, permitting the ease of the transmission of impulses across the synaptic gap. Normal transmission of impulses across synaptic gaps is consistent with a normal rather than a depressed mood. The other options are not associated with blocking neurotransmitter reuptake.

An older adult patient in the intensive care unit has visual and auditory illusions. Which intervention will be most helpful? a. Using the patient's glasses and hearing aids b. Placing personally meaningful objects in view c. Placing large clocks and calendars on the wall d. Assuring that the room is brightly lit but very quiet at all times

ANS: A Illusions are sensory misperceptions. Glasses and hearing aids help clarify sensory perceptions. Without glasses, clocks, calendars, and personal objects are meaningless. Round-the-clock lighting promotes sensory overload and sensory perceptual alterations.

6. A nurse administers a medication that potentiates the action of gamma-aminobutyric acid (GABA). Which finding would be expected? a. Reduced anxiety b. Improved memory c. More organized thinking d. Fewer sensory perceptual alterations

ANS: A Increased levels of GABA reduce anxiety; thus any potentiation of GABA action should result in anxiety reduction. Memory enhancement is associated with acetylcholine and substance P. Thought disorganization is associated with dopamine. GABA is not associated with sensory perceptual alterations.

16. A patient tells the nurse at the clinic, "I haven't been taking my antidepressant medication as directed. I leave out the midday dose. I have lunch with friends and don't want them to ask me about the pills." Select the nurse's most appropriate intervention. a. Investigate the possibility of once-daily dosing of the antidepressant. b. Suggest to the patient to take the medication when no one is watching. c. Explain how taking each dose of medication on time relates to health maintenance. d. Add the following nursing diagnosis to the plan of care: Ineffective therapeutic regimen management, related to lack of knowledge.

ANS: A Investigating the possibility of once-daily dosing of the antidepressant has the highest potential for helping the patient achieve compliance. Many antidepressants can be administered by once-daily dosing, a plan that increases compliance. Explaining how taking each dose of medication on time relates to health maintenance is reasonable but would not achieve the goal; it does not address the issue of stigma. The self-conscious patient would not be comfortable doing this. A better nursing diagnosis would be related to social stigma. The question asks for an intervention, not analysis.

17. A patient being treated in an alcohol rehabilitation unit reveals to the nurse, "I feel terrible guilt for sexually abusing my 6-year-old child before I was admitted." Based on state and federal law, the best action for the nurse to take is to: a. anonymously report the abuse by telephone to the local child abuse hotline. b. reply, "I'm glad you feel comfortable talking to me about it." c. respect the nurse-patient relationship of confidentiality. d. file a written report on the agency letterhead.

ANS: A Laws regarding reporting child abuse discovered by a professional during a suspected abuser's alcohol or drug treatment differ by state. Federal law supersedes state law and prohibits disclosure without a court order except in instances in which the report can be made anonymously or without identifying the abuser as a patient in an alcohol or drug treatment facility. Anonymously reporting the abuse by telephone to the local child abuse hotline meets federal criteria. Respecting nurse-patient confidentiality and replying "I'm glad you feel comfortable talking to me about it" do not accomplish reporting. Filing a written report on agency letterhead violates federal law.

5. A nurse assesses an inpatient psychiatric unit, noting that exits are free from obstruction, no one is smoking, the janitor's closet is locked, and all sharp objects are being used under staff supervision. These observations relate to: a. management of milieu safety. b. coordinating care of patients. c. management of the interpersonal climate. d. use of therapeutic intervention strategies.

ANS: A Members of the nursing staff are responsible for all aspects of milieu management. The observations mentioned in this question directly relate to the safety of the unit. The other options, although part of the nurse's concerns, are unrelated to the observations cited.

20. An advanced practice nurse determines a group of patients would benefit from therapy in which peers and interdisciplinary staff all have a voice in determining the level of the patients' privileges. The nurse would arrange for: a. milieu therapy b. cognitive therapy c. short-term dynamic therapy d. systematic desensitization

ANS: A Milieu therapy is based on the idea that all members of the environment contribute to the planning and functioning of the setting. The other therapies are all individual therapies that do not fit the description.

16. A drug causes muscarinic-receptor blockade. A nurse will assess the patient for: a. dry mouth. b. gynecomastia. c. pseudoparkinsonism. d. orthostatic hypotension.

ANS: A Muscarinic-receptor blockade includes atropine-like side effects such as dry mouth, blurred vision, and constipation. Gynecomastia is associated with decreased prolactin levels. Movement defects are associated with dopamine blockade. Orthostatic hypotension is associated with alpha1-receptor antagonism.

10. Several nurses are concerned that agency policies related to restraint and seclusion are inadequate. Which statement about the relationship of substandard institutional policies and individual nursing practice should guide nursing practice? a. The policies do not absolve an individual nurse of the responsibility to practice according to the professional standards of nursing care. b. Agency policies are the legal standard by which a professional nurse must act and therefore override other standards of care. c. In an institution with substandard policies, the nurse has a responsibility to inform the supervisor and leave the premises. d. Interpretation of policies by the judicial system is rendered on an individual basis and therefore cannot be predicted.

ANS: A Nurses are professionally bound to uphold the American Nurses Association (ANA) standards of practice, regardless of lesser standards established by a health care agency or state. Conversely, if the agency standards are higher than the ANA standards of practice, the agency standards must be upheld. The courts may seek to establish the standard of care through the use of expert witnesses when the issue is clouded.

22. A patient diagnosed with mental illness asks a psychiatric technician, "What's the matter with me?" The technician replies, "Your wing nuts need tightening." The nurse who overheard the exchange should take action based on: a. violation of the patient's right to be treated with dignity and respect. b. the nurse's obligation to report caregiver negligence. c. preventing defamation of the patient's character. d. supervisory liability.

ANS: A Patients have the right to be treated with dignity and respect. Patients should never be made the butt of jokes about their illness. Patient emotional abuse has been demonstrated, not negligence. The technician's response was not clearly defamation. Patient abuse, not supervisory liability, is the issue.

A patient has progressive memory deficits associated with dementia. Which nursing intervention would best help the individual function in the environment? a. Assist the patient to perform simple tasks by giving step-by-step directions. b. Reduce frustration by performing activities of daily living for the patient. c. Stimulate intellectual function by discussing new topics with the patient. d. Read one story from the newspaper to the patient every day.

ANS: A Patients with cognitive impairment should perform all tasks of which they are capable. When simple directions are given in a systematic fashion, the patient is better able to process information and perform simple tasks. Stimulating intellectual functioning by discussing new topics is likely to prove frustrating for the patient. Patients with cognitive deficits may enjoy the attention of someone reading to them, but this activity does not promote their function in the environment.

9. A nurse at the mental health center prepares to administer a scheduled injection of haloperidol decanoate (Haldol depot) to a patient with schizophrenia. As the nurse swabs the site, the patient shouts, "Stop! I don't want to take that medicine anymore. I hate the side effects." Select the nurse's best initial action. a. Stop the medication administration procedure and say to the patient, "Tell me more about the side effects you've been having." b. Say to the patient, "Since I've already drawn the medication in the syringe, I'm required to give it, but let's talk to the doctor about skipping next month's dose." c. Proceed with the injection but explain to the patient that other medications are available that may help reduce the unpleasant side effects. d. Notify other staff members to report to the room for a show of force and proceed with the injection, using restraint if necessary.

ANS: A Patients with mental illness retain their civil rights unless clear, cogent, and convincing evidence of dangerousness exists. The patient in this situation presents no evidence of being dangerous. The nurse, an as advocate and educator, should seek more information about the patient's decision and should not force the medication.

A hospitalized patient diagnosed with delirium misinterprets reality, while a patient diagnosed with dementia wanders about the home. Which outcome is the priority in both scenarios? The patients will: a. remain safe in the environment. b. participate actively in self-care. c. communicate verbally. d. acknowledge reality.

ANS: A Risk for injury is the nurse's priority concern. Safety maintenance is the desired outcome. The other outcomes are lower priorities and may not be realistic.

21. A nurse caring for a patient taking a serotonin reuptake inhibitor (SSRI) will develop outcome criteria related to: a. mood improvement. b. logical thought processes. c. reduced levels of motor activity. d. decreased extrapyramidal symptoms.

ANS: A SSRIs affect mood, relieving depression in many patients. SSRIs do not act to reduce thought disorders. SSRIs reduce depression but have little effect on motor hyperactivity. SSRIs do not produce extrapyramidal symptoms.

13. A community mental health nurse wants to establish a relationship with a very withdrawn patient diagnosed with schizophrenia. The patient lives at home with a supportive family. Select the nurse's best plan. a. Visit daily for 4 days, then every other day for 1 week stay with patient for 20 minutes, accept silence state when the nurse will return. b. Arrange to spend 1 hour each day with the patient focus on asking questions about what the patient is thinking or experiencing avoid silences. c. Visit twice daily sit beside the patient with a hand on the patient's arm leave if the patient does not respond within 10 minutes. d. Visit every other day remind the patient of the nurse's identity encourage the patient to talk while the nurse works on reports.

ANS: A Severe constraints on the community mental health nurse's time will probably not allow more time than what is mentioned in the correct option yet, important principles can be used. A severely withdrawn patient should be met "at the patient's own level," with silence accepted. Short periods of contact are helpful to minimize both the patient's and the nurse's anxiety. Predictability in returning as stated will help build trust. An hour may be too long to sustain a home visit with a withdrawn patient, especially if the nurse persists in leveling a barrage of questions at the patient. Twice-daily visits are probably not possible, and leaving after 10 minutes would be premature. Touch may be threatening. Working on reports suggests the nurse is not interested in the patient. REF: Page 210-211 | Page 215 (Box 12-6) | Page 222 (Case study and Nursing Care Plan 12-1)

13. A student nurse tells the instructor, "I don't need to interact with my patients. I learn what I need to know by observation." The instructor can best interpret the nursing implications of Sullivan's theory to the student by responding: a. "Nurses cannot be isolated. We must interact to provide patients with opportunities to practice interpersonal skills." b. "Observing patient interactions can help you formulate priority nursing diagnoses and appropriate interventions." c. "I wonder how accurate your assessment of the patient's needs can be if you do not interact with the patient." d. "Noting patient behavioral changes is important because these signify changes in personality."

ANS: A Sullivan believed that the nurse's role includes educating patients and assisting them in developing effective interpersonal relationships. Mutuality, respect for the patient, unconditional acceptance, and empathy are cornerstones of Sullivan's theory. The nurse who does not interact with the patient cannot demonstrate these cornerstones. Observations provide only objective data. Priority nursing diagnoses usually cannot be accurately established without subjective data from the patient. The third response pertains to Maslow's theory. The fourth response pertains to behavioral theory.

4. A 4-year-old child grabs toys from siblings, saying, "I want that toy now!" The siblings cry, and the child's parent becomes upset with the behavior. Using the Freudian theory, a nurse can interpret the child's behavior as a product of impulses originating in the: a. id. b. ego. c. superego. d. preconscious.

ANS: A The id operates on the pleasure principle, seeking immediate gratification of impulses. The ego acts as a mediator of behavior and weighs the consequences of the action, perhaps determining that taking the toy is not worth the parent's wrath. The superego would oppose the impulsive behavior as "not nice." The preconscious is a level of awareness.

15. A nurse uses Maslow's hierarchy of needs to plan care for a psychotic patient. Which problem will receive priority? The patient: a. refuses to eat or bathe. b. reports feelings of alienation from family. c. is reluctant to participate in unit social activities. d. needs to be taught about medication action and side effects.

ANS: A The need for food and hygiene is physiological and therefore takes priority over psychological or meta-needs in care planning.

2. A patient is hospitalized for depression and suicidal ideation after their spouse asks for a divorce. Select the nurse's most caring comment. a. "Let's discuss some means of coping other than suicide when you have these feelings." b. "I understand why you're so depressed. When I got divorced, I was devastated too." c. "You should forget about your marriage and move on with your life." d. "How did you get so depressed that hospitalization was necessary?"

ANS: A The nurse's communication should evidence caring and a commitment to work with the patient. This commitment lets the patient know the nurse will help. Probing and advice are not helpful or therapeutic interventions.

7. A 40-year-old adult living with parents states, "I'm happy but I don't socialize much. My work is routine. When new things come up, my boss explains them a few times to make sure I understand. At home, my parents make decisions for me, and I go along with them." A nurse should identify interventions to improve this patient's: a. self-concept. b. overall happiness. c. appraisal of reality. d. control over behavior.

ANS: A The patient feels the need for multiple explanations of new tasks at work and, despite being 40 years of age, allows both parents to make all decisions. These behaviors indicate a poorly developed self-concept. Although the patient reports being happy, the subsequent comments refute that self-appraisal. The patient's comments do not indicate that he/she is out of touch with reality. The patient's needs are broader than control over own behavior.

8. A nurse surveys the medical records for violations of patients' rights. Which finding signals a violation? a. No treatment plan is present in record. b. Patient belongings are searched at admission. c. Physical restraint is used to prevent harm to self. d. Patient is placed on one-to-one continuous observation.

ANS: A The patient has the right to have a treatment plan. Inspecting a patient's belongings is a safety measure. Patients have the right to a safe environment, including the right to be protected against impulses to harm self that occur as a result of a mental disorder.

12. A patient is experiencing delusions of persecution about being poisoned. The patient has refused all hospital meals for 3 days. Which intervention is most likely to be acceptable to the patient? a. Allowing the patient supervised access to food vending machines b. Allowing the patient to phone a local restaurant to deliver meals c. Offering to taste each portion on the tray for the patient d. Providing tube feedings or total parenteral nutrition

ANS: A The patient who is delusional about food being poisoned is likely to believe restaurant food might still be poisoned and to say that the staff member tasting the food has taken an antidote to the poison before tasting. Attempts to tube feed or give nutrition intravenously are seen as aggressive and usually promote violence. Patients perceive foods in sealed containers, packages, or natural shells as being safer. REF: Page 205-206 (Table 12-1) TOP: Nursing Process: Planning

An older adult with moderately severe dementia forgets where the bathroom is and has episodes of incontinence. Which intervention should the nurse suggest to the patient's family? a. Label the bathroom door. b. Take the older adult to the bathroom hourly. c. Place the older adult in disposable adult briefs. d. Limit the intake of oral fluids to 1000 ml per day.

ANS: A The patient with moderately severe dementia has memory loss that begins to interfere with activities. This patient may be able to use environmental cues such as labels on doors to compensate for memory loss. Regular toileting may be helpful, but a 2-hour schedule is often more reasonable. Placing the patient in disposable briefs is more appropriate at a later stage. Severely limiting oral fluid intake would predispose the patient to a urinary tract infection.

What is the priority nursing diagnosis for a patient with fluctuating levels of consciousness, disturbed orientation, and visual and tactile hallucinations? a. Risk for injury related to altered cerebral function, fluctuating levels of consciousness, disturbed orientation, and misperception of the environment b. Bathing/hygiene self-care deficit related to cerebral dysfunction, as evidenced by confusion and inability to perform personal hygiene tasks c. Disturbed thought processes related to medication intoxication, as evidenced by confusion, disorientation, and hallucinations d. Fear related to sensory perceptual alterations as evidenced by visual and tactile hallucinations

ANS: A The physical safety of the patient is of highest priority among the diagnoses given. Many opportunities for injury exist when a patient misperceives the environment as distorted, threatening, or harmful or when the patient exercises poor judgment or when the patient's sensorium is clouded. The other diagnoses may be concerns, but are lower priorities.

15. The family of a patient whose insurance will not pay for continuing hospitalization considers transferring the patient to a public psychiatric hospital. The family expresses concern that the patient will "never get any treatment." Which reply by the nurse would be most helpful? a. "Under the law, treatment must be provided. Hospitalization without treatment violates patients' rights." b. "That's a justifiable concern because the right to treatment extends only to the provision of food, shelter, and safety." c. "Much will depend on other patients, because the right to treatment for a psychotic patient takes precedence over the right to treatment of a patient who is stable." d. "All patients in public hospitals have the right to choose both a primary therapist and a primary nurse."

ANS: A The right to medical and psychiatric treatment was conferred on all patients hospitalized in public mental hospitals with the enactment of the federal Hospitalization of Mentally Ill Act in 1964. Stating that the concern is justifiable supports the family's erroneous belief. The provisions mentioned in the third and fourth options are not part of this or any other statute governing psychiatric care.

9. Which principle takes priority for the psychiatric inpatient staff when addressing behavioral crises? a. Resolve behavioral crises using the least restrictive intervention possible. b. Rights of the majority of patients supersede the rights of individual patients. c. Swift intervention is justified to maintain the integrity of the therapeutic milieu. d. Allow patients opportunities to regain control without intervention if the safety of other patients is not compromised.

ANS: A The rule of using the least restrictive treatment or intervention possible to achieve the desired outcome is the patient's legal right. Planned interventions are nearly always preferable. Intervention may be necessary when the patient threatens harm to self.

14. A psychiatric technician says, "Little of what takes place on the behavioral health unit seems to be theory based." A nurse educates the technician by identifying which common use of Sullivan's theory? a. Structure of the therapeutic milieu of most behavioral health units b. Frequent use of restraint and seclusion for behavior modification c. Assessment tools based on age-appropriate versus arrested behaviors d. Use of the nursing process to determine the best sequence for nursing actions

ANS: A The structure of the therapeutic environment has, as its foci, an accepting atmosphere and provision of opportunities for practicing interpersonal skills. Both constructs are directly attributable to Sullivan's theory of interpersonal relationships. Sullivan's interpersonal theory did not specifically consider the use of restraint or seclusion. Assessment based on the developmental level is associated with Erikson's theories. The nursing process applies concepts from multiple theories.

24. The laboratory report for a patient taking clozapine (Clozaril) shows a white blood cell count of 3000 mm3 and a granulocyte count of 1500 mm3. The nurse should: a. report the laboratory results to the health care provider. b. give the next dose as prescribed. c. administer aspirin and force fluids. d. repeat the laboratory tests.

ANS: A These laboratory values indicate the possibility of agranulocytosis, a serious side effect of clozapine therapy. These results must be immediately reported to the health care provider. The drug should be withheld because the health care provider will discontinue it. The health care provider may repeat the laboratory test, but, in the meantime, the drug should be withheld. Giving aspirin and forcing fluids are measures that are less important than stopping the administration of the drug.

1. A 26-month-old child displays negative behaviors. The parent says, "My child refuses toilet training and shouts, 'No!' when given direction. What do you think is wrong?" Select the nurse's best reply. a. "This is normal for your child's age. The child is striving for independence." b. "The child needs firmer control. Punish the child for disobedience and say, 'No.'" c. "There may be developmental problems. Most children are toilet trained by age 2 years." d. "Some undesirable attitudes are developing. A child psychologist can help you develop a remedial plan."

ANS: A These negative behaviors are typical of a child around the age of 2 years whose developmental task is to develop autonomy. The incorrect options indicate the child's behavior is abnormal.

3. A multidisciplinary health care team meets 12 hours after an adolescent is hospitalized after a suicide attempt. Members of the team report their assessments. What outcome can be expected from this meeting? a. A treatment plan will be formulated. b. The health care provider will order neuroimaging studies. c. The team will request a court-appointed advocate for the patient. d. Assessment of the patient's need for placement outside the home will be undertaken.

ANS: A Treatment plans are formulated early in the course of treatment to streamline the treatment process and reduce costs. It is too early to determine the need for alternative post-discharge living arrangements. Neuroimaging is not indicated for this scenario.

4. When a patient diagnosed with schizophrenia was discharged 6 months ago, haloperidol (Haldol) was prescribed. The patient now says, "I stopped taking those pills. They made me feel like a robot." What are common side effects the nurse should validate with the patient? a. Sedation and muscle stiffness b. Sweating, nausea, and diarrhea c. Mild fever, sore throat, and skin rash d. Headache, watery eyes, and runny nose

ANS: A Typical antipsychotic drugs often produce sedation and extrapyramidal side effects such as stiffness and gait disturbance, effects the patient might describe as making him or her feel like a "robot." The side effects mentioned in the other options are usually not associated with typical antipsychotic therapy or would not have the effect described by the patient. REF: Page 215-216 (Table 12-4) TOP: Nursing Process: Assessment

20. Which employer's health plan is required to include parity provisions related to mental illnesses? a. Employer with more than 50 employees b. Cancer thrift shop staffed by volunteers c. Daycare center that employs 7 teachers d. Church that employs 15 people

ANS: A Under federal parity laws, companies with more than 50 employees may not limit annual or lifetime mental health benefits unless they also limit benefits for physical illnesses.

22. What assessment findings mark the prodromal stage of schizophrenia? a. Withdrawal, misinterpreting, poor concentration, and preoccupation with religion b. Auditory hallucinations, ideas of reference, thought insertion, and broadcasting c. Stereotyped behavior, echopraxia, echolalia, and waxy flexibility d. Loose associations, concrete thinking, and echolalia neologisms

ANS: A Withdrawal, misinterpreting, poor concentration, and preoccupation with religion are prodromal symptoms, the symptoms that are present before the development of florid symptoms. The incorrect options each list the positive symptoms of schizophrenia that might be apparent during the acute stage of the illness. REF: Page 201-202 | Page 204-205 TOP: Nursing Process: Assessment

38. A newly hospitalized patient experiencing psychosis says, "Red chair out town board." Which term should the nurse use to document this finding? a. Word salad c. Anhedonia b. Neologism d. Echolalia

ANS: A Word salad (schizophasia) is a jumble of words that is meaningless to the listener and perhaps to the speaker as well, because of an extreme level of disorganization. REF: Page 205-206 TOP: Nursing Process: Assessment

3. Which situations qualify as abandonment on the part of a nurse? (Select all that apply.) The nurse: a. allows a patient with acute mania to refuse hospitalization without taking further action. b. terminates employment without referring a seriously mentally ill for aftercare. c. calls police to bring a suicidal patient to the hospital after a suicide attempt. d. refers a patient with persistent paranoid schizophrenia to community treatment. e. asks another nurse to provide a patient's care because of concerns about countertransference.

ANS: A, B Abandonment arises when a nurse does not place a patient safely in the hands of another health professional before discontinuing treatment. Calling the police to bring a suicidal patient to the hospital after a suicide attempt and referring a patient with schizophrenia to community treatment both provide for patient safety. Asking another nurse to provide a patient's care because of concerns about countertransference demonstrates self-awareness.

2. A patient diagnosed with schizophrenia was hospitalized after arguing with co-workers and threatening to harm them. The patient is aloof, suspicious, and says, "Two staff members I saw talking were plotting to kill me." Based on data gathered at this point, which nursing diagnoses relate? Select all that apply. a. Risk for other-directed violence b. Disturbed thought processes c. Risk for loneliness d. Spiritual distress e. Social isolation

ANS: A, B Delusions of persecution and ideas of reference support the nursing diagnosis of disturbed thought processes. Risk for other-directed violence is substantiated by the patient's feeling endangered by persecutors. Fearful individuals may strike out at perceived persecutors or attempt self-harm to get away from persecutors. Data are not present to support the other diagnoses. REF: Page 209-210 (Table 12-3) TOP: Nursing Process: Diagnosis/Analysis

1. An experienced nurse says to a new graduate, "When you've practiced as long as I have, you will instantly know how to take care of psychotic patients." What is the new graduate's best analysis of this comment? Select all that apply. a. The experienced nurse may have lost sight of patients' individuality, which may compromise the integrity of practice. b. New research findings must be continually integrated into a nurse's practice to provide the most effective care. c. Experience provides mental health nurses with the tools and skills needed for effective professional practice. d. Experienced psychiatric nurses have learned the best ways to care for psychotic patients through trial and error. e. Effective psychiatric nurses should be continually guided by an intuitive sense of patients' needs.

ANS: A, B Evidence-based practice involves using research findings to provide the most effective nursing care. Evidence is continually emerging; therefore, nurses cannot rely solely on experience. The effective nurse also maintains respect for each patient as an individual. Overgeneralization compromises that perspective. Intuition and trial and error are unsystematic approaches to care.

Which assessment findings would the nurse expect in a patient experiencing delirium? Select all that apply. a. Impaired level of consciousness b. Disorientation to place, time c. Wandering attention d. Apathy e. Agnosia

ANS: A, B, C Disorientation to place and time is an expected finding. Orientation to person (self) usually remains intact. Attention span is short, and difficulty focusing or shifting attention as directed is often noted. Patients with delirium commonly experience illusions and hallucinations. Fluctuating levels of consciousness are expected. Agnosia occurs with dementia. Apathy is associated with depression.

1. A nurse can best address factors of critical importance to successful community treatment for persons with mental illness by including assessments related to which of the following? Select all that apply. a. Housing adequacy and stability b. Income adequacy and stability c. Family and other support systems d. Early psychosocial development e. Substance abuse history and current use

ANS: A, B, C, E Early psychosocial developmental history is less relevant to successful outcomes in the community than the assessments listed in the other options. If a patient is homeless or fears homelessness, focusing on other treatment issues is impossible. Sufficient income for basic needs and medication is necessary. Adequate support is a requisite to community placement. Substance abuse undermines medication effectiveness and interferes with community adjustment.

1. A patient states, "I'm starting cognitive-behavioral therapy. What can I expect from the sessions?" Which responses by the nurse would be appropriate? Select all that apply. a. "The therapist will be active and questioning." b. "You will be given some homework assignments." c. "The therapist will ask you to describe your dreams." d. "The therapist will help you look at your ideas and beliefs about yourself." e. "The goal is to increase subjectivity about thoughts that govern your behavior."

ANS: A, B, D Cognitive therapists are active rather than passive during therapy sessions because they help patients reality-test their thinking. Homework assignments are given and completed outside the therapy sessions. Homework is usually discussed at the next therapy session. The goal of cognitive therapy is to assist the patient in identifying inaccurate cognitions and in reality- testing and formulating new, accurate cognitions. One distracter applies to psychoanalysis. Increasing subjectivity is not desirable.

2. A patient states, "I'm starting cognitive behavioral therapy. What can I expect from the sessions?" Which responses by the nurse are appropriate? Select all that apply. a. "The therapist will be active and questioning." b. "You may be given homework assignments." c. "The therapist will ask you to describe your dreams." d. "The therapist will help you look at ideas and beliefs you have about yourself." e. "The goal is to increase your subjectivity about thoughts that govern your behavior."

ANS: A, B, D Cognitive therapists are active rather than passive during therapy sessions because they help patients to reality test their thinking. Homework assignments are given and completed outside the therapy sessions. Homework is usually discussed at the next therapy session. The goals of cognitive therapy are to assist the patient to identify inaccurate cognitions, to reality test their thinking, and to formulate new, accurate cognitions. Dream describing applies to psychoanalysis, not cognitive behavioral therapy. The desired outcome of cognitive therapy is to assist patients in increasing their objectivity, not subjectivity, about the cognitions that influence behavior.

2. Which patient statements identify qualities of nursing practice with high therapeutic value? (Select all that apply.) "My nurse: a. talks in language I can understand." b. helps me keep track of my medications." c. is willing to go to social activities with me." d. lets me do whatever I choose without interfering." e. looks at me as a whole person with different needs."

ANS: A, B, E Each correct answer demonstrates caring is an example of appropriate nursing foci: communicating at a level understandable to the patient, using holistic principles to guide care, and providing medication supervision. The incorrect options suggest a laissez-faire attitude on the part of the nurse, when the nurse should instead provide thoughtful feedback and help patients test alternative solutions or violate boundaries.

A patient diagnosed with moderately severe Alzheimer's disease has a self-care deficit of dressing and grooming. Designate appropriate interventions to include in the patient's plan of care. Select all that apply. a. Provide clothing with elastic and hook-and-loop closures. b. Label clothing with the patient's name and name of the item. c. Administer anti-anxiety medication before bathing and dressing. d. Provide necessary items and direct the patient to proceed independently. e. If the patient resists dressing, use distraction and try again after a short interval.

ANS: A, B, E Providing clothing with elastic and hook-and-loop closures facilitates patient independence. Labeling clothing with the patient's name and the name of the item maintains patient identity and dignity (provides information if the patient has agnosia). When a patient resists, it is appropriate to use distraction and try again after a short interval because patient moods are often labile. The patient may be willing to cooperate given a later opportunity. Providing the necessary items for grooming and directing the patient to proceed independently are inappropriate. Be prepared to coach by giving step-by-step directions for each task as it occurs. Administering anxiolytic medication before bathing and dressing is inappropriate. This measure would result in unnecessary overmedication.

1. A basic level registered nurse works with patients in a community setting. Which groups should this nurse expect to lead? Select all that apply. a. Symptom management b. Medication education c. Family therapy d. Psychotherapy e. Self-care

ANS: A, B, E Symptom management, medication education, and self-care groups represent psychoeducation, a service provided by the basic level registered nurse. Advanced practice registered nurses provide family therapy and psychotherapy.

2. A community member asks a nurse, "People diagnosed with mental illnesses used to go to a state hospital. Why has that changed?" Select the nurse's accurate responses. Select all that apply. a. "Science has made significant improvements in drugs for mental illness, so now many people may live in their communities." b. "A better selection of less restrictive settings is now available in communities to care for individuals with mental illness." c. "National rates of mental illness have declined significantly. The need for state institutions is actually no longer present." d. "Most psychiatric institutions were closed because of serious violations of patients' rights and unsafe conditions." e. "Federal legislation and payment for treatment of mental illness have shifted the focus to community rather than institutional settings."

ANS: A, B, E The community is a less restrictive alternative than hospitals for the treatment of people with mental illness. Funding for treatment of mental illness remains largely inadequate but now focuses on community rather than institutional care. Antipsychotic medications improve more symptoms of mental illness; hence, management of psychiatric disorders has improved. Rates of mental illness have increased, not decreased. Hospitals were closed because funding shifted to the community. Conditions in institutions have improved.

3. An individual is experiencing problems associated with memory. Which cerebral structures are most likely to be involved in this deficit? Select all that apply. a. Prefrontal cortex b. Occipital lobe c. Temporal lobe d. Parietal lobe e. Basal ganglia

ANS: A, C, D The prefrontal cortex, parietal, and temporal lobes of the cerebrum play a key role in the storage and processing of memories. The occipital lobe is predominantly involved with vision. The basal ganglia influence the integration of physical movement, as well as some thoughts and emotions.

Which comments by an elderly person best indicate successful completion of the developmental task? Select all that apply. a. "I am proud of my children's successes in life." b. "I should have given to community charities more often." c. "My relationship with my father made life more difficult for me." d. "My experiences in the war helped me appreciate the meaning of life." e. "I often wonder what would have happened if I had chosen a different career."

ANS: A, D The developmental crisis for an elderly person relates to integrity versus despair. Pride in one's offspring indicates a sense of fulfillment. Recognition of the wisdom gained from difficult experiences (such as being in a war) indicates a sense of integrity. Blaming and regret indicate despair and unsuccessful resolution of the crisis.

Which comments by an adult best indicate self-actualization? Select all that apply. a. "I am content with a good book." b. "I often wonder if I chose the right career." c. "Sometimes I think about how my parents would have handled problems." d. "It's important for our country to provide basic health care services for everyone." e. "When I was lost at sea for 2 days, I gained an understanding of what is important."

ANS: A, D, E Self-actualized persons enjoy privacy, have a sense of democracy, and show positive outcomes associated with peak experiences. Self-doubt, defensiveness, and blaming are not consistent with self-actualization.

1. A nurse prepares to administer an antipsychotic medication to a patient diagnosed with schizophrenia. Additional monitoring of the medication's effects and side effects will be most important if the patient is also diagnosed with which health problem? Select all that apply. a. Parkinson disease b. Graves disease c. Osteoarthritis d. Epilepsy e. Diabetes

ANS: A, D, E Antipsychotic medications may produce weight gain, which complicates the care of a patient with diabetes or lowers the seizure threshold or both, which complicates the care of a patient with epilepsy. Parkinson disease involves changes in transmission of dopamine and acetylcholine; therefore these drugs also complicate the care of a patient with the disorder. Osteoarthritis and Graves disease should have no synergistic effect with this medication.

1. A nurse at the mental health clinic plans a series of psychoeducational groups for persons newly diagnosed with schizophrenia. Which two topics take priority? a. "The importance of taking your medication correctly" b. "How to complete an application for employment" c. "How to dress when attending community events" d. "How to give and receive compliments" e. "Ways to quit smoking"

ANS: A, E Stabilization is maximized by adherence to the antipsychotic medication regimen. Because so many persons with schizophrenia smoke cigarettes, this topic relates directly to the patients' physiological well-being. The other topics are also important but are not priority topics. REF: Page 204 | Page 211-212 | Page 215 (Box 12-6) | Page 224

Which patient statement would lead the nurse to suspect unsuccessful completion of the developmental task of infancy? a. "I have very warm and close friendships." b. "I'm afraid to allow anyone to really get to know me." c. "I'm always absolutely right, so don't bother saying more." d. "I'm ashamed that I didn't do things correctly in the first place."

ANS: B According to Erikson, the developmental task of infancy is the development of trust. The correct response is the only statement clearly showing lack of ability to trust others. Warm, close relationships suggest the developmental task of infancy was successfully completed; rigidity and self-absorption are reflected in the belief one is always right; and shame for past actions suggests failure to resolve the crisis of initiative versus guilt.

A patient says, "All my life I've been surrounded by stupidity. Everything I buy breaks because the entire American workforce is incompetent." This patient is experiencing a: a. self-esteem deficit. b. cognitive distortion. c. deficit in motivation. d. deficit in love and belonging.

ANS: B Automatic thoughts, or cognitive distortions, are irrational and lead to false assumptions and misinterpretations. See related audience response question.

A patient says, "I always feel good when I wear a size 2 petite." Which type of cognitive distortion is evident? a. Disqualifying the positive b. Overgeneralization c. Catastrophizing d. Personalization

ANS: B Automatic thoughts, or cognitive distortions, are irrational and lead to false assumptions and misinterpretations. The stem offers an example of overgeneralization. See related audience response question.

A patient is fearful of riding on elevators. The therapist first rides an escalator with the patient. The therapist and patient then stand in an elevator with the door open for five minutes and later with the elevator door closed for five minutes. Which technique has the therapist used? a. Classic psychoanalytic therapy b. Systematic desensitization c. Rational emotive therapy d. Biofeedback

ANS: B Systematic desensitization is a form of behavior modification therapy that involves the development of behavior tasks customized to the patient's specific fears. These tasks are presented to the patient while using learned relaxation techniques. The patient is incrementally exposed to the fear.

A student nurse says, "I don't need to interact with my patients. I learn what I need to know by observation." An instructor can best interpret the nursing implications of Sullivan's theory to this student by responding: a. "Interactions are required in order to help you develop therapeutic communication skills." b. "Nurses cannot be isolated. We must interact to provide patients with opportunities to practice interpersonal skills." c. "Observing patient interactions will help you formulate priority nursing diagnoses and appropriate interventions." d. "It is important to pay attention to patients' behavioral changes, because these signify adjustments in personality."

ANS: B The nurse's role includes educating patients and assisting them in developing effective interpersonal relationships. Mutuality, respect for the patient, unconditional acceptance, and empathy are cornerstones of Sullivan's theory. The nurse who does not interact with the patient cannot demonstrate these cornerstones. Observations provide only objective data. Priority nursing diagnoses usually cannot be accurately established without subjective data from the patient. The other distracters relate to Maslow and behavioral theory. This item relates to an audience response question.

A nurse influenced by Peplau's interpersonal theory works with an anxious, withdrawn patient. Interventions should focus on: a. rewarding desired behaviors. b. use of assertive communication. c. changing the patient's self-concept. d. administering medications to relieve anxiety.

ANS: B The nurse-patient relationship is structured to provide a model for adaptive interpersonal relationships that can be generalized to others. Helping the patient learn to use assertive communication will improve the patient's interpersonal relationships. The distracters apply to theories of cognitive, behavioral, and biological therapy.

The parent of a child diagnosed with schizophrenia tearfully asks the nurse, "What could I have done differently to prevent this illness?" Select the nurse's best response. a. "Although schizophrenia results from impaired family relationships, try not to feel guilty. No one can predict how a child will respond to parental guidance." b. "Schizophrenia is a biological illness resulting from changes in how the brain and nervous system function. You are not to blame for your child's illness." c. "There is still hope. Changing your parenting style can help your child learn to cope effectively with the environment." d. "Most mental illnesses result from genetic inheritance. Your genes are more at fault than your parenting."

ANS: B The parent's comment suggests feelings of guilt or inadequacy. The nurse's response should address these feelings as well as provide information. Patients and families need reassurance that the major mental disorders are biological in origin and are not the "fault" of parents. One distracter places the burden of having faulty genes on the shoulders of the parents. The other distracters are neither wholly accurate nor reassuring.

A patient had psychotherapy weekly for 5 months. The therapist used free association, dream analysis, and facilitated transference to help the patient understand conflicts and foster change. Select the term that applies to this method. a. Rational-emotive behavior therapy b. Psychodynamic psychotherapy c. Cognitive-behavioral therapy d. Operant conditioning

ANS: B The techniques are aspects of psychodynamic psychotherapy. The distracters use other techniques.

A parent says, "My 2-year-old child refuses toilet training and shouts 'No!' when given directions. What do you think is wrong?" Select the nurse's best reply. a. "Your child needs firmer control. It is important to set limits now." b. "This is normal for your child's age. The child is striving for independence." c. "There may be developmental problems. Most children are toilet trained by age 2." d. "Some undesirable attitudes are developing. A child psychologist can help you develop a plan."

ANS: B This behavior is typical of a child around the age of 2 years, whose developmental task is to develop autonomy. The distracters indicate the child's behavior is abnormal.

A patient says to the nurse, "My father has been dead for over 10 years, but talking to you is almost as comforting as the talks he and I had when I was a child." Which term applies to the patient's comment? a. Superego b. Transference c. Reality testing d. Counter-transference

ANS: B Transference refers to feelings a patient has toward the health care workers that were originally held toward significant others in his or her life. Counter-transference refers to unconscious feelings that the health care worker has toward the patient. The superego represents the moral component of personality; it seeks perfection.

Which assessment finding would be likely for a patient experiencing a hallucination? The patient: a. looks at shadows on a wall and says, "I see scary faces." b. states, "I feel bugs crawling on my legs and biting me." c. reports telepathic messages from the television. d. speaks in rhymes.

ANS: B A hallucination is a false sensory perception occurring without a corresponding sensory stimulus. Feeling bugs on the body when none are present is a tactile hallucination. Misinterpreting shadows as faces is an illusion. An illusion is a misinterpreted sensory perception. The other incorrect options apply to thought insertion and clang associations.

14. A voluntarily hospitalized patient tells the nurse, "Get me the forms for discharge against medical advice so I can leave now." What is the nurse's best initial response? a. "I can't give you those forms without your health care provider's knowledge." b. "I will get them for you, but let's talk about your decision to leave treatment." c. "Since you signed your consent for treatment, you may leave if you desire." d. "I'll get the forms for you right now and bring them to your room."

ANS: B A patient who has been voluntarily admitted as a psychiatric inpatient has the right to demand and obtain release in most states. However, as a patient advocate, the nurse is responsible for weighing factors related to the patient's wishes and best interests. By asking for information, the nurse may be able to help the patient reconsider the decision. The statement that discharge forms cannot be given without the health care provider's knowledge is not true. Facilitating discharge without consent is not in the patient's best interest before exploring the reason for the request.

3. A patient is admitted to the psychiatric hospital. Which assessment finding best indicates that the patient has a mental illness? The patient: a. describes coping and relaxation strategies used when feeling anxious. b. describes mood as consistently sad, discouraged, and hopeless. c. can perform tasks attempted within the limits of own abilities. d. reports occasional problems with insomnia.

ANS: B A patient who reports having a consistently negative mood is describing a mood alteration. The incorrect options describe mentally healthy behaviors and common problems that do not indicate mental illness.

8. Which patient statement would lead a nurse to suspect that the developmental task of infancy was not successfully completed? a. "I have very warm and close friendships." b. "I'm afraid to let anyone really get to know me." c. "I am always right and confident about my decisions." d. "I'm ashamed that I didn't do it correctly in the first place."

ANS: B According to Erikson, the developmental task of infancy is the development of trust. The patient's statement that he or she is afraid of becoming acquainted with others clearly shows a lack of ability to trust other people. Having warm and close friendships suggests the developmental task of infancy was successfully completed. Believing one is always right suggests rigidity rather than mistrust. Feelings of shame suggest failure to resolve the crisis of Initiative versus Guilt.

Consider these diagnostic findings: apolipoprotein E (apoE) malfunction, neurofibrillary tangles, neuronal degeneration in the hippocampus, and brain atrophy. Which health problem corresponds to these diagnostic findings? a. Huntington's disease b. Alzheimer's disease c. Parkinson's disease d. Vascular dementia

ANS: B All of the options relate to dementias; however, the pathophysiological phenomena described apply to Alzheimer's disease. Parkinson's disease is associated with dopamine dysregulation. Huntington's disease is genetic. Vascular dementia is the consequence of circulatory changes.

12. Which historical nursing leader helped focus practice to recognize the importance of science in psychiatric nursing? a. Abraham Maslow b. Hildegard Peplau c. Kris Martinsen d. Harriet Bailey

ANS: B Although all these leaders included science as an important component of practice, Hildegard Peplau most influenced its development in psychiatric nursing. Maslow was not a nurse, but his theories influence how nurses prioritize problems and care. Bailey wrote a textbook in the 1930s on psychiatric nursing interventions. Kris Martinsen emphasized the importance of caring in nursing practice.

21. After leaving work, a staff nurse realizes that documentation of the administration of a medication to a patient was omitted. This off-duty nurse telephones the unit and tells the nurse, "Please document the administration of the medication I forgot to do. My password is alpha1." The nurse should: a. fulfill the request. b. refer the matter to the charge nurse to resolve. c. access the record and document the information. d. report the request to the patient's health care provider.

ANS: B At most hospitals, termination is a possible penalty for unauthorized entry into a patient record. Referring the matter to the charge nurse will allow the observance of hospital policy while ensuring that documentation occurs. Making an exception and fulfilling the request places the on-duty staff nurse in jeopardy. Reporting the request to the patient's health care provider would be unnecessary. Accessing the record and documenting the information would be unnecessary when the charge nurse can resolve the problem.

What is the priority intervention for a patient diagnosed with delirium who has fluctuating levels of consciousness, disturbed orientation, and perceptual alterations? a. Distraction using sensory stimulation b. Careful observation and supervision c. Avoidance of physical contact d. Activation of the bed alarm

ANS: B Careful observation and supervision are of ultimate importance because an appropriate outcome would be that the patient will remain safe and free from injury. Physical contact during care cannot be avoided. Activating a bed alarm is only one aspect of providing for the patient's safety.

6. A patient's care plan includes monitoring for auditory hallucinations. Which assessment findings suggest the patient may be hallucinating? a. Detachment and overconfidence b. Darting eyes, tilted head, mumbling to self c. Euphoric mood, hyperactivity, distractibility d. Foot tapping and repeatedly writing the same phrase

ANS: B Clues to hallucinations include eyes looking around the room as though to find the speaker, tilting the head to one side as though listening intently, and grimacing, mumbling, or talking aloud as though responding conversationally to someone. REF: Page 206-207 | Page 212-213 TOP: Nursing Process: Assessment

During morning care, a nurse asks a patient diagnosed with dementia, "How was your night?" The patient replies, "It was lovely. I went out to dinner and a movie with my friend." Which term applies to the patient's response? a. Sundown syndrome b. Confabulation c. Perseveration d. Delirium

ANS: B Confabulation refers to making up of stories or answers to questions by a person who does not remember. It is a defensive tactic to protect self-esteem and prevent others from noticing memory loss. The patient's response was not sundown syndrome. Perseveration refers to repeating a word or phrase over and over. Delirium is not present in this scenario.

22. A patient's spouse, who is a chemist, asks a nurse how serotonin reuptake inhibitors (SSRIs) lift depression. The nurse should explain that SSRIs: a. destroy increased amounts of neurotransmitters. b. make more serotonin available at the synaptic gap. c. increase production of acetylcholine and dopamine. d. block muscarinic and alpha1-norepinephrine receptors.

ANS: B Depression is thought to be related to the lowered availability of the neurotransmitter serotonin. SSRIs act by blocking the reuptake of serotonin, leaving a higher concentration available at the synaptic cleft. They actually prevent the destruction of serotonin, have no effect on acetylcholine and dopamine production, and do not block muscarinic or alpha1-norepinephrine receptors.

7. Which nursing intervention demonstrates false imprisonment? a. A confused and combative patient says, "I'm getting out of here and no one can stop me." The nurse restrains this patient without a health care provider's order and then promptly obtains an order. b. A patient has been irritating, seeking the attention of nurses most of the day. Now a nurse escorts the patient down the hall, saying, "Stay in your room or you'll be put in seclusion." c. An involuntarily hospitalized patient with suicidal ideation runs out of the psychiatric unit. A nurse rushes after the patient and convinces the patient to return to the unit. d. An involuntarily hospitalized patient with suicidal ideation attempts to leave the unit. A nurse calls the security team and uses established protocols to prevent the patient from leaving.

ANS: B False imprisonment involves holding a competent person against his or her will. Actual force is not a requirement of false imprisonment. The individual needs only to be placed in fear of imprisonment by someone who has the ability to carry out the threat. The patient in one distractor is not competent, and the nurse is acting beneficently. The patients in the other distractors have been admitted as involuntary patients and should not be allowed to leave without permission of the treatment team.

5. A patient shows the nurse an article from the Internet about a health problem. Which characteristic of the web site's address most alerts the nurse that the site may have biased and prejudiced information? a. Address ends in ".org." b. Address ends in ".com." c. Address ends in ".gov." d. Address ends in ".net."

ANS: B Financial influences on a site are a clue that the information may be biased. ".com" at the end of the address indicates that the site is a commercial one. ".gov" indicates that the site is maintained by a government entity. ".org" indicates that the site is nonproprietary; the site may or may not have reliable information, but it does not profit from its activities. ".net" can have multiple meanings.

14. A patient is hospitalized for major depressive disorder. Of the medications listed, a nurse can expect to provide the patient with teaching about: a. chlordiazepoxide (Librium). b. fluoxetine (Prozac). c. clozapine (Clozaril). d. tacrine (Cognex).

ANS: B Fluoxetine is a selective serotonin reuptake inhibitor (SSRI), an antidepressant that blocks the reuptake of serotonin with few anticholinergic and sedating side effects; clozapine (Clozaril) is an antipsychotic medication; chlordiazepoxide (Librium) is an anxiolytic drug; and tacrine (Cognex) is used to treat Alzheimer disease.

13. A community mental health nurse has worked for 6 months to establish a relationship with a delusional, suspicious patient. The patient recently lost employment and stopped taking medications because of inadequate money. The patient says, "Only a traitor would make me go to the hospital." Which solution is best? a. Arrange a bed in a local homeless shelter with nightly onsite supervision. b. Negotiate a way to provide medication so the patient can remain at home. c. Hospitalize the patient until the symptoms have stabilized. d. Seek inpatient hospitalization for up to 1 week.

ANS: B Hospitalization may damage the nurse-patient relationship even if it provides an opportunity for rapid stabilization. If medication can be obtained and restarted, the patient can possibly be stabilized in the home setting, even if it takes a little longer. A homeless shelter is inappropriate and unnecessary. Hospitalization may be necessary later, but a less restrictive solution should be tried first because the patient is not dangerous.

2. A newly admitted patient diagnosed with schizophrenia is hypervigilant and constantly scans the environment. The patient states, "I saw two doctors talking in the hall. They were plotting to kill me." The nurse may correctly assess this behavior as: a. echolalia. c. a delusion of infidelity. b. an idea of reference. d. an auditory hallucination.

ANS: B Ideas of reference are misinterpretations of the verbalizations or actions of others that give special personal meanings to these behaviors for example, when seeing two people talking, the individual assumes they are talking about him or her. The other terms do not correspond with the scenario. REF: Page 206 (Table 12-1) TOP: Nursing Process: Assessment

2. A 26-month-old child displays negative behavior, refuses toilet training, and often shouts, "No!" when given directions. Using Freud's stages of psychosexual development, a nurse would assess the child's behavior is based on which stage? a. Oral b. Anal c. Phallic d. Genital

ANS: B In Freud's stages of psychosexual development, the anal stage occurs from age 1 to 3 years and has, as its focus, toilet training and learning to delay immediate gratification. The oral stage occurs between birth and 1 year, the phallic stage occurs between 3 and 5 years, and the genital stage occurs between 13 and 20 years.

24. A patient diagnosed with schizophrenia begins a new prescription for lurasidone HCL (Latuda). The patient is 5'6" and currently weighs 204 lbs. Which topic is most important for the nurse to include in the teaching plan related to this medication? a. How to recognize tardive dyskinesia c. Ways to manage constipation b. Weight management strategies d. Sleep hygiene measures

ANS: B Lurasidone HCL (Latuda) is a second-generation antipsychotic medication. The incidence of weight gain, diabetes, and high cholesterol is high with this medication. The patient is overweight now, so weight management will be especially important. The incidence of tardive dyskinesia is low with second-generation antipsychotic medications. Constipation may occur, but it is less important than weight management. This drug usually produces drowsiness. REF: Page 218-219 (Table 12-5) TOP: Nursing Process: Planning

37. A patient insistently states, "I can decipher codes of DNA just by looking at someone." Which problem is evident? a. Visual hallucinations c. Idea of reference b. Magical thinking d. Thought insertion

ANS: B Magical thinking is evident in the patient's appraisal of his own abilities. There is no evidence of the distracters. REF: Page 205-206 TOP: Nursing Process: Assessment

11. A patient taking medication for mental illness develops restlessness and an uncontrollable need to be in motion. A nurse can correctly analyze that these symptoms are related to which drug action? a. Anticholinergic effects b. Dopamine-blocking effects c. Endocrine-stimulating effects d. Ability to stimulate spinal nerves

ANS: B Medications that block dopamine often produce disturbances of movement such as akathisia because dopamine affects neurons involved in both the thought processes and movement regulation. Anticholinergic effects include dry mouth, blurred vision, urinary retention, and constipation. Akathisia is not caused by endocrine stimulation or spinal nerve stimulation.

A patient with stage 3 Alzheimer's disease tires easily and prefers to stay home rather than attend social activities. The spouse does the grocery shopping because the patient cannot remember what to buy. Which nursing diagnosis applies at this time? a. Self-care deficit b. Impaired memory c. Caregiver role strain d. Adult failure to thrive

ANS: B Memory impairment begins at stage 2 and progresses in stage 3. This patient is able to perform most self-care activities. Caregiver role strain and adult failure to thrive occur later.

26. A patient diagnosed with schizophrenia has taken a conventional antipsychotic medication for a year. Hallucinations are less intrusive, but the patient continues to have apathy, poverty of thought, and social isolation. The nurse would expect a change to which medication? a. Haloperidol (Haldol) c. Chlorpromazine (Thorazine) b. Olanzapine (Zyprexa) d. Diphenhydramine (Benadryl)

ANS: B Olanzapine is a second-generation atypical antipsychotic that targets both positive and negative symptoms of schizophrenia. Haloperidol and chlorpromazine are conventional antipsychotics that target only positive symptoms. Diphenhydramine is an antihistamine. See relationship to audience response question. REF: Page 219 TOP: Nursing Process: Planning

10. A patient diagnosed with schizophrenia demonstrates little spontaneous movement and has waxy flexibility. The patient's activities of daily living are severely compromised. An appropriate outcome would be that the patient will: a. demonstrate increased interest in the environment by the end of week 1. b. perform self-care activities with coaching by the end of day 3. c. gradually take the initiative for self-care by the end of week 2. d. accept tube feeding without objection by day 2.

ANS: B Outcomes related to self-care deficit nursing diagnoses should deal with increasing ability to perform self-care tasks independently, such as feeding, bathing, dressing, and toileting. Performing the tasks with coaching by nursing staff denotes improvement over the complete inability to perform the tasks. The incorrect options are not directly related to self-care activities, difficult to measure, and unrelated to maintenance of nutrition. REF: Page 209-210 TOP: Nursing Process: Outcomes Identification

A nurse counsels the family of a patient diagnosed with Alzheimer's disease who lives at home and wanders at night. Which action is most important for the nurse to recommend to enhance safety? a. Apply a medical alert bracelet to the patient. b. Place locks at the tops of doors. c. Discourage daytime napping. d. Obtain a bed with side rails.

ANS: B Placing door locks at the top of the door makes it more difficult for the patient with dementia to unlock the door because the ability to look up and reach upward is diminished. The patient will try to climb over side rails, increasing the risk for injury and falls. Avoiding daytime naps may improve the patient's sleep pattern but does not assure safety. A medical alert bracelet will be helpful if the patient leaves the home, but it does not prevent wandering or assure the patient's safety.

1. Which outcome, focused on recovery, would be expected in the plan of care for a patient living in the community and diagnosed with serious and persistent mental illness? Within 3 months, the patient will: a. deny suicidal ideation. b. report a sense of well-being. c. take medications as prescribed. d. attend clinic appointments on time.

ANS: B Recovery emphasizes managing symptoms, reducing psychosocial disability, and improving role performance. The goal of recovery is to empower the individual with mental illness to achieve a sense of meaning and satisfaction in life and to function at the highest possible level of wellness. The incorrect options focus on the classic medical model rather than recovery.

A patient with severe dementia no longer recognizes family members and becomes anxious and agitated when they attempt reorientation. Which alternative could the nurse suggest to the family members? a. Wear large name tags. b. Focus interaction on familiar topics. c. Frequently repeat the reorientation strategies. d. Place large clocks and calendars strategically.

ANS: B Reorientation may seem like arguing to a patient with cognitive deficit and increases the patient's anxiety. Validating, talking with the patient about familiar, meaningful things, and reminiscing give meaning to existence both for the patient and family members. The option that suggests using validating techniques when communicating is the only option that addresses an interactional strategy. Wearing large name tags and placing large clocks and calendars strategically are reorientation strategies. Frequently repeating the reorientation strategies is inadvisable because patients with dementia sometimes become more agitated with reorientation.

8. Which research evidence would most influence a group of nurses to change their practice? a. Expert committee report of recommendations for practice b. Systematic review of randomized controlled trials c. Nonexperimental descriptive study d. Critical pathway

ANS: B Research findings are graded using a hierarchy of evidence. A systematic review of randomized controlled trials is Level A and provides the strongest evidence for changing practice. Expert committee recommendations and descriptive studies lend less powerful and influential evidence. A critical pathway is not evidence; it incorporates research findings after they have been analyzed.

Chapter 12: Schizophrenia and Schizophrenia Spectrum Disorders 1. A person has had difficulty keeping a job because of arguing with co-workers and accusing them of conspiracy. Today the person shouts, "They're all plotting to destroy me. Isn't that true?" Select the nurse's most therapeutic response. a. "Everyone here is trying to help you. No one wants to harm you." b. "Feeling that people want to destroy you must be very frightening." c. "That is not true. People here are trying to help you if you will let them." d. "Staff members are health care professionals who are qualified to help you."

ANS: B Resist focusing on content instead, focus on the feelings the patient is expressing. This strategy prevents arguing about the reality of delusional beliefs. Such arguments increase patient anxiety and the tenacity with which the patient holds to the delusion. The other options focus on content and provide opportunity for argument. REF: Page 205-206 | Page 213-215 (Box 12-4)

17. A community psychiatric nurse assesses that a patient diagnosed with a mood disorder is more depressed than on the previous visit a month ago; however, the patient says, "I feel the same." Which intervention supports the nurse's assessment while preserving the patient's autonomy? a. Arrange for a short hospitalization. b. Schedule weekly clinic appointments. c. Refer the patient to the crisis intervention clinic. d. Call the family and ask them to observe the patient closely.

ANS: B Scheduling clinic appointments at shorter intervals will give the opportunity for more frequent assessment of symptoms and allow the nurse to use early intervention. If the patient does not admit to having a crisis or problem, a referral would be useless. The remaining options may produce unreliable information, violate the patient's privacy, and waste scarce resources.

1. A patient asks a nurse, "What are neurotransmitters? My doctor says mine are out of balance." The best reply would be: a. "You must feel relieved to know that your problem has a physical basis." b. "Neurotransmitters are chemicals that pass messages between brain cells." c. "It is a high-level concept to explain. You should ask the doctor to tell you more." d. "Neurotransmitters are substances we eat daily that influence memory and mood."

ANS: B Stating that neurotransmitters are chemicals that pass messages between brain cells gives the most accurate information. Neurotransmitters are messengers in the central nervous system. They are released from the axon terminal, diffuse across the synapse, and attach to specialized receptors on the postsynaptic neuron. The incorrect responses do not answer the patient's question, are demeaning, and provide untrue and misleading information.

2. Which organization actively seeks to reduce the stigma associated with mental illness through public presentations such as "In Our Own Voice" (IOOV)? a. American Psychiatric Association (APA) b. National Alliance on Mental Illness (NAMI) c. United States Department of Health and Human Services (USDHHS) d. North American Nursing Diagnosis Association International (NANDA-I)

ANS: B Stigma represents the bias and prejudice commonly held regarding mental illness. NAMI actively seeks to dispel misconceptions about mental illness. NANDA-I defines approved nursing diagnoses. The APA publishes the DSM 5. The USDHHS regulates and administers health policies.

19. A patient took trifluoperazine 30 mg po daily for 3 years. The clinic nurse notes that the patient grimaces and constantly smacks both lips. The patient's neck and shoulders twist in a slow, snakelike motion. Which problem would the nurse suspect? a. Agranulocytosis c. Tourette's syndrome b. Tardive dyskinesia d. Anticholinergic effects

ANS: B Tardive dyskinesia is a neuroleptic-induced condition involving the face, trunk, and limbs. Involuntary movements, such as tongue thrusting licking, blowing, irregular movements of the arms, neck, and shoulders, rocking, hip jerks and pelvic thrusts, are seen. These symptoms are frequently not reversible even when the drug is discontinued. The scenario does not present evidence consistent with the other disorders mentioned. Agranulocytosis is a blood disorder. Tourette's syndrome is a condition in which tics are present. Anticholinergic effects include dry mouth, blurred vision, flushing, constipation, and dry eyes. REF: Page 215-216 (Table 12-4) TOP: Nursing Process: Evaluation

An elderly patient is admitted with delirium secondary to a urinary tract infection. The family asks whether the patient will ever recover. Select the nurse's best response. a. "The health care provider is the best person to answer your question." b. "The confusion will probably get better as we treat the infection." c. "Unfortunately, delirium is a progressively disabling disorder." d. "I will be glad to contact the chaplain to talk with you."

ANS: B Usually, as the underlying cause of the delirium is treated, the symptoms of delirium clear. The distracters mislead the family.

6. A nurse's neighbor asks, "Why aren't people with mental illness kept in state institutions anymore?" What is the nurse's best response? a. "Many people are still in psychiatric institutions. Inpatient care is needed because many people who are mentally ill are violent." b. "Less restrictive settings are now available to care for individuals with mental illness." c. "Our nation has fewer persons with mental illness; therefore fewer hospital beds are needed." d. "Psychiatric institutions are no longer popular as a consequence of negative stories in the press."

ANS: B The community is a less restrictive alternative than hospitals for the treatment of people with mental illness. The remaining options are incorrect and part of the stigma of mental illness.

5. Which hallucination necessitates the nurse to implement safety measures? The patient says, a. "I hear angels playing harps." b. "The voices say everyone is trying to kill me." c. "My dead father tells me I am a good person." d. "The voices talk only at night when I'm trying to sleep."

ANS: B The correct response indicates the patient is experiencing paranoia. Paranoia often leads to fearfulness, and the patient may attempt to strike out at others to protect self. The distracters are comforting hallucinations or do not indicate paranoia. REF: Page 207 | Page 212-213 TOP: Nursing Process: Planning

20. A patient diagnosed with schizophrenia believes evil spirits are being summoned by a local minister and verbally threatens to bomb a local church. The psychiatrist notifies the minister. The psychiatrist has: a. released information without proper authorization. b. demonstrated the duty to warn and protect. c. violated the patient's confidentiality. d. avoided charges of malpractice.

ANS: B The duty of a health care professional is to warn or notify an intended victim after a threat of harm has been made. Informing a potential victim of a threat is a legal responsibility of the health care professional and not considered a violation of confidentiality.

Consider these health problems: Lewy body disease, frontal-temporal lobar degeneration, and Huntington's disease. Which term unifies these problems? a. Cyclothymia b. Dementia c. Delirium d. Amnesia

ANS: B The listed health problems are all forms of dementia.

12. A patient diagnosed with schizophrenia has been stable in the community. Today, the spouse reports the patient is expressing delusional thoughts. The patient says, "I'm willing to take my medicine, but I forgot to get my prescription refilled." Which outcome should the nurse add to the plan of care? a. Nurse will obtain prescription refills every 90 days and deliver them to the patient. b. Patient's spouse will mark dates for prescription refills on the family calendar. c. Patient will report to the hospital for medication follow-up every week. d. Patient will call the nurse weekly to discuss medication-related issues.

ANS: B The nurse should use the patient's support system to meet patient needs whenever possible. Delivery of medication by the nurse should be unnecessary for the nurse to do if the patient or a significant other can be responsible. The patient may not need more intensive follow-up as long as he or she continues to take the medications as prescribed. No patient issues except failure to obtain medication refills were identified.

18. A nurse uses Peplau's interpersonal therapy while working with an anxious, withdrawn patient. Interventions should focus on: a. changing the patient's perceptions about self. b. improving the patient's interactional skills. c. using medications to relieve anxiety. d. reinforcing specific behaviors.

ANS: B The nurse-patient relationship is structured to provide a model for adaptive interpersonal relationships that can be generalized to others. Changing the patient's perceptions about his- or herself would be appropriate for cognitive therapy. Reinforcing specific behaviors would be used in behavioral therapy. Using medications is the focus of biological therapy.

35. A client says, "Facebook has a new tracking capacity. If I use the Internet, Homeland Security will detain me as a terrorist." Select the nurse's best initial action. a. Tell the client, "Facebook is a safe website. You don't need to worry about Homeland Security." b. Tell the client, "You are in a safe place where you will be helped." c. Administer a prn dose of an antipsychotic medication. d. Tell the client, "You don't need to worry about that."

ANS: B The patient is experiencing paranoia and delusional thinking, which leads to fear. Explaining that the patient is in a safe place will help relieve the fear. It is not therapeutic to disagree or give advice. Medication will not relieve the immediate concern. REF: Page 205-206 (Table 12-1) | Page 213-215 (Box 12-4)

3. A patient diagnosed with schizophrenia says, "My co-workers are out to get me. I also saw two doctors plotting to kill me." How does this patient perceive the environment? a. Disorganized c. Supportive b. Dangerous d. Bizarre

ANS: B The patient sees the world as hostile and dangerous. This assessment is important because the nurse can be more effective by using empathy to respond to the patient. Data are not present to support any of the other options. REF: Page 210 (Table 12-3) | Page 213 (Box 12-4)

7. Two nursing students discuss career plans after graduation. One student wants to enter psychiatric nursing. The other student asks, "Why would you want to be a psychiatric nurse? All they do is talk. You will lose your skills." Select the best response by the student interested in psychiatric nursing. a. "Psychiatric nurses practice in safer environments than other specialties. Nurse-to-patient ratios must be better because of the nature of patients' problems." b. "Psychiatric nurses use complex communication skills, as well as critical thinking, to solve multidimensional problems. I'm challenged by those situations." c. "I think I will be good in the mental health field. I do not like clinical rotations in school, so I do not want to continue them after I graduate." d. "Psychiatric nurses do not have to deal with as much pain and suffering as medical surgical nurses. That appeals to me."

ANS: B The practice of psychiatric nursing requires a different set of skills than medical surgical nursing, although substantial overlap does exist. Psychiatric nurses must be able to help patients with medical and mental health problems, reflecting the holistic perspective these nurses must have. Nurse-patient ratios and workloads in psychiatric settings have increased, similar to other specialties. Psychiatric nursing involves clinical practice, not simply documentation. Psychosocial pain is real and can cause as much suffering as physical pain.

15. A patient hospitalized with a mood disorder has aggression, agitation, talkativeness, and irritability. A nurse begins the care plan based on the expectation that the health care provider is most likely to prescribe a medication classified as a(n): a. anticholinergic. b. mood stabilizer. c. psychostimulant. d. tricyclic antidepressant.

ANS: B The symptoms describe a manic attack. Mania is effectively treated by the antimanic drug lithium and selected anticonvulsants such as carbamazepine, valproic acid, and lamotrigine. No drugs from the other classifications listed are effective in the treatment of mania.

11. A nurse observes a catatonic patient standing immobile, facing the wall with one arm extended in a salute. The patient remains immobile in this position for 15 minutes, moving only when the nurse gently lowers the arm. What is the name of this phenomenon? a. Echolalia c. Depersonalization b. Waxy flexibility d. Thought withdrawal

ANS: B Waxy flexibility is the ability to hold distorted postures for extended periods of time, as though the patient were molded in wax. Echolalia is a speech pattern. Depersonalization refers to a feeling state. Thought withdrawal refers to an alteration in thinking. REF: Page 207-208 TOP: Nursing Process: Assessment

Which activities represent the art of nursing? Select all that apply. a. Administering medications on time to a group of patients b. Listening to a new widow grieve her husband's death c. Helping a patient obtain groceries from a food bank d. Teaching a patient about a new medication e. Holding the hand of a frightened patient

ANS: B, C, E Peplau described the science and art of professional nursing practice. The art component of nursing consists of the care, compassion, and advocacy nurses provide to enhance patient comfort and well-being. The science component of nursing involves the application of knowledge to understand a broad range of human problems and psychosocial phenomena, intervening to relieve patients' suffering and promote growth. See related audience response question.

1. A patient in the emergency department reports, "I hear voices saying someone is stalking me. They want to kill me because I found the cure for cancer. I will stab anyone that threatens me." Which aspects of mental health have the greatest immediate concern to a nurse? Select all that apply. a. Happiness b. Appraisal of reality c. Control over behavior d. Effectiveness in work e. Healthy self-concept

ANS: B, C, E The aspects of mental health of greatest concern are the patient's appraisal of and control over behavior. The patient's appraisal of reality is inaccurate, and auditory hallucinations are evident, as well as delusions of persecution and grandeur. In addition, the patient's control over behavior is tenuous, as evidenced by the plan to "stab" anyone who seems threatening. A healthy self-concept is lacking. Data are not present to suggest that the other aspects of mental health (happiness and effectiveness in work) are of immediate concern.

Operant conditioning is part of the treatment plan to encourage speech in a child who is nearly mute. Which technique applies? a. Encourage the child to observe others talking. b. Include the child in small group activities. c. Give the child a small treat for speaking. d. Teach the child relaxation techniques.

ANS: C Operant conditioning involves giving positive reinforcement for a desired behavior. Treats are rewards and reinforce speech through positive reinforcement.

A nurse consistently encourages patient to do his or her own activities of daily living (ADLs). If the patient is unable to complete an activity, the nurse helps until the patient is once again independent. This nurse's practice is most influenced by which theorist? a. Betty Neuman b. Patricia Benner c. Dorothea Orem d. Joyce Travelbee

ANS: C Orem emphasizes the role of the nurse in promoting self-care activities of the patient; this has relevance to the seriously and persistently mentally ill patient.

A psychotherapist works with an anxious, dependent patient. Which strategy is most consistent with psychoanalytic psychotherapy? a. Identifying the patient's strengths and assets b. Praising the patient for describing feelings of isolation c. Focusing on feelings developed by the patient toward the therapist d. Providing psychoeducation and emphasizing medication adherence

ANS: C Positive or negative feelings of the patient toward the therapist indicate transference. Transference is a psychoanalytic concept that can be used to explore previously unresolved conflicts. The distracters relate to biological therapy and supportive psychotherapy. Use of psychoeducational materials is a common "homework" assignment used in cognitive therapy.

A college student received an invitation to attend the wedding of a close friend who lives across the country. The student is afraid of flying. Which type of therapy would be most helpful for this patient? a. Psychoanalysis b. Milieu therapy c. Systematic desensitization d. Short-term dynamic therapy

ANS: C Systematic desensitization is a type of therapy aimed at extinguishing a specific behavior, such as the fear of flying. Psychoanalysis and short-term dynamic therapy seek to uncover conflicts. Milieu therapy involves environmental factors.

Which statement about mental illness is true a) menta illness is a matter of individual nonconformity with social norms b) mental illness is present when individuals irrational and illogical behaviors occur c) mental illness is defined in relation to culture, time in history, political system and group in which it occurs d) mental illness is evaluated solely by considering individual control over behavior and appraisal of reality

C

The parent of a 4-year-old rewards and praises the child for helping a younger sibling, being polite, and using good manners. The nurse supports this use of praise related to these behaviors. These qualities are likely to be internalized and become part of which system of the personality? a. Id b. Ego c. Superego d. Preconscious

ANS: C The superego contains the "thou shalts," or moral standards internalized from interactions with significant others. Praise fosters internalization of desirable behaviors. The id is the center of basic instinctual drives, and the ego is the mediator. The ego is the problem-solving and reality-testing portion of the personality that negotiates solutions with the outside world. The preconscious is a level of awareness from which material can be retrieved easily with conscious effort. This item relates to an audience response question.

An adult says, "I never know the answers," and "My opinion doesn't count." Which psychosocial crisis was unsuccessfully resolved for this adult? a. Initiative versus guilt b. Trust versus mistrust c. Autonomy versus shame and doubt d. Generativity versus self-absorption

ANS: C These statements show severe self-doubt, indicating that the crisis of gaining control over the environment was not met successfully. Unsuccessful resolution of the crisis of initiative versus guilt results in feelings of guilt. Unsuccessful resolution of the crisis of trust versus mistrust results in poor interpersonal relationships and suspicion of others. Unsuccessful resolution of the crisis of generativity versus self-absorption results in self-absorption that limits the ability to grow as a person.

4. A patient has delusions and hallucinations. Before beginning treatment with a psychotropic medication, the health care provider wants to rule out the presence of a brain tumor. For which test will a nurse need to prepare the patient? a. Cerebral arteriogram b. Functional magnetic resonance imaging (fMRI) c. Computed tomography (CT) scan or magnetic resonance imaging (MRI) d. Positron emission tomography (PET) or single-photon emission computed tomography (SPECT)

ANS: C A CT scan and an MRI visualize neoplasms and other structural abnormalities. A PET scan, SPECT scan, and fMRI, which give information about brain function, are not indicated. An arteriogram would not be appropriate.

10. To provide comprehensive care to patients, which competency is more important for a nurse who works in a community mental health center than a psychiatric nurse who works in an inpatient unit? a. Problem-solving skills b. Calm and caring manner c. Ability to cross service systems d. Knowledge of psychopharmacology

ANS: C A community mental health nurse must be able to work with schools, corrections facilities, shelters, health care providers, and employers. The mental health nurse working in an inpatient unit needs only to be able to work within the single setting. Problem-solving skills are needed by all nurses. Nurses in both settings must have knowledge of psychopharmacology.

An older adult was stopped by police for driving through a red light. When asked for a driver's license, the adult hands the police officer a pair of sunglasses. What sign of dementia is evident? a. Aphasia b. Apraxia c. Agnosia d. Anhedonia

ANS: C Agnosia refers to the loss of sensory ability to recognize objects. Aphasia refers to the loss of language ability. Apraxia refers to the loss of purposeful movement. Anhedonia refers to a loss of joy in life.

11. A suspicious and socially isolated patient lives alone, eats one meal a day at a nearby shelter, and spends the remaining daily food allowance on cigarettes. Select the community psychiatric nurse's best initial action. a. Report the situation to the manager of the shelter. b. Tell the patient, "You must stop smoking to save money." c. Assess the patient's weight; determine the foods and amounts eaten. d. Seek hospitalization for the patient while a new plan is being formulated.

ANS: C Assessment of biopsychosocial needs and general ability to live in the community is called for before any action is taken. Both nutritional status and income adequacy are critical assessment parameters. A patient may be able to maintain adequate nutrition while eating only one meal a day. Nurses assess before taking action. Hospitalization may not be necessary.

6. A nurse must assess several new patients at a community mental health center. Conclusions concerning current functioning should be made on the basis of: a. the degree of conformity of the individual to society's norms. b. the degree to which an individual is logical and rational. c. a continuum from mentally healthy to unhealthy. d. the rate of intellectual and emotional growth.

ANS: C Because mental health and mental illness are relative concepts, assessment of functioning is made by using a continuum. Mental health is not based on conformity; some mentally healthy individuals do not conform to society's norms. Most individuals occasionally display illogical or irrational thinking. The rate of intellectual and emotional growth is not the most useful criterion to assess mental health or mental illness.

23. A cognitive strategy a nurse could use to assist a very dependent patient would be to help the patient: a. reveal dream content. b. take prescribed medications. c. examine thoughts about being autonomous. d. role model ways to ask for help from others.

ANS: C Cognitive theory suggests that one's thought processes are the basis of emotions and behavior. Changing faulty learning makes the development of new adaptive behaviors possible. Revealing dream content would be used in psychoanalytical therapy. Taking prescribed medications is an intervention associated with biological therapy. A dependent patient needs to develop independence.

A nurse gives anticipatory guidance to the family of a patient diagnosed with stage 3, mild cognitive decline Alzheimer's disease. Which problem common to that stage should the nurse address? a. Violent outbursts b. Emotional disinhibition c. Communication deficits d. Inability to feed or bathe self

ANS: C Families should be made aware that the patient will have difficulty concentrating and following or carrying on in-depth or lengthy conversations. The other symptoms are usually seen at later stages of the disease.

5. The parent of a 4-year-old rewards and praises the child for helping a younger sibling, being polite, and using good manners. A nurse supports the use of praise because, according to the Freudian theory, these qualities will likely be internalized and become part of the child's: a. id. b. ego. c. superego. d. preconscious.

ANS: C In the Freudian theory, the superego contains the "thou shalts" or moral standards internalized from interactions with significant others. Praise fosters internalization of desirable behaviors. The id is the center of basic instinctual drives, and the ego is the mediator. The ego is the problem-solving and reality-testing portion of the personality that negotiates solutions with the outside world. The preconscious is a level of awareness from which material can be easily retrieved with conscious effort.

An older adult drove to a nearby store but was unable to remember how to get home or state an address. When police intervened, they found that this adult was wearing a heavy coat and hat, even though it was July. Which stage of Alzheimer's disease is evident? a. Preclinical Alzheimer's disease b. Mild cognitive decline c. Moderately severe cognitive decline d. Severe cognitive decline

ANS: C In the moderately severe stage, deterioration is evident. Memory loss may include the inability to remember addresses or the date. Activities such as driving may become hazardous, and frustration by the increasing difficulty of performing ordinary tasks may be experienced. The individual has difficulty with clothing selection. Mild cognitive decline (early-stage) Alzheimer's can be diagnosed in some, but not all, individuals. Symptoms include misplacing items and misuse of words. In the stage of severe cognitive decline, personality changes may take place, and the patient needs extensive help with daily activities. This patient has symptoms, so the preclinical stage does not apply.

8. A patient should be considered for involuntary commitment for psychiatric care when he or she: a. is noncompliant with the treatment regimen. b. sells and distributes illegal drugs. c. threatens to harm self and others. d. fraudulently files for bankruptcy.

ANS: C Involuntary commitment protects patients who are dangerous to themselves or others and cannot care for their own basic needs. Involuntary commitment also protects other individuals in society. The behaviors described in the other options are not sufficient to require involuntary hospitalization.

Which medication prescribed to patients diagnosed with Alzheimer's disease antagonizes N-Methyl-D-Aspartate (NMDA) channels rather than cholinesterase? a. Donepezil (Aricept) b. Rivastigmine (Exelon) c. Memantine (Namenda) d. Galantamine (Razadyne)

ANS: C Memantine blocks the NMDA channels and is used in moderate-to-late stages of the disease. Donepezil, rivastigmine, and galantamine are all cholinesterace inhibitors. These drugs increase the availability of acetylcholine and are most often used to treat mild-to-moderate Alzheimer's disease

5. The nurse wants to assess for disturbances in circadian rhythms in a patient admitted for major depressive disorder. Which question best implements this assessment? a. "Do you ever see or hear things that others do not?" b. "Do you have problems with short-term memory?" c. "What are your worst and best times of day?" d. "How would you describe your thinking?"

ANS: C Mood changes throughout the day are related to circadian rhythms. Questions about sleep pattern would also be relevant to circadian rhythms. The question about seeing or hearing things is relevant to the assessment for illusions and hallucinations. The question about thinking is relevant to the assessment of thought processes. The other question is relevant to assessment of memory.

21. The nurse assesses a patient diagnosed with schizophrenia. Which assessment finding would the nurse regard as a negative symptom of schizophrenia? a. Auditory hallucinations c. Poor personal hygiene b. Delusions of grandeur d. Psychomotor agitation

ANS: C Negative symptoms include apathy, anhedonia, poor social functioning, and poverty of thought. Poor personal hygiene is an example of poor social functioning. The distracters are positive symptoms of schizophrenia. See relationship to audience response question. REF: Page 207-208 (Table 12-2) TOP: Nursing Process: Assessment

36. Which finding constitutes a negative symptom associated with schizophrenia? a. Hostility c. Poverty of thought b. Bizarre behavior d. Auditory hallucinations

ANS: C Negative symptoms include apathy, anhedonia, poor social functioning, and poverty of thought. Poor personal hygiene is an example of poor social functioning. The distracters are positive symptoms of schizophrenia. See relationship to audience response question. REF: Page 207-208 (Table 12-2) TOP: Nursing Process: Assessment

16. Operant conditioning will be used to encourage speech in a child who is nearly mute. Which technique would a nurse include in the treatment plan? a. Ignore the child for using silence. b. Have the child observe others talking. c. Give the child a small treat for speaking. d. Teach the child relaxation techniques, then coax speech.

ANS: C Operant conditioning involves giving positive reinforcement for a desired behavior. Treats are rewards to reinforce speech. Ignoring the child will not change the behavior. Having the child observe others describes modeling. Teaching relaxation techniques and then coaxing speech is an example of systematic desensitization.

17. The parent of a child diagnosed with schizophrenia tearfully asks a nurse, "What could I have done differently to prevent this illness?" Select the nurse's most caring response. a. "Although schizophrenia is caused by impaired family relationships, try not to feel guilty. No one can predict how a child will respond to parental guidance." b. "Most of the damage is done, but there is still hope. Changing your parenting style can help your child learn to cope more effectively with the environment." c. "Schizophrenia is a biological illness with similarities to diabetes and heart disease. You are not to blame for your child's illness." d. "Most mental illnesses result from genetic inheritance. Your genes are more at fault than your parenting."

ANS: C Patients and families need reassurance that the major mental disorders are biological in origin and are not the "fault" of parents. Knowing the biological nature of the disorder relieves feelings of guilt over being responsible for the illness. The incorrect responses are neither wholly accurate nor reassuring; they fall short of being reassuring and place the burden of having faulty genes on the shoulders of the parents.

3. Two hospitalized patients fight when they are in the same room. During a team meeting, a nurse asserts that safety is of paramount importance and therefore the treatment plans should call for both patients to be secluded to prevent them from injuring each other. This assertion: a. reveals that the nurse values the principle of justice. b. reinforces the autonomy of the two patients. c. violates the civil rights of the two patients. d. represents the intentional tort of battery.

ANS: C Patients have a right to treatment in the least restrictive setting. Less restrictive measures should be tried first. Unnecessary seclusion may result in a charge of false imprisonment. Seclusion removes the patient's autonomy. The principle by which the nurse is motivated is beneficence, not justice. The tort represented is false imprisonment, not battery.

9. A patient diagnosed with schizophrenia exhibits little spontaneous movement and demonstrates waxy flexibility. Which patient needs are of priority importance? a. Self-esteem c. Physiological b. Psychosocial d. Self-actualization

ANS: C Physiological needs must be met to preserve life. A patient with waxy flexibility must be fed by hand or tube, toileted, given range-of-motion exercises, and so forth to preserve physiological integrity. Higher level needs are of lesser concern. REF: Page 207 | Page 209-210 TOP: Nursing Process: Planning

16. A patient diagnosed with schizophrenia has taken fluphenazine (Prolixin) 5 mg po bid for 3 weeks. The nurse now observes a shuffling propulsive gait, a mask-like face, and drooling. Which term applies to these symptoms? a. Neuroleptic malignant syndrome c. Pseudoparkinsonism b. Hepatocellular effects d. Akathisia

ANS: C Pseudoparkinsonism induced by antipsychotic medication mimics the symptoms of Parkinson's disease. It frequently appears within the first month of treatment and is more common with first-generation antipsychotic drugs. Hepatocellular effects would produce abnormal liver test results. Neuroleptic malignant syndrome is characterized by autonomic instability. Akathisia produces motor restlessness. REF: Page 215-216 (Table 12-4) TOP: Nursing Process: Assessment

Two patients in a residential care facility have dementia. One shouts to the other, "Move along, you're blocking the road." The other patient turns, shakes a fist, and shouts, "You're trying to steal my car." What is the nurse's best action? a. Administer one dose of an antipsychotic medication to both patients. b. Reinforce reality. Say to the patients, "Walk along in the hall. This is not a traffic intersection." c. Separate and distract the patients. Take one to the day room and the other to an activities area. d. Step between the two patients and say, "Please quiet down. We do not allow violence here."

ANS: C Separating and distracting prevents escalation from verbal to physical acting out. Neither patient loses self-esteem during this intervention. Medication probably is not necessary. Stepping between two angry, threatening patients is an unsafe action, and trying to reinforce reality during an angry outburst will probably not be successful when the patients are cognitively impaired.

15. A newly admitted patient diagnosed with schizophrenia says, "The voices are bothering me. They yell and tell me I am bad. I have got to get away from them." Select the nurse's most helpful reply. a. "Do you hear the voices often?" b. "Do you have a plan for getting away from the voices?" c. "I'll stay with you. Focus on what we are talking about, not the voices. " d. "Forget the voices and ask some other patients to play cards with you."

ANS: C Staying with a distraught patient who is hearing voices serves several purposes: ongoing observation, the opportunity to provide reality orientation, a means of helping dismiss the voices, the opportunity of forestalling an action that would result in self-injury, and general support to reduce anxiety. Asking if the patient hears voices is not particularly relevant at this point. Asking if the patient plans to "get away from the voices" is relevant for assessment purposes but is less helpful than offering to stay with the patient while encouraging a focus on their discussion. Suggesting playing cards with other patients shifts responsibility for intervention from the nurse to the patient and other patients. REF: Page 206-207 | Page 212-213 (Box 12-3)

18. A patient hurriedly tells the community mental health nurse, "Everything's a disaster! I can't concentrate. My disability check didn't come. My roommate moved out, and I can't afford the rent. My therapist is moving away. I feel like I'm coming apart." Which nursing diagnosis applies? a. Decisional conflict, related to challenges to personal values b. Spiritual distress, related to ethical implications of treatment regimen c. Anxiety, related to changes perceived as threatening to psychological equilibrium d. Impaired environmental interpretation syndrome, related to solving multiple problems affecting security needs

ANS: C Subjective and objective data obtained by the nurse suggest the patient is experiencing anxiety caused by multiple threats to security needs. Data are not present to suggest Decisional conflict, Spiritual distress caused by ethical conflicts, or Impaired environmental interpretation syndrome.

25. A nurse administering psychotropic medications should be prepared to intervene when giving a drug that blocks the attachment of norepinephrine to alpha1 receptors because the patient may experience: a. increased psychotic symptoms. b. severe appetite disturbance. c. orthostatic hypotension. d. hypertensive crisis.

ANS: C Sympathetic-mediated vasoconstriction is essential for maintaining normal blood pressure in the upright position. Blockage of alpha1 receptors leads to vasodilation and orthostatic hypotension. Orthostatic hypotension may cause fainting and falls. Patients should be taught ways of minimizing this phenomenon.

25. A person received an invitation to be in the wedding of a friend who lives across the country. The individual is afraid of flying. What type of therapy should the nurse recommend? a. Psychoanalysis b. Milieu therapy c. Systematic desensitization d. Short-term dynamic therapy

ANS: C Systematic desensitization is a type of therapy aimed at extinguishing a specific behavior, such as the fear of flying. Psychoanalysis and short-term dynamic therapy are aimed at uncovering conflicts. Milieu therapy involves environmental factors.

2. A patient is hospitalized for a reaction to a psychotropic medication and then is closely monitored for 24 hours. During a predischarge visit, the case manager learns the patient received a notice of eviction on the day of admission. The most appropriate intervention for the case manager is to: a. cancel the patient's discharge from the hospital. b. contact the landlord who evicted the patient to discuss the situation. c. arrange a temporary place for the patient to stay until new housing can be arranged. d. document that the adverse medication reaction was feigned because the patient had nowhere to live.

ANS: C The case manager should intervene by arranging temporary shelter for the patient until suitable housing can be found. This is part of the coordination and delivery of services that falls under the case manager role. The other options are not viable alternatives.

A patient with fluctuating levels of awareness, confusion, and disturbed orientation shouts, "Bugs are crawling on my legs. Get them off!" Which problem is the patient experiencing? a. Aphasia b. Dystonia c. Tactile hallucinations d. Mnemonic disturbance

ANS: C The patient feels bugs crawling on both legs, even though no sensory stimulus is actually present. This description meets the definition of a hallucination, a false sensory perception. Tactile hallucinations may be part of the symptom constellation of delirium. Aphasia refers to a speech disorder. Dystonia refers to excessive muscle tonus. Mnemonic disturbance is associated with dementia rather than delirium.

7. A student nurse prepares to administer oral medications to a patient diagnosed with major depressive disorder, but the patient refuses the medication. The student nurse should: a. tell the patient, "I'll get an unsatisfactory grade if I don't give you the medication." b. tell the patient, "Refusing your medication is not permitted. You are required to take it." c. discuss the patient's concerns about the medication, and report to the staff nurse. d. document the patient's refusal of the medication without further comment.

ANS: C The patient has the right to refuse medication in most cases. The patient's reason for refusing should be ascertained, and the refusal should be reported to a unit nurse. Sometimes refusals are based on unpleasant side effects that can be ameliorated. Threats and manipulation are inappropriate. Medication refusal should be reported to permit appropriate intervention.

4. The goal for a patient is to increase resiliency. Which outcome should a nurse add to the plan of care? Within 3 days, the patient will: a. describe feelings associated with loss and stress. b. meet own needs without considering the rights of others. c. identify healthy coping behaviors in response to stressful events. d. allow others to assume responsibility for major areas of own life.

ANS: C The patient's ability to identify healthy coping behaviors indicates adaptive, healthy behavior and demonstrates an increased ability to recover from severe stress. Describing feelings associated with loss and stress does not move the patient toward adaptation. The remaining options are maladaptive behaviors.

7. A patient comments, "I never know the right answer" and "My opinion is not important." Using Erikson's theory, which psychosocial crisis did the patient have difficulty resolving? a. Initiative versus Guilt b. Trust versus Mistrust c. Autonomy versus Shame and Doubt d. Generativity versus Self-Absorption

ANS: C These statements show severe self-doubt, indicating that the crisis of gaining control over the environment is not being successfully met. Unsuccessful resolution of the crisis of Initiative versus Guilt results in feelings of guilt. Unsuccessful resolution of the crisis of Trust versus Mistrust results in poor interpersonal relationships and suspicion of others. Unsuccessful resolution of the crisis of Generativity versus Self-Absorption results in self-absorption that limits the ability to grow as a person.

16. Which individual with a mental illness may need emergency or involuntary hospitalization for mental illness? The individual who: a. resumes using heroin while still taking methadone. b. reports hearing angels playing harps during thunderstorms. c. throws a heavy plate at a waiter at the direction of command hallucinations. d. does not show up for an outpatient appointment with the mental health nurse.

ANS: C Throwing a heavy plate is likely to harm the waiter and is evidence of being dangerous to others. This behavior meets the criteria for emergency or involuntary hospitalization for mental illness. The behaviors in the other options evidence mental illness but not dangerousness.

2. In which situations does a nurse have a duty to intervene and report? Select all that apply. a. A peer is unable to write behavioral outcomes. b. A health care provider consults the Physicians' Desk Reference. c. A peer tries to provide patient care in an alcohol-impaired state. d. A team member has violated the boundaries of a vulnerable patient. e. A patient refuses a medication prescribed by a licensed health care provider.

ANS: C, D Both instances jeopardize patient safety. The nurse must practice within the Code of Ethics for Nurses. A peer being unable to write behavioral outcomes is a concern but can be informally resolved. A health care provider consulting the Physicians' Desk Reference is acceptable practice.

2. Which statements most clearly reflect the stigma of mental illness? Select all that apply. a. "Many mental illnesses are hereditary." b. "Mental illness can be evidence of a brain disorder." c. "People claim mental illness so they can get disability checks." d. "If people with mental illness went to church, they would be fine." e. "Mental illness is a result of the breakdown of the American family."

ANS: C, D, E Stigma is represented by judgmental remarks that discount the reality and validity of mental illness. Many mental illnesses are genetically transmitted. Neuroimaging can show changes associated with some mental illnesses.

Which nursing diagnoses are most applicable for a patient diagnosed with severe Alzheimer's disease? Select all that apply. a. Acute confusion b. Anticipatory grieving c. Urinary incontinence d. Disturbed sleep pattern e. Risk for caregiver role strain

ANS: C, D, E The correct answers are consistent with problems frequently identified for patients with late-stage Alzheimer's disease. Confusion is chronic, not acute. The patient's cognition is too impaired to grieve.

1. A nurse volunteers for a committee that must revise the hospital policies and procedures for suicide precautions. Which resources would provide the best guidance? Select all that apply. a. Diagnostic and Statistical Manual of Mental Disorders (fifth edition) (DSM-5) b. State's nurse practice act c. State and federal regulations that govern hospitals d. Summary of common practices of several local hospitals e. American Nurses Association Scope and Standards of Practice for Psychiatric-Mental Health Nursing

ANS: C, E Regulations regarding hospitals provide information about the minimal standard. The American Nurses Association (ANA) national standards focus on elevating practice by setting high standards for nursing practice. The DSM-5 and the state's nurse practice act would not provide relevant information. A summary of common practices of several local hospitals cannot be guaranteed to be helpful because the customs may or may not comply with laws or best practices.

A nurse listens to a group of recent retirees. One says, "I volunteer with Meals on Wheels, coach teen sports, and do church visitation." Another laughs and says, "I'm too busy taking care of myself to volunteer to help others." Which developmental task do these statements contrast? a. Trust and mistrust b. Intimacy and isolation c. Industry and inferiority d. Generativity and self-absorption

ANS: D Both retirees are in middle adulthood, when the developmental crisis to be resolved is generativity versus self-absorption. One exemplifies generativity; the other embodies self-absorption. This developmental crisis would show a contrast between relating to others in a trusting fashion and being suspicious and lacking trust. Failure to negotiate this developmental crisis would result in a sense of inferiority or difficulty learning and working as opposed to the ability to work competently. Behaviors that would be contrasted would be emotional isolation and the ability to love and commit oneself.

A patient expresses a desire to be cared for by others and often behaves in a helpless fashion. Which stage of psychosexual development is most relevant to the patient's needs? a. Latency b. Phallic c. Anal d. Oral

ANS: D Fixation at the oral stage sometimes produces dependent infantile behaviors in adults. Latency fixations often result in difficulty identifying with others and developing social skills, resulting in a sense of inadequacy and inferiority. Phallic fixations result in having difficulty with authority figures and poor sexual identity. Anal fixation sometimes results in retentiveness, rigidity, messiness, destructiveness, and cruelty. This item relates to an audience response question.

A person says, "I was the only survivor in a small plane crash. Three business associates died. I got depressed and saw a counselor twice a week for 4 weeks. We talked about my feelings related to being a survivor, and I'm better now." Which type of therapy was used? a. Milieu therapy b. Psychoanalysis c. Behavior modification d. Interpersonal psychotherapy

ANS: D Interpersonal psychotherapy returned the patient to his former level of functioning by helping him come to terms with the loss of friends and guilt over being a survivor. Milieu therapy refers to environmental therapy. Psychoanalysis would call for a long period of exploration of unconscious material. Behavior modification would focus on changing a behavior rather than helping the patient understand what is going on in his life.

Although ego defense mechanisms and security operations are mainly unconscious and designed to relieve anxiety, the major difference is that: a. defense mechanisms are intrapsychic and not observable. b. defense mechanisms cause arrested personal development. c. security operations are masterminded by the id and superego. d. security operations address interpersonal relationship activities.

ANS: D Sullivan's theory explains that security operations are interpersonal relationship activities designed to relieve anxiety. Because they are interpersonal, they are observable. Defense mechanisms are unconscious and automatic. Repression is entirely intrapsychic, but other mechanisms result in observable behaviors. Frequent, continued use of many defense mechanisms often results in reality distortion and interference with healthy adjustment and emotional development. Occasional use of defense mechanisms is normal and does not markedly interfere with development. Security operations are ego-centered. This item relates to an audience response question.

A 26-month-old displays negative behavior, refuses toilet training, and often says, "No!" Which psychosocial crisis is evident? a. Trust versus mistrust b. Initiative versus guilt c. Industry versus inferiority d. Autonomy versus shame and doubt

ANS: D The crisis of autonomy versus shame and doubt relates to the developmental task of gaining control of self and environment, as exemplified by toilet training. This psychosocial crisis occurs during the period of early childhood. Trust versus mistrust is the crisis of the infant. Initiative versus guilt is the crisis of the preschool and early-school-aged child. Industry versus inferiority is the crisis of the 6- to 12-year-old child.

A nurse supports a parent for praising a child behaving in a helpful way. When this child behaves with politeness and helpfulness in adulthood, which feeling will most likely result? a. Guilt b. Anxiety c. Humility d. Self-esteem

ANS: D The individual will be living up to the ego ideal, which will result in positive feelings about self. The other options are incorrect because each represents a negative feeling.

Which comment best indicates a patient is self-actualized? a. "I have succeeded despite a world filled with evil." b. "I have a plan for my life. If I follow it, everything will be fine." c. "I'm successful because I work hard. No one has ever given me anything." d. "My favorite leisure is walking on the beach, hearing soft sounds of rolling waves."

ANS: D The self-actualized personality is associated with high productivity and enjoyment of life. Self-actualized persons experience pleasure in being alone and an ability to reflect on events.

8. A patient has disorganized thinking associated with schizophrenia. Neuroimaging would most likely show dysfunction in which part of the brain? a. Brainstem b. Cerebellum c. Temporal lobe d. Prefrontal cortex

ANS: D The prefrontal cortex is responsible for intellectual functioning. The temporal lobe is responsible for the sensation of hearing. The cerebellum regulates skeletal muscle coordination and equilibrium. The brainstem regulates internal organs.

5. Which scenario is an example of a tort? a. The primary nurse does not complete the plan of care for a patient within 24 hours of the patient's admission. b. An advanced practice nurse recommends that a patient who is dangerous to self and others be voluntarily hospitalized. c. A patient's admission status is changed from involuntary to voluntary after the patient's hallucinations subside. d. A nurse gives an as-needed dose of an antipsychotic drug to a patient to prevent violence because a unit is short staffed.

ANS: D A tort is a civil wrong against a person that violates his or her rights. Giving unnecessary medication for the convenience of staff members controls behavior in a manner similar to secluding a patient; thus false imprisonment is a possible charge. The other options do not exemplify torts.

12. Cognitive therapy was provided for a patient who frequently said, "I'm stupid." Which statement by the patient indicates the therapy was effective? a. "I'm disappointed in my lack of ability." b. "I always fail when I try new things." c. "Things always go wrong for me." d. "Sometimes I do stupid things."

ANS: D "I'm stupid" is a cognitive distortion or irrational thought. A more rational thought is, "Sometimes I do stupid things." The latter thinking promotes emotional self-control. The incorrect options reflect irrational thinking.

11. Which belief by a nurse supports the highest degree of patient advocacy during a multidisciplinary patient care planning session? a. All mental illnesses are culturally determined. b. Schizophrenia and bipolar disorder are cross-cultural disorders. c. Symptoms of mental disorders are constant from culture to culture. d. Some symptoms of mental disorders may reflect a person's cultural patterns.

ANS: D A nurse who understands that a patient's symptoms are influenced by culture will be able to advocate for the patient to a greater degree than a nurse who believes that culture is of little relevance. All mental illnesses are not culturally determined. Schizophrenia and bipolar disorder are cross-cultural disorders, but this understanding has little relevance to patient advocacy. Symptoms of mental disorders change from culture to culture.

3. In the shift-change report, an off-going nurse criticizes a patient who wears heavy makeup. Which comment by the nurse who receives the report best demonstrates advocacy? a. "This is a psychiatric hospital. Craziness is what we are all about." b. "Let's all show acceptance of this patient by wearing lots of makeup too." c. "Your comments are inconsiderate and inappropriate. Keep the report objective." d. "Our patients need our help to learn behaviors that will help them get along in society."

ANS: D Accepting patients' needs for self-expression and seeking to teach skills that will contribute to their well-being demonstrate respect and are important parts of advocacy. The on-coming nurse needs to take action to ensure that others are not prejudiced against the patient. Humor can be appropriate within the privacy of a shift report but not at the expense of respect for patients. Judging the off-going nurse in a critical way will create conflict. Nurses must show compassion for each other.

10. An adult expresses the wish to be taken care of and often behaves in a helpless fashion. This adult has needs related to which of Freud's stages of psychosexual development? a. Latency b. Phallic c. Anal d. Oral

ANS: D According to Freud, fixation at the oral stage sometimes produces dependent infantile behaviors in adults. Latency fixations often result in a difficulty identifying with others and developing social skills, resulting in a sense of inadequacy and inferiority. Phallic fixations result in having difficulty with authority figures and poor sexual identity. Anal fixation sometimes results in retentiveness, rigidity, messiness, destructiveness, and cruelty.

6. The following patients are seen in the emergency department. The psychiatric unit has one bed available. Which patient should the admitting officer recommend for admission to the hospital? The patient who: a. is experiencing dry mouth and tremor related to side effects of haloperidol (Haldol). b. is experiencing anxiety and a sad mood after a separation from a spouse of 10 years. c. self-inflicted a superficial cut on the forearm after a family argument. d. is a single parent and hears voices saying, "Smother your infant."

ANS: D Admission to the hospital would be justified by the risk of patient danger to self or others. The other patients have issues that can be handled with less restrictive alternatives than hospitalization.

A patient diagnosed with Alzheimer's disease calls the fire department saying, "My smoke detectors are going off." Firefighters investigate and discover that the patient misinterpreted the telephone ringing. Which problem is this patient experiencing? a. Hyperorality b. Aphasia c. Apraxia d. Agnosia

ANS: D Agnosia is the inability to recognize familiar objects, parts of one's body, or one's own reflection in a mirror. Hyperorality refers to placing objects in the mouth. Aphasia refers to the loss of language ability. Apraxia refers to the loss of purposeful movements, such as being unable to dress.

11. A newly admitted patient who is acutely psychotic is a private patient of the senior psychiatrist. To whom does the psychiatric nurse who is assigned to this patient owe the duty of care? a. Health care provider b. Profession c. Hospital d. Patient

ANS: D Although the nurse is accountable to the health care provider, the agency, the patient, and the profession, the duty of care is owed to the patient.

7. A health care provider considers which antipsychotic medication to prescribe for a patient diagnosed with schizophrenia who has auditory hallucinations and poor social function. The patient is also overweight and hypertensive. Which drug should the nurse advocate? a. Clozapine (Clozaril) c. Olanzapine (Zyprexa) b. Ziprasidone (Geodon) d. Aripiprazole (Abilify)

ANS: D Aripiprazole is a third-generation atypical antipsychotic effective against both positive and negative symptoms of schizophrenia. It causes little or no weight gain and no increase in glucose, high- or low-density lipoprotein cholesterol, or triglycerides, making it a reasonable choice for a patient with obesity or heart disease. Clozapine may produce agranulocytosis, making it a poor choice as a first-line agent. Ziprasidone may prolong the QT interval, making it a poor choice for a patient with cardiac disease. Olanzapine fosters weight gain. REF: Page 215-219 (Table 12-5) TOP: Nursing Process: Planning

4. A nurse assesses a newly admitted patient diagnosed with major depressive disorder. Which statement is an example of "attending"? a. "We all have stress in life. Being in a psychiatric hospital isn't the end of the world." b. "Tell me why you felt you had to be hospitalized to receive treatment for your depression." c. "You will feel better after we get some antidepressant medication started for you." d. "I'd like to sit with you a while so you may feel more comfortable talking with me."

ANS: D Attending is a technique that demonstrates the nurse's commitment to the relationship and reduces feelings of isolation. This technique shows respect for the patient and demonstrates caring. Generalizations, probing, and false reassurances are non-therapeutic.

23. A patient has taken many conventional antipsychotic drugs over years. The health care provider, who is concerned about early signs of tardive dyskinesia, prescribes risperidone (Risperdal). A nurse planning care for this patient understands that atypical antipsychotics: a. are less costly. b. have higher potency. c. are more readily available. d. produce fewer motor side effects.

ANS: D Atypical antipsychotic drugs often exert their action on the limbic system rather than the basal ganglia. The limbic system is not involved in motor disturbances. Atypical antipsychotic medications are not more readily available. They are not considered to be of higher potency; rather, they have different modes of action. Atypical antipsychotic drugs tend to be more expensive.

1. A psychiatric nurse best implements the ethical principle of autonomy when he or she: a. intervenes when a self-mutilating patient attempts to harm self. b. stays with a patient who is demonstrating a high level of anxiety. c. suggests that two patients who are fighting be restricted to the unit. d. explores alternative solutions with a patient, who then makes a choice.

ANS: D Autonomy is the right to self-determination, that is, to make one's own decisions. When the nurse explores alternatives with the patient, the patient is better equipped to make an informed, autonomous decision. Staying with a highly anxious patient or intervening with a self-mutilating patient demonstrates beneficence and fidelity. Suggesting that two fighting patients be restricted to the unit demonstrates the principles of fidelity and justice.

14. A participant at a community education conference asks, "What is the most prevalent type of mental disorder in the United States?" Select the nurse's best response. a. "Why do you ask?" b. "Schizophrenia" c. "Affective disorders" d. "Anxiety disorders"

ANS: D The prevalence for schizophrenia is 1.1% per year. The prevalence of all affective disorders (e.g., depression, dysthymic disorder, bipolar) is 9.5%. The prevalence of anxiety disorders is 13.3%.

13. A patient has symptoms of acute anxiety related to the death of a parent in an automobile accident 2 hours earlier. The nurse should anticipate administering a medication from which group? a. Tricyclic antidepressants b. Atypical antipsychotics c. Anticonvulsants d. Benzodiazepines

ANS: D Benzodiazepines provide anxiety relief. Tricyclic antidepressants are used to treat symptoms of depression. Anticonvulsants are used to treat bipolar disorder or seizures. Antipsychotic drugs are used to treat psychosis.

11. A nurse listens to a group of recent retirees. One says, "I volunteer with Meals on Wheels, coach teen sports, and do church visitation." Another laughs and says, "I'm too busy taking care of myself to volunteer. I don't have time to help others." These comments contrast which developmental tasks? a. Trust versus Mistrust b. Industry versus Inferiority c. Intimacy versus Isolation d. Generativity versus Self-Absorption

ANS: D Both retirees are in middle adulthood, when the developmental crisis to be resolved is Generativity versus Self-Absorption. One exemplifies generativity; the other embodies self-absorption. The developmental crisis of Trust versus Mistrust would show a contrast between relating to others in a trusting fashion and being suspicious and lacking trust. Failure to negotiate the developmental crisis of Industry versus Inferiority would result in a sense of inferiority or difficulty learning and working as opposed to the ability to work competently. Behaviors that would be contrasted in the crisis of Intimacy versus Isolation would be emotional isolation and the ability to love and commit to oneself.

13. An adolescent hospitalized after a violent physical outburst tells the nurse, "I'm going to kill my father, but you can't tell anyone." Select the nurse's best response. a. "You're right. Federal law requires me to keep that information private." b. "Those kinds of thoughts will make your hospitalization longer." c. "You really should share this thought with your psychiatrist." d. "I am required to share information with the treatment team."

ANS: D Breach of nurse-patient confidentiality does not pose a legal dilemma for the nurse in this circumstance because a team approach to the delivery of psychiatric care presumes communication of patient information to other staff members to develop treatment plans and outcome criteria. The patient should know that the team may have to warn the father of the risk for harm.

10. An informal group of patients discuss their perceptions of nursing care. Which comment best indicates a patient's perception that his or her nurse is caring? a. "My nurse always asks me which type of juice I want to help me swallow my medication." b. "My nurse explained my treatment plan to me and asked for my ideas about how to make it better." c. "My nurse told me that if I take all the medicines the doctor prescribes I will get discharged soon." d. "My nurse spends time listening to me talk about my problems. That helps me feel like I'm not alone."

ANS: D Caring evidences empathic understanding, as well as competency. It helps change pain and suffering into a shared experience, creating a human connection that alleviates feelings of isolation. The incorrect options give examples of statements that demonstrate advocacy or giving advice.

24. A single parent is experiencing feelings of inadequacy related to work and family since one teenaged child ran away several weeks ago. The parent seeks the help of a therapist specializing in cognitive therapy. The psychotherapist who uses cognitive therapy will treat the patient by: a. discussing ego states. b. focusing on unconscious mental processes. c. negatively reinforcing an undesirable behavior. d. helping the patient identify and change faulty thinking.

ANS: D Cognitive therapy emphasizes the importance of changing erroneous ways people think about themselves. Once faulty thinking changes, the individual's behavior changes. Focusing on unconscious mental processes is a psychoanalytic approach. Negatively reinforcing undesirable behaviors is behavior modification, and discussing ego states relates to transactional analysis.

18. A nurse can anticipate anticholinergic side effects are likely to occur when a patient is taking: a. lithium (Lithobid). b. buspirone (BuSpar). c. risperidone (Risperdal). d. fluphenazine (Prolixin).

ANS: D Fluphenazine, a first-generation antipsychotic medication, exerts muscarinic blockade, resulting in dry mouth, blurred vision, constipation, and urinary retention. Lithium therapy is more often associated with fluid balance problems, including polydipsia, polyuria, and edema. Risperidone therapy is more often associated with movement disorders, orthostatic hypotension, and sedation. Buspirone is associated with anxiety reduction without major side effects.

Which statement about nonverbal behavior is accurate a) a calm expression means that the patient is experiencing low levels of anxiety b) patients response more consistently to therapeutic touch than to verbal interventions c) the meaning of nonverbal behaviors varies with cultural and individual differences d) eye contact is a reliable measure of the patients degree of attentiveness and engagement

C

13. In the majority culture of the United States, which individual is at greatest risk to be incorrectly labeled mentally ill? a. Person who is usually pessimistic but strives to meet personal goals b. Wealthy person who gives $20 bills to needy individuals in the community c. Person with an optimistic viewpoint about life and getting his or her own needs met d. Person who attends a charismatic church and describes hearing God's voice

ANS: D Hearing voices is generally associated with mental illness; however, in charismatic religious groups, hearing the voice of God or a prophet is a desirable event. In this situation, cultural norms vary, making it more difficult to make an accurate DSM-5 diagnosis. The individuals described in the other options are less likely to be labeled as mentally ill.

34. The nurse is developing a plan for psychoeducational sessions for several adults diagnosed with schizophrenia. Which goal is best for this group? Members will: a. gain insight into unconscious factors that contribute to their illness. b. explore situations that trigger hostility and anger. c. learn to manage delusional thinking. d. demonstrate improved social skills.

ANS: D Improved social skills help patients maintain relationships with others. These relationships are important to management of the disorder. Most patients with schizophrenia think concretely, so insight development is unlikely. Not all patients with schizophrenia experience delusions. REF: Page 211-215 (Box 12-6) TOP: Nursing Process: Planning

What is the priority need for a patient with late-stage dementia? a. Promotion of self-care activities b. Meaningful verbal communication c. Preventing the patient from wandering d. Maintenance of nutrition and hydration

ANS: D In late-stage dementia, the patient often seems to have forgotten how to eat, chew, and swallow. Nutrition and hydration needs must be met if the patient is to live. The patient is incapable of self-care, ambulation, or verbal communication.

7. On the basis of current knowledge of neurotransmitter effects, a nurse anticipates that the treatment plan for a patient with memory difficulties may include medications designed to: a. inhibit GABA production. b. increase dopamine sensitivity. c. decrease dopamine at receptor sites. d. prevent destruction of acetylcholine.

ANS: D Increased acetylcholine plays a role in learning and memory. Preventing the destruction of acetylcholine by acetylcholinesterase results in higher levels of acetylcholine, with the potential for improved memory. GABA is known to affect anxiety level rather than memory. Increased dopamine causes symptoms associated with schizophrenia or mania rather than improves memory. Decreasing dopamine at receptor sites is associated with Parkinson disease rather than improving memory.

22. A person tells a nurse, "I was the only survivor in a small plane crash, but three business associates died. I got anxious and depressed and saw a counselor three times a week for a month. We talked about my feelings related to being a survivor, and now I'm fine, back to my old self." Which type of therapy was used? a. Milieu therapy b. Psychoanalysis c. Behavior modification d. Interpersonal therapy

ANS: D Interpersonal therapy returns the patient to the former level of functioning by helping the patient come to terms with the loss of friends and guilt over being a survivor. Milieu therapy refers to environmental therapy. Psychoanalysis calls for a long period of exploration of unconscious material. Behavior modification focuses on changing a behavior rather than helping the patient understand what is going on in his or her life.

31. A patient diagnosed with schizophrenia and auditory hallucinations anxiously tells the nurse, "The voice is telling me to do things." Select the nurse's priority assessment question. a. "How long has the voice been directing your behavior?" b. "Does what the voice tell you to do frighten you?" c. "Do you recognize the voice speaking to you?' d. "What is the voice telling you to do?"

ANS: D Learning what a command hallucination is telling the patient to do is important because the command often places the patient or others at risk for harm. Command hallucinations can be terrifying and may pose a psychiatric emergency. The incorrect questions are of lesser importance than identifying the command. REF: Page 207-209 TOP: Nursing Process: Assessment

25. A patient diagnosed with schizophrenia says, "It's beat. Time to eat. No room for the cat." What type of verbalization is evident? a. Neologism c. Thought broadcasting b. Idea of reference d. Associative looseness

ANS: D Looseness of association refers to jumbled thoughts incoherently expressed to the listener. Neologisms are newly coined words. Ideas of reference are a type of delusion. Thought broadcasting is the belief that others can hear one's thoughts. REF: Page 205 TOP: Nursing Process: Assessment

10. The spouse of a patient diagnosed with schizophrenia says, "I don't understand why childhood experiences have anything to do with this disabling illness." Select the nurse's response that will best help the spouse understand this condition. a. "Psychological stress is actually at the root of most mental disorders." b. "We now know that all mental illnesses are the result of genetic factors." c. "It must be frustrating for you that your spouse is sick so much of the time." d. "Although this disorder more likely has a biological rather than psychological origin, the support and involvement of caregivers is very important."

ANS: D Many of the most prevalent and disabling mental disorders have been found to have strong biological influences. Helping the spouse understand the importance of his or her role as a caregiver is also important. Empathy is important but does not increase the spouse's level of knowledge about the cause of the patient's condition. Not all mental illnesses are the result of genetic factors. Psychological stress is not at the root of most mental disorders.

8. A patient tells a nurse, "I have psychiatric problems and am in and out of hospitals all the time. Not one of my friends or relatives has these problems." Select the nurse's best response. a. "Comparing yourself with others has no real advantages." b. "Why do you blame yourself for having a psychiatric illness?" c. "Mental illness affects 50% of the adult population in any given year." d. "It sounds like you are concerned that others don't experience the same challenges as you."

ANS: D Mental illness affects many people at various times in their lives. No class, culture, or creed is immune to the challenges of mental illness. The correct response also demonstrates the use of reflection, a therapeutic communication technique. It is not true that mental illness affects 50% of the population in any given year. Asking patients if they blame themselves is an example of probing.

12. A patient has anxiety, increased heart rate, and fear. The nurse would suspect the presence of a high concentration of which neurotransmitter? a. GABA b. Histamine c. Acetylcholine d. Norepinephrine

ANS: D Norepinephrine is the neurotransmitter associated with sympathetic nervous system stimulation, preparing the individual for the "fight or flight" response. GABA is a mediator of anxiety level. A high concentration of histamine is associated with an inflammatory response. A high concentration of acetylcholine is associated with parasympathetic nervous system stimulation.

4. The relapse of a patient diagnosed with schizophrenia is related to medication noncompliance. The patient is hospitalized for 5 days, medication is restarted, and the patient's thoughts are now more organized. The patient's family members are upset and say, "It's too soon for discharge. Hospitalization is needed for at least a month." The nurse should: a. call the psychiatrist to come explain the discharge rationale. b. explain that health insurance will not pay for a longer stay for the patient. c. call security to handle the disturbance and escort the family off the unit. d. explain that the patient will continue to improve if medication is taken regularly.

ANS: D Patients no longer stay in the hospital until every vestige of a symptom disappears. The nurse must assume responsibility to advocate for the patient's right to the least restrictive setting as soon as the symptoms are under control and for the right of citizens to control health care costs. The health care provider will use the same rationale. Shifting blame will not change the discharge. Calling security is unnecessary. The nurse can handle this matter.

20. A nurse cares for patients taking various medications, including buspirone (BuSpar), haloperidol (Haldol), trazodone (Desyrel), and phenelzine (Nardil). The nurse will order a special diet for the patient taking: a. buspirone. b. haloperidol. c. trazodone. d. phenelzine.

ANS: D Patients taking phenelzine, an MAOI, must be on a tyramine-free diet to prevent hypertensive crisis.

15. Which assessment finding for a patient living in the community requires priority intervention by the nurse? The patient: a. receives Social Security disability income plus a small check from a trust fund. b. lives in an apartment with two patients who attend day hospital programs. c. has a sibling who is interested and active in care planning. d. purchases and uses marijuana on a frequent basis.

ANS: D Patients who regularly buy illegal substances often become medication noncompliant. Medication noncompliance, along with the disorganizing influence of illegal drugs on cellular brain function, promotes relapse. The remaining options do not suggest problems.

17. A patient begins therapy with a phenothiazine medication. What teaching should a nurse provide related to the drug's strong dopaminergic effect? a. Chew sugarless gum. b. Increase dietary fiber. c. Arise slowly from bed. d. Report muscle stiffness.

ANS: D Phenothiazines are conventional antipsychotic medications that block dopamine receptors in both the limbic system and basal ganglia. Dystonia is likely to occur early in the course of treatment and is often heralded by sensations of muscle stiffness. Early intervention with an antiparkinsonian medication can increase the patient's comfort and prevent dystonic reactions.

21. A nurse psychotherapist works with an anxious, dependent patient. The therapeutic strategy most consistent with the framework of psychoanalytic psychotherapy is: a. emphasizing medication compliance. b. identifying the patient's strengths and assets. c. offering psychoeducational materials and groups. d. focusing on feelings developed by the patient toward the nurse.

ANS: D Positive or negative feelings of the patient toward the nurse or therapist represent transference. Transference is a psychoanalytic concept that can be used to explore previously unresolved conflicts. Emphasizing medication compliance is more related to biological therapy. Identifying patient strengths and assets would be consistent with supportive psychotherapy. The use of psychoeducational materials is a common "homework" assignment used in cognitive therapy.

12. An example of a breach of a patient's right to privacy occurs when a nurse: a. asks a family to share information about a patient's prehospitalization behavior. b. discusses the patient's history with other staff members during care planning. c. documents the patient's daily behaviors during hospitalization. d. releases information to the patient's employer without consent.

ANS: D The release of information without patient authorization violates the patient's right to privacy. The other options are acceptable nursing practices.

1. An 86-year-old, previously healthy and independent, falls after an episode of vertigo. Which behavior by this patient best demonstrates resilience? The patient: a. says, "I knew this would happen eventually." b. stops attending her weekly water aerobics class. c. refuses to use a walker and says, "I don't need that silly thing." d. says, "Maybe some physical therapy will help me with my balance."

ANS: D Resiliency is the ability to recover from or adjust to misfortune and change. The correct response indicates that the patient is hopeful and thinking positively about ways to adapt to the vertigo. Saying "I knew this would happen eventually" and discontinuing healthy activities suggest a hopeless perspective on the health change. Refusing to use a walker indicates denial.

28. A patient diagnosed with schizophrenia has been stable for a year however, the family now reports the patient is tense, sleeps 3 to 4 hours per night, and has difficulty concentrating. The patient says, "My computer is sending out infected radiation beams." The nurse can correctly assess this information as an indication of: a. the need for psychoeducation. c. chronic deterioration. b. medication noncompliance. d. relapse.

ANS: D Signs of potential relapse include feeling tense, difficulty concentrating, trouble sleeping, increased withdrawal, and increased bizarre or magical thinking. Medication noncompliance may not be implicated. Relapse can occur even when the patient is taking medication regularly. Psychoeducation is more effective when the patient's symptoms are stable. Chronic deterioration is not the best explanation. REF: Page 213-215 (Box 12-6) TOP: Nursing Process: Assessment

32. A patient receiving risperidone (Risperdal) reports severe muscle stiffness at 1030. By 1200, the patient has difficulty swallowing and is drooling. By 1600, vital signs are 102.8° F pulse 110 respirations 26 150/90. The patient is diaphoretic. Select the nurse's best analysis and action. a. Agranulocytosis institute reverse isolation. b. Tardive dyskinesia withhold the next dose of medication. c. Cholestatic jaundice begin a high-protein, high-cholesterol diet. d. Neuroleptic malignant syndrome notify health care provider stat.

ANS: D Taking an antipsychotic medication coupled with the presence of extrapyramidal symptoms, such as severe muscle stiffness and difficulty swallowing, hyperpyrexia, and autonomic symptoms (pulse elevation), suggest neuroleptic malignant syndrome, a medical emergency. The symptoms given in the scenario are not consistent with the medical problems listed in the incorrect options. REF: Page 210 (Table 12-3) | Page 219-220

5. A nurse at a behavioral health clinic sees an unfamiliar psychiatric diagnosis on a patient's insurance form. Which resource should the nurse consult to discern the criteria used to establish this diagnosis? a. A psychiatric nursing textbook b. NANDA International (NANDA-I ) c. A behavioral health reference manual d. Diagnostic and Statistical Manual of Mental Disorders (DSM-5)

ANS: D The DSM-5 gives the criteria used to diagnose each mental disorder. The NANDA-I focuses on nursing diagnoses. A psychiatric nursing textbook or behavioral health reference manual may not contain diagnostic criteria.

15. A nurse wants to find a description of diagnostic criteria for a person diagnosed with schizophrenia. Which resource should the nurse consult? a. U.S. Department of Health and Human Services b. Journal of the American Psychiatric Association c. North American Nursing Diagnosis Association International (NANDA-I) d. Diagnostic and Statistical Manual of Mental Disorders (DSM-5)

ANS: D The DSM-5 identifies diagnostic criteria for psychiatric diagnoses. The other sources have useful information but are not the best resources for finding a description of the diagnostic criteria for a psychiatric disorder.

14. A community psychiatric nurse facilitates medication compliance for a patient by having the health care provider prescribe depot medications by injection every 3 weeks at the clinic. For this plan to be successful, which factor will be of critical importance? a. Attitude of significant others toward the patient b. Nutritional services in the patient's neighborhood c. Level of trust between the patient and the nurse d. Availability of transportation to the clinic

ANS: D The ability of the patient to get to the clinic is of paramount importance to the success of the plan. The depot medication relieves the patient of the necessity to take medication daily, but if he or she does not receive the injection at 3-week intervals, noncompliance will again be the issue. Attitude toward the patient, trusting relationships, and nutrition are important but not fundamental to this particular problem.

3. A 26-month-old child displays negative behavior, refuses toilet training, and often shouts, "No!" when given direction. The nurse's counseling with the parent should be based on the premise that the child is engaged in which of Erikson's psychosocial crises? a. Trust versus Mistrust b. Initiative versus Guilt c. Industry versus Inferiority d. Autonomy versus Shame and Doubt

ANS: D The crisis of Autonomy versus Shame and Doubt is related to the developmental task of gaining control of self and environment, as exemplified by toilet training. This psychosocial crisis occurs during the period of early childhood. Trust versus Mistrust is the crisis of the infant, Initiative versus Guilt is the crisis of the preschool and early school-aged child, and Industry versus Inferiority is the crisis of the 6- to 12-year-old child.

23. Which documentation of a patient's behavior best demonstrates a nurse's observations? a. Isolates self from others. Frequently fell asleep during group. Vital signs stable. b. Calmer and more cooperative. Participated actively in group. No evidence of psychotic thinking. c. Appeared to hallucinate. Patient frequently increased volume on television, causing conflict with others d. Wears four layers of clothing. States, "I need protection from dangerous bacteria trying to penetrate my skin."

ANS: D The documentation states specific observations of the patient's appearance and the exact statements made. The other options are vague or subjective statements and can be interpreted in different ways.

19. A nurse cares for an older adult patient admitted for treatment of depression. The health care provider prescribes an antidepressant medication, but the dose is more than the usual adult dose. The nurse should: a. implement the order. b. consult a drug reference. c. give the usual geriatric dosage. d. hold the medication and consult the health care provider.

ANS: D The dose of an antidepressant medication for older adult patients is often less than the usual adult dose. The nurse should withhold the medication and consult the health care provider who wrote the order. The nurse's duty is to intervene and protect the patient. Consulting a drug reference is unnecessary because the nurse already knows the dose is excessive. Implementing the order is negligent. Giving the usual geriatric dose would be wrong; a nurse without prescriptive privileges cannot change the dose.

18. The spouse of a patient who has delusions asks the nurse, "Are there any circumstances under which the treatment team is justified in violating the patient's right to confidentiality?" The nurse must reply that confidentiality may be breached: a. under no circumstances. b. at the discretion of the psychiatrist. c. when questions are asked by law enforcement. d. if the patient threatens the life of another person.

ANS: D The duty to warn a person whose life has been threatened by a patient under psychiatric treatment overrides the patient's right to confidentiality. The right to confidentiality is not suspended at the discretion of the therapist or for legal investigations.

6. A nurse supports parental praise of a child who is behaving in a helpful way. When the individual behaves with politeness and helpfulness in adulthood, which feeling will most likely result? a. Guilt b. Anxiety c. Loneliness d. Self-esteem

ANS: D The individual will be living up to the ego ideal, which will result in positive feelings about self. The other options are incorrect; each represents a negative feeling.

12. A patient's history shows intense and unstable relationships with others. The patient initially idealizes an individual and then devalues the person when the patient's needs are not met. Which aspect of mental health is a problem? a. Effectiveness in work b. Communication skills c. Productive activities d. Fulfilling relationships

ANS: D The information provided centers on relationships with others, which are described as intense and unstable. The relationships of mentally healthy individuals are stable, satisfying, and socially integrated. Data are not present to describe work effectiveness, communication skills, or activities.

9. A critical care nurse asks a psychiatric nurse about the difference between a diagnosis in the Diagnostic and Statistical Manual of Mental Disorders (DSM-5) and a nursing diagnosis. Select the psychiatric nurse's best response. a. "No functional difference exists between the two diagnoses. Both serve to identify a human deviance." b. "The DSM-5 diagnosis disregards culture, whereas the nursing diagnosis includes cultural variables." c. "The DSM-5 diagnosis profiles present distress or disability, whereas a nursing diagnosis considers past and present responses to actual mental health problems." d. "The DSM-5 diagnosis influences the medical treatment; the nursing diagnosis offers a framework to identify interventions for problems a patient has or may experience."

ANS: D The medical diagnosis, defined according to the DSM-5, is concerned with the patient's disease state, causes, and cures, whereas the nursing diagnosis focuses on the patient's response to stress and possible caring interventions. Both the DSM-5 and a nursing diagnosis consider culture. Nursing diagnoses also consider potential problems.

4. In a team meeting a nurse says, "I'm concerned whether we are behaving ethically by using restraint to prevent one patient from self-mutilation while the care plan for another patient who has also self-mutilated calls for one-on-one supervision." Which ethical principle most clearly applies to this situation? a. Beneficence b. Autonomy c. Fidelity d. Justice

ANS: D The nurse is concerned about justice, that is, the fair treatment with the least restrictive methods for both patients. Beneficence means promoting the good of others. Autonomy is the right to make one's own decisions. Fidelity is the observance of loyalty and commitment to the patient.

2. The parent of an adolescent diagnosed with schizophrenia asks a nurse, "My child's doctor ordered a positron-emission tomography (PET) scan. What is that?" Select the nurse's best reply. a. "PET uses a magnetic field and gamma waves to identify problems areas in the brain. Does your teenager have any metal implants?" b. "It's a special type of x-ray image that shows structures of the brain and whether a brain injury has ever occurred." c. "PET is a scan that passes an electrical current through the brain and shows brain wave activity. PET can help diagnose seizures." d. "PET is a special scan that shows blood flow and activity in the brain."

ANS: D The parent is seeking information about PET scans. It is important to use terms the parent can understand. The correct option is the only reply that provides factual information relevant to PET scans. The incorrect responses describe magnetic resonance imaging (MRI), computed tomographic (CT) scans, and electroencephalography (EEG).

30. A patient diagnosed with schizophrenia anxiously says, "I can see the left side of my body merging with the wall, then my face appears and disappears in the mirror." While listening, the nurse should: a. sit close to the patient. b. place an arm protectively around the patient's shoulders. c. place a hand on the patient's arm and exert light pressure. d. maintain a normal social interaction distance from the patient.

ANS: D The patient is describing phenomena that indicate personal boundary difficulties and depersonalization. The nurse should maintain appropriate social distance and not touch the patient because the patient is anxious about the inability to maintain ego boundaries and merging or being swallowed by the environment. Physical closeness or touch could precipitate panic. REF: Page 204 | Page 212-213 TOP: Nursing Process: Implementation

20. A nurse sits with a patient diagnosed with schizophrenia. The patient starts to laugh uncontrollably, although the nurse has not said anything funny. Select the nurse's best response. a. "Why are you laughing?" b. "Please share the joke with me." c. "I don't think I said anything funny." d. "You're laughing. Tell me what's happening."

ANS: D The patient is likely laughing in response to inner stimuli, such as hallucinations or fantasy. Focus on the hallucinatory clue (the patient's laughter) and then elicit the patient's observation. The incorrect options are less useful in eliciting a response: no joke may be involved, "why" questions are difficult to answer, and the patient is probably not focusing on what the nurse said in the first place. REF: Page 206-207 | Page 212-213 (Box 12-3)

23. A patient diagnosed with schizophrenia says, "Contagious bacteria are everywhere. When they get in your body, you will be locked up with other infected people." Which problem is evident? a. Poverty of content c. Neologisms b. Concrete thinking d. Paranoia

ANS: D The patient's unrealistic fear of harm indicates paranoia. Neologisms are invented words. Concrete thinking involves literal interpretation. Poverty of content refers to an inadequate fund of information. REF: Page 205-206 TOP: Nursing Process: Assessment

19. A patient underwent psychotherapy weekly for 3 years. The therapist used free association, dream analysis, and facilitated transference to help the patient understand unconscious processes and foster personality changes. Which type of therapy was used? a. Short-term dynamic psychotherapy b. Transactional analysis c. Cognitive therapy d. Psychoanalysis

ANS: D The therapy described is traditional psychoanalysis. Short-term dynamic psychotherapy would last less than 1 year. Neither transactional analysis nor cognitive therapy makes use of the techniques described.

A older patient diagnosed with severe, late-stage dementia no longer recognizes family members. The family asks how long it will be before this patient recognizes them when they visit. What is the nurse's best reply? a. "Your family member will never again be able to identify you." b. "I think that is a question the health care provider should answer." c. "One never knows. Consciousness fluctuates in persons with dementia." d. "It is disappointing when someone you love no longer recognizes you."

ANS: D Therapeutic communication techniques can assist the family to come to terms with the losses and irreversibility dementia imposes on both the loved one and themselves. Two incorrect responses close communication. The nurse should take the opportunity to foster communication. Consciousness does not fluctuate in patients with dementia.

19. Which patient would a nurse refer to partial hospitalization? An individual who: a. spent yesterday in the 24-hour supervised crisis care center and continues to be actively suicidal. b. because of agoraphobia and panic episodes needs psychoeducation for relaxation therapy. c. has a therapeutic lithium level and reports regularly for blood tests and clinic follow-up. d. states, "I'm not sure I can avoid using alcohol when my spouse goes to work every morning."

ANS: D This patient could profit from the structure and supervision provided by spending the day at the partial hospitalization program. During the evening, at night, and on weekends, the spouse could assume supervision responsibilities. The patient who is actively suicidal needs inpatient hospitalization. The patient in need of psychoeducation can be referred to home care. The patient who reports regularly for blood tests and clinical follow-up can continue on the same plan.

8. A patient diagnosed with schizophrenia tells the nurse, "I eat skiller. Tend to end. Easter. It blows away. Get it?" Select the nurse's best response. a. "Nothing you are saying is clear." b. "Your thoughts are very disconnected." c. "Try to organize your thoughts and then tell me again." d. "I am having difficulty understanding what you are saying."

ANS: D When a patient's speech is loosely associated, confused, and disorganized, pretending to understand is useless. The nurse should tell the patient that he or she is having difficulty understanding what the patient is saying. If a theme is discernible, ask the patient to talk about the theme. The incorrect options tend to place blame for the poor communication with the patient. The correct response places the difficulty with the nurse rather than being accusatory. See relationship to audience response question. REF: Page 205 | Page 213-214 TOP: Nursing Process: Implementation

14. Withdrawn patients diagnosed with schizophrenia: a. are usually violent toward caregivers. b. universally fear sexual involvement with therapists. c. exhibit a high degree of hostility as evidenced by rejecting behavior. d. avoid relationships because they become anxious with emotional closeness.

ANS: D When an individual is suspicious and distrustful and perceives the world and the people in it as potentially dangerous, withdrawal into an inner world can be a defense against uncomfortable levels of anxiety. When someone attempts to establish a relationship with such a patient, the patient's anxiety rises until trust is established. There is no evidence that withdrawn patients with schizophrenia universally fear sexual involvement with therapists. In most cases, it is untrue that withdrawn patients with schizophrenia are commonly violent or exhibit a high degree of hostility by demonstrating rejecting behavior. REF: Page 211 TOP: Nursing Process: Evaluation

A patient with fluctuating levels of consciousness, disturbed orientation, and perceptual alteration begs, "Someone get these bugs off me." What is the nurse's best response? a. "No bugs are on your legs. You are having hallucinations." b. "I will have someone stay here and brush off the bugs for you." c. "Try to relax. The crawling sensation will go away sooner if you can relax." d. "I don't see any bugs, but I can tell you are frightened. I will stay with you."

ANS: D When hallucinations are present, the nurse should acknowledge the patient's feelings and state the nurse's perception of reality, but not argue. Staying with the patient increases feelings of security, reduces anxiety, offers the opportunity for reinforcing reality, and provides a measure of physical safety. Denying the patient's perception without offering help does not support the patient emotionally. Telling the patient to relax makes the patient responsible for self-soothing. Telling the patient that someone will brush the bugs away supports the perceptual distortions.

2. The spouse of a patient diagnosed with schizophrenia asks, "Which neurotransmitters are more active when a person has schizophrenia?" The nurse should state, "The current thinking is that the thought disturbances are related to increased activity of: (Select all that apply.) a. GABA." b. substance P." c. histamine." d. dopamine." e. norepinephrine."

ANS: D, E Dopamine plays a role in the integration of thoughts and emotions, and excess dopamine is implicated in the thought disturbances of schizophrenia. Increased activity of norepinephrine also occurs. Substance P is most related to the pain experience. Histamine decrease is associated with depression. Increased GABA is associated with anxiety reduction.

A male patient frequently inquired about the female student nurses boyfriend, social activities, and school experiences. Which of the following initial responses by the student best addresses the issue raised by this behavior a) the student request assignment to a patient of the same gender as the student b) she limits sharing personal information and stresses he patient centered focus of the conversations c) she tells him that she will not talk about her personal life d) she explains that is he persists in focusing on her, she cannot work with him

B

A nurse assesses a confused older adult. The nurse experiences sadness and reflects "the patient is like one of my grandparents,... so helpless" which response is the nurse demonstrating a) transference b) countertransference c) catastrophic reaction d) defensive coping reaction

B

A 49 year old patient diagnosed with schizophrenia at 22 year old is prescribed risperidone. Which nursing assessment is the priority for this patient a) monitoring blood levels to avoid toxicity b) monitoring for abnormal involuntary movements c) observing for secondary mania d) observing for memory changes

B

A black patient says to a white nurse, "Theres no sense in talking. You wouldnt understand because you live in a white world:. The nurses best action would be to a) explain "yes i do understand. Everyone goes through the same experience" b) say "please give an examples of something you think i wouldnt understand" c) reassure the patient that nurses interact with people from all cultures d) change the subject to one that is les emotionally disturbing

B

A client states "i will a;ways be alone because nobody could ever love me" the nurse recognized that the clinch is expressing what cognitive behavioral concept a) emotional consequence b) schema c) actualization d) aversion

B

As a patient disposed with a mental illness is being discharged from a facilities, a nurse invited the patient to the annual staff picnic. What is the best analysis of this scenario a) the invitation facilitated dependency on the nurse b) the nurses action ours the boundaries of the therapeutic relationship c) the invitation is therapeutic for the patents diversional activity deficit d) the nurses action assists the patients integration into community living

B

Brian a patient with schizophrenia has been ordered an antipsychotic medication. the medication will likely benefit him, but there are side effects i na small percentages of patients and ay cause the dangerous side effects. after the medication teaching, Brian is unable to identify side effects and responses "i won't have an side effects because i knew i am iron and cannot be killed" which response would be most appropriate under the circumstances a) administer the medication because Brian has made a decision to take the medication, and care should be patient centers b) petition the court to appoint a guardian as a substitute for Brian as he is unable to comprehend he proposed treatment c) administer the mediation because Brians needs for treatment is the clear priority d) withhold the medication that until Brian is able to identify the benefits and risks of both consenting and refusing consent to the medications

B

During the first interview with a patient whose child died in a car accident, the nurse feels empathetic and reaches out to take the patients hand. Select the correct analysis of the nurses behavior a) it shoes empathy and compassion. it will encourage the patient to continue to express feelings b) the gesture is premature. the patients cultural and individual interpretation of touch if unknown c) the patient will perceive the gesture as intrusive and overstepping boundaries d) the action is inappropriate. Psychiatric patients should not be touches

B

Homeostasis is promoted by interaction between the brain and internal organs and is mediated by component of the nervous system a)the central nervous system b) the autonomic nervous system c) the sympathetic nervous system d) the parasympathetic nervous system

B

Recognizing the frequency of depression among American population, the nurse should advocate for which mental health promotion intervention? a. including discussions on depression as part of school health class b. providing regular depression screening for adolescent and teenage students c. increasing the number of community based depression hotlines available to the public d. encouraging senior centers to provide information on accessing community depression resources

B

Role playing is associated with which type of psychotherapy a) psychoanalysis b) modeling c) operant conditioning d) systematic desensitization

B

The nurse is caring for an adult who experienced severe psychical abuse form the age of 2 through 12. What information should the nurse provide the client concerning the function of the "id" and the ability to function as an adult a) is has control over the emotional frustration felt as an adult b) it is the source of ones survival instincts c) it is severely damaged by abuse experienced before the age of 5 d) it provides an individual with the ability to differentiate believed and real experiences

B

The patient says 'my marriage is just great. My spouse and i always agree". The nurse observes the patients foot moving continuously as the patients twirls a shirt button. The conclusion the nurse can draw is that the patients communication is a) clear b) mixed c) precise d) inadequate

B

Two hospitalized patients fight whenever they are together. during a team meeting, a nurse asserts that safety is of paramount importance, so treatment plans should call for both patients to be secluded to keep them from injuring each other. this assertion a) reinforced the autonomy of the 2 patients b) violated the civil rights of both patients c) represents the intentional tort of battery d) correctly places emphasis on safety

B

Using Maslows model of news, the nurse providing care for an anxious client identifies which intervention as being priority a) assessing the clients ability to fulfill appropriate developmental level tasks b) assessing the client for strengths upon which a nurse client relationship can be based c) planning one-on-one time to assist in identifying the fears triggre the clients anxiety d) evaluating the client ability to learn and rain essential information regarding their current condition

B

When considering stigmatization, which statement made by the nurse demonstrates a need for immediate intervention by the nurse manager? a. "depression seems to be a real problem among the teenage population" b. " my experience has been that the Irish have a problem with alcohol abuse." c. " women are at greater risk for developing suicidal thoughts then acting on them." d. " we've admitted several military veterans with PTSD this month."

B

Which assessment should the nurse perform the evaluate the pharmacokinetic affect of a monoamine oxidase inhibitors (MAOIs) antidepressant medication a) the status of the clients appetite b) the results of the liver function test c) the level of depression exhibited by the client d) the client current sleeping patterns

B

Which drug group calls for nursing assessment for development of abnormal movement disorders among individuals who take therapeutic dosages a) SSRIs b) antipsychotics c) benzodiazepines d) tricucilc antidepressants

B

An adolescent hospitalized after a violent physical outburst tells the nurse, "I'm going to kill my father, but you can't tell anyone." Select the nurse's best response. A. "You are right. Federal law requires me to keep clinical information private." B. "I am obligated to share that information with the treatment team." C. "Those kinds of thoughts will make your hospitalization longer." D. "You should share this thought with your psychiatrist."

B Breach of nurse-patient confidentiality does not pose a legal dilemma for nurses in these circumstances because a team approach to delivery of psychiatric care presumes communication of patient information to other staff members to develop treatment plans and outcome criteria. The patient should also know that the team has a duty to warn the father of the risk for harm.

A novice nurse tells a mentor "i want to convey to my patients that i am interested n them and that i want to listen to what they have to say". Which behaviors will be helpful in meeting the nurses goal? select all that apply a) sitting behind a desk, facing the patient b) introducing se to a patient and identifying own role c0 maintaining control of discussions by asking direct questions d) using facial expressions to convey interest and encouragement e) assuming an open body posture and sometimes mirror image

B, D, E

Which nursing intervention demonstrates false imprisonment? A. A confused and combative patient says, "I'm getting out of here, and no one can stop me." The nurse restrains this patient without a health care provider's order and then promptly obtains an order. B. A patient has been irritating and attention-seeking much of the day. A nurse escorts the patient down the hall saying, "Stay in your room, or you'll be put in seclusion." C. An involuntarily hospitalized patient with suicidal ideation runs out of the psychiatric unit. The nurse rushes after the patient and convinces the patient to return to the unit. D. An involuntarily hospitalized patient with homicidal ideation attempts to leave the facility. A nurse calls the security team and uses established protocols to prevent the patient from leaving.

B False imprisonment involves holding a competent person against his or her will. Actual force is not a requirement of false imprisonment. The individual needs only to be placed in fear of imprisonment by someone who has the ability to carry out the threat. If a patient is not competent (confused), then the nurse should act with beneficence. Patients admitted involuntarily should not be allowed to leave without permission of the treatment team.

A patient diagnosed with schizophrenia believes a local minister stirred evil spirits. The patient threatens to bomb a local church. The psychiatrist notifies the minister. Select the answer with the correct rationale. The psychiatrist: A. released information without proper authorization. B. demonstrated the duty to warn and protect. C. violated the patient's confidentiality. D. avoided charges of malpractice.

B It is the health care professional's duty to warn or notify an intended victim after a threat of harm has been made. Informing a potential victim of a threat is a legal responsibility of the health care professional. It is not a violation of confidentiality.

Two hospitalized patients fight whenever they are together. During a team meeting, a nurse asserts that safety is of paramount importance, so treatment plans should call for both patients to be secluded to keep them from injuring each other. This assertion: A.reinforces the autonomy of the two patients. B.violates the civil rights of both patients. C.represents the intentional tort of battery. D.correctly places emphasis on safety.

B Patients have a right to treatment in the least restrictive setting. Safety is important, but less restrictive measures should be tried first. Unnecessary seclusion may result in a charge of false imprisonment. Seclusion violates the patient's autonomy. The principle by which the nurse is motivated is beneficence, not justice. The tort represented is false imprisonment.

A patient experiencing psychosis asks a psychiatric technician, "What's the matter with me?" The technician replies, "Nothing is wrong with you. You just need to use some self-control." The nurse who overheard the exchange should take action based on: A. the technician's unauthorized disclosure of confidential clinical information. B. violation of the patient's right to be treated with dignity and respect. C. the nurse's obligation to report caregiver negligence. D. the patient's right to social interaction.

B Patients have the right to be treated with dignity and respect. The technician's comment disregards the seriousness of the patient's illness. The Code of Ethics for Nurses requires intervention. Patient emotional abuse has been demonstrated, not negligence. An interaction with the technician is not an aspect of social interaction. The technician did not disclose clinical information.

A prescription for which medication would require the nurse to monitor the client for potential development of the side effects of hypothyroidism a) fluoxetine b) bupropion c) lithium d) imipramine

C

A nurse prepares to administer a scheduled injection of haloperidol decanoate (Haldol depot) to an outpatient with schizophrenia. As the nurse swabs the site, the patient shouts, "Stop! I don't want to take that medicine anymore. I hate the side effects." Select the nurse's best action. A. Assemble other staff for a show of force and proceed with the injection, using restraint if necessary. B. Stop the medication administration procedure and say to the patient, "Tell me more about the side effects you've been having." C. Proceed with the injection but explain to the patient that there are medications that will help reduce the unpleasant side effects. D. Say to the patient, "Since I've already drawn the medication in the syringe, I'm required to give it, but let's talk to the doctor about delaying next month's dose."

B Patients with mental illness retain their civil rights unless there is clear, cogent, and convincing evidence of dangerousness. The patient in this situation presents no evidence of dangerousness. The nurse, as an advocate and educator, should seek more information about the patient's decision and not force the medication.

Which action by a nurse constitutes a breach of a patient's right to privacy? A. Documenting the patient's daily behavior during hospitalization B. Releasing information to the patient's employer without consent C. Discussing the patient's history with other staff during care planning D. Asking family to share information about a patient's pre-hospitalization behavior

B Release of information without patient authorization violates the patient's right to privacy. The other options are acceptable nursing practices. See relationship to audience response question.

A nurse finds a psychiatric advance directive in the medical record of a patient experiencing psychosis. The directive was executed during a period when the patient was stable and competent. The nurse should: A.review the directive with the patient to ensure it is current. B.ensure that the directive is respected in treatment planning. C.consider the directive only if there is a cardiac or respiratory arrest. D.encourage the patient to revise the directive in light of the current health problem.

B The nurse has an obligation to honor the right to self-determination. An advanced psychiatric directive supports that goal. Since the patient is currently psychotic, the terms of the directive now apply.

Epidemiological studies contribute to improvement in care for individuals with mental disorders by: (SATA) a. providing information about effective nursing techniques b. identifying risk factors that contribute to the development of a disorder c. identifying individuals in the general population who will develop a specific disorder d. identifying which individuals will respond favorably to a specific treatment

B, D

An aide in a psychiatric hospital says to the nurse, "We don't have time every day to help each patient complete a menu selection. Let's tell dietary to prepare popular choices and send them to our unit." Select the nurse's best response. A. "Thanks for the suggestion, but that idea may not work because so many patients take MAOI (monoamine oxidase inhibitor) antidepressants." B. "Thanks for the idea, but it's important to treat patients as individuals. Giving choices is one way we can respect patients' individuality." C. "Thank you for the suggestion, but the patients' bill of rights requires us to allow patients to select their own diet." D. "Thank you. That is a very good idea. It will make meal preparation easier for the dietary department."

B The nurse's response to the aide should recognize patients' rights to be treated with dignity and respect as well as promote autonomy. This response also shows respect for the aide and fulfills the nurse's obligation to provide supervision of unlicensed personnel. The incorrect responses have flawed rationale or do not respect patients as individuals.

A patient cries as the nurse explores the patient's feelings about the death of a close friend. The patient sobs, "I shouldn't be crying like this. It happened a long time ago". Which responses by the nurse facilitate communication? Select all that apply a) why do you think you are so upset b) i can see that you feel sad about this situation c) the loss of a close friend is very painful for you d) crying is a way of expressing the hurt you are experiencing e) lets talk about something else because this subject is upsetting you

B, C, D

According to Freud, a client experiencing dysfunction of the conscious as apart of the mind will have problems will which aspect of memory a) recent memory b) long term memory c) all memories d) painful memories

C

As a nurse escorts a patient being discharged after treatment for major depression, the patient give the nurse a necklace with a heart pendant and says :thank you for helping mend my broken heart" Which is the nurses best response a) accepting gifts violated the policies and procedures of the facilities b) im glad you feel so much better now. thank you of the beautiful necklace c) im glad i could help you, but i can't accept the gift. My reward if seeing you with a renewed sense of hope d) helping people is what nursing is all about. its rewarding for me when patients recognize how hard we work

C

10. Tomas is a 21-year-old male with a recent diagnosis of schizophrenia. Tomas's nurse recognizes that self-medicating with excessive alcohol is common in this disease and can co-occur along with: a. Generally good health despite the mental illness. b. An aversion to drinking fluids. c. Anxiety and depression. d. The ability to express his needs.

C

7. Which therapeutic communication statement might a psychiatric-mental health registered nurse use when a patient's nursing diagnosis is altered thought processes? a. "I know you say you hear voices, but I cannot hear them." b. "Stop listening to the voices, they are NOT real." c. "You say you hear voices, what are they telling you?" d. "Please tell the voices to leave you alone for now."

C

A nurse is providing care to a 28 year old patient diagnosed with bipolar disorder who has admitted in a manic state. According to Maslow Hierarchy of Needs theory, the nurse should identify which patient symptom as having priority a) rapid, pressured speech b) grandiose thoughts c) lack of sleep d) hyperactive behavior

C

A nurse makes a post on a social media page about his peer taking care of a patient with a crime related gunshot wound during his shift in the emergency department. He does not use the name of the patients. it can be concluded that a) the nurse has not violated confidentiality laws because he did not use the patents name b) the nurse cannot be eld liable for violating confidentiality laws because he was not the primary nurse for the patient c) the nurse has violated confidentiality laws and can be held liable d) the nurse cannot be held liable because postings o na social media site are excluded from confidentiality laws

C

A patent tells the nurse "i don't think i can get out of here". Select the nurses most therapeutic response a) don't talk that way. of course you will leave here b) keep up the good work and you certainly will c) you don't think you're making progress? d) everyone feels that way sometimes

C

A patient discloses several concerns and associated feelings. If the nurse wants to seek clarification, which comment would be appropriate a)what are the common elements here b) tell me again about your experience c) am i correct in understanding that ... d) tell me everything from the beginning

C

A patient says "I'm still on restriction, but i want to attend some off unit activities. Would you ask the doctor to change my privileges?". What is the nurses best response a) why are you asking me when you're able to speak for yourself b) i will be glad to address it when i see your doctor later today c) thats a good topic for you to discuss with your doctor d) do you think you can't speak to a doctor

C

Documentation in a patients chart show, "throughout a 5 minute interaction, patient fidgeted and tapped left foot, periodically covered face with hands, and looked under the chair while stating ' i enjoy spending time with you'". Which analysis is most accurate? a) the patient is giving positive feedback about the nurses communication techniques b) the nurse if viewing the patients behavior through a cultural filter c) the patients verbal and nonverbal messages are incongruent d) the patient is demonstrating psychotic behaviors

C

During which phase of the nurse patient relationship can the nurse anticipate that identified patient issues will be explored and resolved a) pre-orientation b) orientation c) working d) termination

C

Emily is a 28 year old nurse on a psychiatric unit. She has been working with Jenna, a 27 year old who was admitted with depression. Emily and henna find they have so much in comment, including each having a 2 year old daughter and each having graduated the same high school. Emily and Jenna discuss setting together for lunch with their daughters after Jenna is discharged. This situation reflects a) successful termination b) promoting interdependence c) boundary blurring d) strong therapeutic relationship

C

How does Harry Stack Sullivan's interpersonal theory view anxiety a) an emotional experience fel gayer the age of 5 years old b) a sign of guilt in adults c) a painful emotion arising from social insecurities d) the result of trying to go beyond experiences of guilt and pain

C

James is a 42 years old patient with schizophrenia. He approaches you as you drive for day shift and anxiously reports "last night demons came to my room and tried to rape me". Which response would be most therapeutic a) there are not such things as demons, what you saw were hallucinations b) it is not possible for anyone to enter you room at night, you are safe here c) you seem very upset, please tell me more about what you experiences last night d) that much have been frightening, but well check on you at night and you'll be safe

C

Termination of a therapeutic nurse patient relationship has been successful when the nurse a) avoid upsetting the patient by shifting focus to other patients before the discharge b) gives the patient a personal telephone number and permission to call other discharge c) discusses with the patient changes that happened during the relationship and evaluate outcomes d) offers to meet the patient for coffee and conversation 3 times a week after discharge

C

The nurse is planning care for a 14 year old. The nurse demonstrated an understanding of the developmental task appropriate for this client by providing which experience a) spending one-on-one time with staff to establish trust b) providing them wit the opportunity to select which unit activities they will participate in o gain autonomy c) encouraging them to talk about their school plans to help achieve identity d) assign them to help clean up the dayroom to develop a sense of industry

C

What is the desirable outcome for the orientation stage of a nurse patient relationship? the patent will demonstrate behaviors that indicate a) sel responsibility and autonomy b)a greater sense of independence c) rapport and trust with the nurses d) resolved transference

C

Which behavior shows that a nurse values autonomy? the nurses a) suggests one-on-one supervision for a patent with suicidial thoughts b) informs a patient that the spouse will not be in during visiting hours c) discusses options and helps the patient weigh the consequences d) sets limits on a patents romantic overtures toward the nurse

C

Which issues should a nurse address during the first interview with a patient with a psychiatric disorder a) trust, congruence, attitudes, and boundaries b) goals, resistance, unconscious motivations and diversions c) relationship parameters, the contract , confidentiality, and termination d) transference, countertransference, intimacy and developing recourses

C

a voluntarily hospitalized patient tells the nurse "get me the forms for discharge, i want to leave now"/. select the nurses best response a) i will get the forms for you right now and bring them to the room b) since you signed your consent for treatment, you may leave if you desire c) i will get them for you, but lets talk about your decision to leave treatment d) i cannot give you those forms without your health care providers permission

C

which individual diagnosed with a mental illness may need involuntary hospitalization? an individual a) who has a panic attack after her child gets lost in a shopping mall b) with visions of demons emerging from cemetery plots throughout the community c) who takes 38 acetaminophen tablets after the persons stock portfolio becomes worthless d) diagnosed with major depression who stops taking prescribed antidepressant medication

C

A voluntarily hospitalized patient tells the nurse, "Get me the forms for discharge. I want to leave now." Select the nurse's best response. A. "I will get the forms for you right now and bring them to your room." B. "Since you signed your consent for treatment, you may leave if you desire." C. "I will get them for you, but let's talk about your decision to leave treatment." D. "I cannot give you those forms without your health care provider's permission."

C A voluntarily admitted patient has the right to demand and obtain release in most states. However, as a patient advocate, the nurse is responsible for weighing factors related to the patient's wishes and best interests. By asking for information, the nurse may be able to help the patient reconsider the decision. Facilitating discharge without consent is not in the patient's best interests before exploring the reason for the request.

A new antidepressant is prescribed for an elderly patient with major depression, but the dose is more than the usual geriatric dose. The nurse should: A. consult a reliable drug reference. B. teach the patient about possible side effects and adverse effects. C. withhold the medication and confer with the health care provider. D. encourage the patient to increase oral fluids to reduce drug concentration.

C The dose of antidepressants for elderly patients is often less than the usual adult dose. The nurse should withhold the medication and consult the health care provider who wrote the order. The nurse's duty is to practice according to professional standards as well as intervene and protect the patient.

After leaving work, a nurse realizes documentation of administration of a PRN medication was omitted. This off-duty nurse phones the nurse on duty and says, "Please document administration of the medication for me. My password is alpha1." The nurse receiving the call should: A. fulfill the request promptly. B. document the caller's password. C. refer the matter to the charge nurse to resolve. D. report the request to the patient's health care provider.

C Fraudulent documentation may be grounds for discipline by the state board of nursing. Referring the matter to the charge nurse will allow observance of hospital policy while ensuring that documentation occurs. Notifying the health care provider would be unnecessary when the charge nurse can resolve the problem. Nurses should not provide passwords to others.

Which individual diagnosed with a mental illness may need involuntary hospitalization? An individual: A. who has a panic attack after her child gets lost in a shopping mall B. with visions of demons emerging from cemetery plots throughout the community C. who takes 38 acetaminophen tablets after the person's stock portfolio becomes worthless D. diagnosed with major depression who stops taking prescribed antidepressant medication

C Involuntary hospitalization protects patients who are dangerous to themselves or others and cannot care for their own basic needs. Involuntary hospitalization also protects other individuals in society. An overdose of acetaminophen indicates dangerousness to self. The behaviors described in the other options are not sufficient to require involuntary hospitalization.

Which comments but the nurse demonstrate use of therapeutic communication techniques? Select all that apply a) Why do you think theses events happened to you b) there are people with problems much worse than yours c) im glad you were able to tell me how you felt about your loss d) i noticed your hands trembling when you told me about your accident e) you looked very nice today. I'm proud you took more time with your appearance

C, D

In which situations would a nurse have the duty to intervene and report? Select all that apply. A. A peer has difficulty writing measurable outcomes. B. A health care provider gives a telephone order for medication. C. A peer tries to provide patient care in an alcohol-impaired state. D. A team member violates relationship boundaries with a patient. E. A patient refuses medication prescribed by a licensed health care provider.

C, D Both keyed answers are events that jeopardize patient safety. The distracters describe situations that may be resolved with education or that are acceptable practices.

Which action violate the civil rights of a psychiatric patient? the nurse (select all that apply) a) performs mouth check after over hearing a patient say "I've been spitting out my medication" b) begins suicide precautions before a patient is assessed by the health care provider c) opens and reads a letter a patient left at the nurses station to be mailed d) places a patients expensive watch in the hospital business office sale e) restrains a patient who uses profanity when speaking to the nurse

C, E

Which actions violate the civil rights of a psychiatric patient? The nurse: (select all that apply) A. performs mouth checks after overhearing a patient say, "I've been spitting out my medication." B. begins suicide precautions before a patient is assessed by the health care provider. C. opens and reads a letter a patient left at the nurse's station to be mailed. D. places a patient's expensive watch in the hospital business office safe. E. restrains a patient who uses profanity when speaking to the nurse.

C, E The patient has the right to send and receive mail without interference. Restraint is not indicated because a patient uses profanity; there are other less restrictive ways to deal with this behavior. The other options are examples of good nursing judgment and do not violate the patient's civil rights.

Responsibility

Consumers have a personal responsibility for their own self-care and recovery, for understanding and giving meaning to their experiences, and for identifying coping strategies and healing processes to promote their own wellness

Empowering

Consumers have the authority to choose from a range of options, participate in all decisions that will affect their lives, and be educated and supported in so doing.

Self-directed

Consumers lead, control, exercise choice over, and determine their own path of recovery

A school age child tells the school nurse, "Other kids call me mean names and will not sit with me at lunch. Nobody likes me" Select the nurses most therapeutic response a) just ignore them and they will leave you alone b) you should make friends with other children c) call them names if they do that to you d) tell me more about how you feel

D

Axis V of the DSM multi axial system a) refers to medical illness b) report psychosocial and environmental problems c) indicates a need for substance abuse treatment d) describes a persons level of functioning

D

Blockage of dopamine transmission can lead to increased pituitary secretions of prolactin. In women, this hyperprolactinemia can result in a) dry mouth b) amenorrhea c) increased production of testosterone d) blurred vision

D

A Filipino American patient had a nursing diagnosis of situational low self-esteem related to poor social skills as evidence by lack of eye contact. Interventional were used to raise the patens self esteem, but after 3 weeks, the patients eye contact did not improve. what is the most accurate analysis of this scenario a) the patients eye contact should have been directly addressed by role playing to increase comfort with eye contact b) the nurse should not have independently embarked on assessment, diagnosis and planning for this patient c) the patients poor eye contact is indicative of anger and hostility that were unaddressed d) the nurse should have assessed the patients culture before making this diagnosis and plan

D

A family member of a patient with delusions of persecution asks the nurse "are there and circumstances under which the treatment team is justified in violating a patents right to confidentiality?" the nurse should reply that confidentiality may be breaches a) under no circumstances b) at the discretion of the psychiatrist c) when questions are asked by law enforcement d) if the patent threatened the life of another person

D

A nurse expresses an exclusion believe in the biological model for mental illness, when stating "its the only on i really believe". what conclusion should be drawn from this statement a) the biological model is the oldest and most reliable model for explaining mental illness b) the biological model has been proven to be successful in finding the cause of most symptoms of mental illness c) the biological model is the most popular theory among leading psychiatric and therefore the one that should be fully embraced d) in believing only in the biological model, other influences on mental health including cultural, environmental, social, and spiritual influences are not taken into account

D

A nurse interacts with a newly hospitalized patient. Select the nurses comment that applies the communication technique of "offering self" a) I've also had traumatic live experiences. Maybe it would help if i told you about them b) why d you think you had so much difficulty adjusting to this change in your life c) i hope you will feel better after getting accustomed to how this unit operates d) id like to sit with you for while to help you et comfortable talking to me

D

A nurse is talking with a patient, and 5 minutes remain in the session. The patient has ben silent most of the session. Another patient comes to the door of the room, interrupts, and says to the nurse "i really need tot talk to you". The nurse should a) invite the interrupting patient to join in the session with the current patient b) say to the interrupting patient "i am not available to talk with you at the present time" c) end the unproductive sessions with the current patient and sped time with the interrupting patient d) tell the interrupting patient "this session is 5 more minutes, then i will talk with you"

D

A nurses caring for a withdrawn, suspicious patient recognizes development of feelings of anger toward the patient. The nurse should a) suppress the angry feelings b) express the anger openly and directly wit the patient c) tell the nurse manager to assign the patient to another nurse d) discuss the anger with a clinician during a supervisory season

D

A patient says "people should be allowed to commit suicide without interference from others". A nurse replies "you're wrong. nothing is bad enough to justify death" What sis the best analysis of this interchange a) the patient is correct b) the nurse is correct c) neither person is correct d) differing values are reflected in the two statements

D

A patient says to the nurse "i dreamed i was stones. when i woke up i felt emotionally drained, as thorough i hadn't rested well". Which response should the nurse use to clarify the patients comment a) it sounds as though you were uncomfortable with the content of your dream b) i understand what you're saying. bad dreams leave me feeling tired too c) so you feel as thought you did not get enough adequate sleep last night d) can you give me an example of what you mean by "stoned"

D

4. A female patient diagnosed with schizophrenia has been prescribed a first-generation antipsychotic medication. What information should the nurse provide to the patient regarding her signs and symptoms? a. Her memory problems will likely decrease. b. Depressive episodes should be less severe. c. She will probably enjoy social interactions more. d. She should experience a reduction in hallucinations.

D

5. Which characteristic presents the greatest risk for injury to others by the patient diagnosed with schizophrenia? a. Depersonalization b. Pressured speech c. Negative symptoms d. Paranoia

D

6. Gilbert, age 19, is described by his parents as a "moody child" with an onset of odd behavior about at age 14, which caused Gilbert to suffer academically and socially. Gilbert has lost the ability to complete household chores, is reluctant to leave the house, and is obsessed with the locks on the windows and doors. Due to Gilbert's early and slow onset of what is now recognized as schizophrenia, his prognosis is considered: a. Favorable with medication b. In the relapse stage c. Improvable with psychosocial interventions d. To have a less positive outcome

D

8. When patients diagnosed with schizophrenia suffer from anosognosia, they often refuse medication, believing that: a. Medications provided are ineffective. b. Nurses are trying to control their minds. c. The medications will make them sick. d. They are not actually ill.

D

David has an overnight pass and he plans to spend this time with his sister and her family.As you meet with the patient and his sister just prior to the pass, the sister mentions that she has missed her brother and need him to babysit. You notice that the patient becomes visibly agitated when she says this. How do you balance safety and the patents right to confidentiality a) cancel the pass without explanation to the sister and reschedule it for a time when babysitting would not be required of the patient b) suggest that the sister make other arrangements for child care, but withhold the information the patient shared regarding his concerns about harming children c) speak with a patent about the safety risk involved in babysitting, seeking his permission to share this information and advising against the pass if he declines to share the information d) meet with the patents sister, sharing with her the patients previous disclosure about his anger toward children and the resultant risk that his babysitting what present

D

During an interview, a patient attempts to shift the focus from self to the nurse by asking personal questions. The nurse should respond by saying a) why do you keep asking about me b) nurses direct the interviews with patients c) do not ask questions about my personal life d) the time we spend together is to discuss your concerns

D

In a team meeting, a nurse says "I'm concerned about whether we are behaving ethically by using restraint to prevent one patient from self-mutilation, while the care plan for another self mutating patient requires one-on-one supervision. Which ethical principle most clearly applies to this situation a) beneficience b) autonomy c) fidelity d) justice

D

The family who is worried that an adult female might hurt herself. As assessment indicates moderate depression with no risk factors for suicide other than a depressed mood. the patient denies any intent or thoughts about self harm. the family agrees that the patient has not done or said anything to suggest that she might be a danger to herself. which of the following responses is consistent with the concept of the "least restrictive alternative" doctrine a) admit the patient as a temporary inpatient admission b) persuade the patent to agree to a voluntary inpatient admission c) admit the patient involuntarily to a inpatient mental health treatment unit d) arrange for an outpatient counseling appointment the next day

D

The nurse providing anticipatory operant conditioning guidance to the mother of a toddler should advise that childhood temper tantrum are best handled by which intervention a) giving the child what he/she is asking for b) scolding the child when he/she displays tantrum behaviors c) spanking the child at the onset of tantrum behaviors d) ignoring the tantrum and giving attention when the child act appropriately

D

What is the legal significance of a nurses action when a patient verbally refused medication and the nurse gives the medication over the patients objection? the nurse a) has been negligent b) committed malpractice c) fulfilled the standard of care d) can be charged with battery

D

When providing respectful nursing care, how should the nurse identify the patient and his or her observable characteristics? a. the maniac patient in room 234 b. the patient in room 234 is a maniac c. the patient in room 234 is possible a maniac d. the patient in room 234 is displaying maniac behavior

D

Which client problem would be most suited to the use of interpersonally therapy a) disturbed sensory perception b) impaired sensory perception c) medication noncompliance d) dysfunctional grieving

D

Which comment by the nurse would be appropriate to being a new nurse patient relationship a) which of you're problems is most serious b) i want you to tell me about your problems c) im an experienced nurse, you can trust me d) what would you like to tell me about yourself

D

Which documentation of a patients behavior best demonstrates a nurses observations a) isolates self from others. frequently fell asleep during groups. vital signs stable b) calmer, more cooperative. participated actively in group. no evidence of psychotic thinking c) appeared to hallucinate. frequently increased volume on television, causing conflict with others d) wore fout layers of clothing. States "i need protection from evil bacteria trying to pierce my skin"

D

Which patient meets criteria for involuntary hospitalization for psychiatric treatment? The patent who a) is noncompliant with the treatment regiment b) fraudulently flied fro bankruptcy c) sold and disrupted illegal drugs d) threatens to hard self and others

D

Which remark by a patient indicates passage from orientation to the working phase of a nurse patient relationship a) i don have any problems b) it is so difficult for me to talk about problems c) i don't know how it will help to talk to you about my problems d) i want to fin a way to deal with my anger without becoming violent

D

Which statement best describes a major difference between a DSM-IV-TR diagnosis and a nursing diagnosis a) there is more functional difference between the two. both serve to identify a human deviance b) the DSM-IV-TR diagnosis disregards culture, whereas the nursing diagnosis takes culture into account c) the DSM-IV-TR is associated with present distress or disability, whereas a nursing diagnosis considers past and present responses to actual mental health problem d) the DSM-IV-TR diagnosis distinguishes a person's specific psychiatric disorder, whereas a nursing diagnosis offers a framework for identifying the interventions for phenomena a patient is experiencing

D

a newly admitted acutely psychotic patient is a private patient of the medical director and a private pay patient. to whom does the psychiatric nurse assigned to the patient owe the duty of care a) medical director b) hospital c)profession d) patient

D

a patient with psychosis became aggressive, struck another patient, and required seclusion. select the best documentation a) patient struck another patent who attempted to leave day room to go to the bathroom. seclusion necessary at 1415. plan: maintain seclusion for 8 hours and keep these two patients away from each other for 24 hours b) seclusion ordered by physician at 1415 ager command hallucinations told the patient to hit another patient. careful monitoring of patient maintained during period of seclusion c) seclusion ordered by MD for aggressive behavior. begun at 1415. Maintained for 2 hours without incident. Outcome: patient calmer and apologized for outburst d) patient pacing, shouting. Haloperidol 5 mg given PO at 1300. At 1415 patient yelled "ill punch anyone who gets near me", and struck another patient with fist. Physically placed in seclusion at 1420. Seclusion order obtained from MD at 1430

D

which individual with mental illness may need emergency or involuntary admission? the individual who a) resumes using heroin while still taking naltrexone (ReVia) b) reports hearing angels playing harps during thunderstorms c) does not keep an outpatient appointment with the mental health nurse d) throws a heavy plate at a waiter at the direction of command hallucinations

D

A newly admitted acutely psychotic patient is a private patient of the medical director and a private-pay patient. To whom does the psychiatric nurse assigned to the patient owe the duty of care? A. Medical director B. Hospital C. Profession D. Patient

D Although the nurse is accountable to the health care provider, the agency, the patient, and the profession, the duty of care is owed to the patient.

What is the legal significance of a nurse's action when a patient verbally refuses medication and the nurse gives the medication over the patient's objection? The nurse: A. has been negligent. B. committed malpractice. C. fulfilled the standard of care. D. can be charged with battery.

D Battery is an intentional tort in which one individual violates the rights of another through touching without consent. Forcing a patient to take medication after the medication was refused constitutes battery. The charge of battery can be brought against the nurse. The medication may not necessarily harm the patient; harm is a component of malpractice.

A patient with psychosis became aggressive, struck another patient, and required seclusion. Select the best documentation. A. Patient struck another patient who attempted to leave day room to go to bathroom. Seclusion necessary at 1415. Plan: Maintain seclusion for 8 hours and keep these two patients away from each other for 24 hours. B. Seclusion ordered by physician at 1415 after command hallucinations told the patient to hit another patient. Careful monitoring of patient maintained during period of seclusion. C. Seclusion ordered by MD for aggressive behavior. Begun at 1415. Maintained for 2 hours without incident. Outcome: Patient calmer and apologized for outburst. D. Patient pacing, shouting. Haloperidol 5 mg given PO at 1300. No effect by 1315. At 1415 patient yelled, "I'll punch anyone who gets near me," and struck another patient with fist. Physically placed in seclusion at 1420. Seclusion order obtained from MD at 1430.

D Documentation must be specific and detail the key aspects of care. It should demonstrate implementation of the least restrictive alternative. Justification for why a patient was secluded should be recorded, along with interventions attempted in an effort to avoid seclusion. Documentation should include a description of behavior and verbalizations, interventions tried and their outcomes, and the name of the health care provider ordering the use of seclusion.

Which patient meets criteria for involuntary hospitalization for psychiatric treatment? The patient who: A. is noncompliant with the treatment regimen. B. fraudulently files for bankruptcy. C. sold and distributed illegal drugs. D. threatens to harm self and others.

D Involuntary hospitalization protects patients who are dangerous to themselves or others and cannot care for their own basic needs. Involuntary commitment also protects other individuals in society. The behaviors described in the other options are not sufficient to require involuntary hospitalization.

Which documentation of a patient's behavior best demonstrates a nurse's observations? A. Isolates self from others. Frequently fell asleep during group. Vital signs stable. B. Calmer; more cooperative. Participated actively in group. No evidence of psychotic thinking. C. Appeared to hallucinate. Frequently increased volume on television, causing conflict with others. D. Wore four layers of clothing. States, "I need protection from evil bacteria trying to pierce my skin."

D The documentation states specific observations of the patient's appearance and the exact statements made. The other options are vague or subjective statements and can be interpreted in different ways.

A family member of a patient with delusions of persecution asks the nurse, "Are there any circumstances under which the treatment team is justified in violating a patient's right to confidentiality?" The nurse should reply that confidentiality may be breached: A. under no circumstances. B. at the discretion of the psychiatrist. C. when questions are asked by law enforcement. D. if the patient threatens the life of another person.

D The duty to warn a person whose life has been threatened by a psychiatric patient overrides the patient's right to confidentiality. The right to confidentiality is not suspended at the discretion of the therapist or for legal investigations.

In a team meeting a nurse says, "I'm concerned about whether we are behaving ethically by using restraint to prevent one patient from self-mutilation, while the care plan for another self-mutilating patient requires one-on-one supervision." Which ethical principle most clearly applies to this situation? A.Beneficence B.Autonomy C.Fidelity D.Justice

D The nurse is concerned about justice, that is, fair distribution of care, which includes treatment with the least restrictive methods for both patients. Beneficence means promoting the good of others. Autonomy is the right to make one's own decisions. Fidelity is the observance of loyalty and commitment to the patient.

Which individual with mental illness may need emergency or involuntary admission? The individual who: A. resumes using heroin while still taking naltrexone (ReVia). B. reports hearing angels playing harps during thunderstorms. C. does not keep an outpatient appointment with the mental health nurse. D. throws a heavy plate at a waiter at the direction of command hallucinations.

D Throwing a heavy plate is likely to harm the waiter and is evidence of dangerousness to others. This behavior meets the criteria for emergency or involuntary hospitalization for mental illness. The behaviors in the other options evidence mental illness but not dangerousness. See related audience response question.

A nurse stops in to interview a patient on a medical unit and finds the patient lying supine in her bed with the head elevated at 10 degrees. Which initial response(s) would most enhance the changes of achieving a therapeutic interaction? Select all that apply a) apologize for the differential in height and process wile standing to avoid delay b) if permitted, raise the head f the best and, with the patients permission, sit on the bed c) if permitted, raise the head of the bed to approximate the nurses height while standing d) sit in whatever chair if available in the room to convey informality and increase comfort e) locate a change or stole that would place the nurse at approximately the level of the patient f) remain standing and proceed so as not to create distraction by altering the arrangements

E

Venlafaxine exerts its antidepressant effect by selectively blocking the repute of a) GABA b) dopamine c) serotonin d) norepinephrine e) c and d

E

psychiatry's definition of mental health

Evolves over time and is shaped by the prevailing culture and societal values. Reflects changes in cultural norms, society's expectations, and political climates.

GOALS FOR A TRANSFORMED MENTAL HEALTH SYSTEM IN THE UNITED STATES

Goal 1: Americans understand that mental health is essential to overall health. Goal 2: Mental health care is consumer- and family-driven. Goal 3: Disparities in mental health services are eliminated. Goal 4: Early mental health screening, assessment, and referral to services are common practice. Goal 5: Excellent mental health care is delivered, and research is accelerated. Goal 6: Technology is used to access mental health care and information.

DSM-5

diagnostic and statistical manual; written by the american psychiatric association. classifies 157 separate disorders

Axis V (DSM IV-TR)

Number corresponding to the level at which the physician, NP, PA, Psychologist sees the patient functioning in daily life. Scale goes from 1 to 100 with 100 being best possible functioning and 1 being worst possible functioning. Zero may be used to indicate insufficient data for dx. If two numbers are seen, the first usually indicates current level of functioning; the second indicates best level of functioning within the past year. Ex: "35 / 60". or they may be labeled with a time frame. Ex: "Current 35; Past year 60"

Strengths-based

Recovery is focused on valuing and building on the multiple capacities, resiliencies, talents, coping abilities, and inherent worth of individuals.

Hope

Recovery provides the essential motivating message of a better future: that people can and do overcome the barriers and obstacles that confront them. Hope is the catalyst of the recovery process.

registered nurse-psychiatric mental health (RN-PMH)

basic level: work as staff nurse, case manager, home care nurse 2 yrs of full-time work: after certified exam = RN-BC

Key points to remember

• Mental health and illness are not either/or propositions but endpoints on a continuum. • The study of epidemiology can help identify high-risk groups and behaviors. In turn, this can lead to a better understanding of the causes of some disorders. Prevalence rates help us identify the proportion of a population experiencing a specific mental disorder at a given time. • Recognizing that mental disorders are biologically based with environmental mediation, it is easier to see how they can be classified as medical disorders. • The DSM-5 provides criteria for psychiatric disorders and a basis for the development of comprehensive and appropriate interventions. • Culture influences behavior, and symptoms may reflect a person's cultural patterns or beliefs. Symptoms must be understood in terms of a person's cultural background. • Psychiatric mental health nurses work with a broad population of patients in diverse settings to promote optimal mental health. • Standardized nursing classification systems (NANDA-I, NOC, NIC) are used to form and communicate patient problems, outcomes, and interventions specific to nursing care. • Psychiatric mental health nurses function at a basic or advanced level of practice with clearly defined roles. • Due to social, cultural, scientific, and political factors, the future holds many challenges and possibilities for the psychiatric mental health nurse


Ensembles d'études connexes

NU144- Chapter 41: Management of Patients With Musculoskeletal Disorders

View Set

PSYCH107: Chapter 2 Quiz pages 54-60

View Set

Resistor Code Coding Chapter Test

View Set

Theories of Persuasion: Hourly Exam 2

View Set

You're welcome B*tches!! A Sociology of the Family Inquisitive

View Set